Unit 3 - GI

¡Supera tus tareas y exámenes ahora con Quizwiz!

55-1 An older adult is admitted to the hospital. The client's height is 5 feet, 6 inches (1.68 meters), and weight is 250 lb (113.3 kg). The nurse calculates the client's current body mass index (BMI) as _______. Fill in the blank. Round your answer to the nearest whole number.

40.0 Rationale: BMI is calculated by dividing weight in kg by height squared in meters. 113.3 kg divided by 2.83 (which is 1.68 squared) = 40.3, which rounds to 40 as the nearest whole number.

E - 52 A male client with a long history of ulcerative colitis experienced massive bleeding and had emergency surgery for creation of an ileostomy. He is very concerned that sexual intercourse with his wife will be impossible because of his new ileostomy pouch. How would the nurse respond? A - "A change in position may be what is needed for you to have intercourse with your wife." B - "You must get clearance from your primary health care provider before you attempt to have intercourse." C - "What has your wife said about your pouch system?" D - "Have you considered going to see a marriage counselor with your wife?"

A - "A change in position may be what is needed for you to have intercourse with your wife." The nurse tells the client who had an emergency ileostomy that a simple change in positioning during intercourse may alleviate apprehension and facilitate sexual relations with his wife. Suggesting marriage counseling may address the client's concerns, but it focuses on the wrong issue. The client has not stated that he has relationship problems. Asking the client what his wife has said about the pouch may address some of the client's concerns, but it similarly focuses on the wrong issue.

ATI 30 A nurse is teaching a client who has anemia and a new prescription for a liquid iron supplement. Which of the following information should the nurse include in the teaching? Select all A - "Add foods that are high in fiber to your diet." B - "Rinse your mouth after taking the medication." C - "Expect stool to be green or black in color." D - "Take the medication with a glass of milk." E - "Add red meat to your diet."

A - "Add foods that are high in fiber to your diet." B - "Rinse your mouth after taking the medication." C - "Expect stool to be green or black in color." E - "Add red meat to your diet." Rational A - Fiber can prevent constipation B - Iron can stain teeth when in liquid form C - Dark green or black stool can occur when taking iron supplements D - Dairy products and caffeine can decrease the absorption of iron supplements. Iron supplements are maximally absorbed when taken on an empty stomach or 1 hr before meals E - Red meat are high in iron and recommended

ATI - P A nurse is planning teaching for a client who has been prescribed loperamide to treat diarrhea. Which of the following statements should the nurse plan to include? A - "Avoid driving or activities requiring alertness." B - "If you miss a dose, double the next dose." C - "Rinsing the mouth with alcohol-based mouthwash can reduce dryness." D - "Having one glass of wine each night can improve medication effectiveness."

A - "Avoid driving or activities requiring alertness." Rational A - Loperamide can cause drowsiness and dizziness. The client should avoid driving or activities requiring alertness while taking the drug. B - If the client misses a dose, they should not take skipped doses or double a dose. The client should continue the drug as prescribed, either taking it at timed intervals or with each incidence of diarrhea. C - For dry mouth, the nurse should instruct the client to use oral rinses, sugarless gum, or candy for relief. Alcohol-based mouthwashes can have more of a drying effect than special oral rinses designed for dry mouth. D - The client should avoid alcohol consumption while taking loperamide as it has the same CNS depressant effect as the drug. The client should also avoid using histamines, sedatives, or opioid analgesics while taking loperamide for this reason.

E - 52 The nurse is teaching a family how to prevent the client's transmission of gastroenteritis at home. Which instructions will the nurse include in the health teaching? Select all that apply. A - "Clean and disinfect all bathrooms often to avoid stool exposure." B - "Everyone in the home should wash their hands for at least 30 seconds with an antibacterial soap using friction." C - "Contact the primary health care provider if GI symptoms last more than 3 days." D - "Wear a mask at home to prevent transmission of the disease." E - "Do not share dishes, glasses, and silverware among members of the family."

A - "Clean and disinfect all bathrooms often to avoid stool exposure." B - "Everyone in the home should wash their hands for at least 30 seconds with an antibacterial soap using friction." C - "Contact the primary health care provider if GI symptoms last more than 3 days." E - "Do not share dishes, glasses, and silverware among members of the family." All of these interventions are important to prevent the spread of gastroenteritis except there is no need to wear a mask because the disease is spread via the fecal-oral route rather than by droplets.

E - 55 The nurse is teaching a group of clients on the complications of obesity that develop when weight is not controlled through diet and exercise. Which lifestyle change does the nurse emphasize? Select all that apply. A - "Consume a diet that is moderate in salt and sugar and low in fats and cholesterol." B - "Begin a weight-training program for building muscle mass." C - "Liquid dietary supplements can be substituted safely for solid food." D - "Engage in moderate physical activity for at least 30 minutes each day." E - "Foods eaten away from home tend to be higher in fat than foods made at home." F - "Eat a variety of foods, especially grain products, vegetables, and fruits."

A - "Consume a diet that is moderate in salt and sugar and low in fats and cholesterol." D - "Engage in moderate physical activity for at least 30 minutes each day." E - "Foods eaten away from home tend to be higher in fat than foods made at home." F - "Eat a variety of foods, especially grain products, vegetables, and fruits." Lifestyle changes the nurse emphasizes include consuming a diet that is moderate in salt and sugar and low in fats and cholesterol, and moderate physical activity for at least 30 minutes each day. Eating a variety of foods, especially grain products, vegetables, and fruits, helps people achieve weight loss. These are foods that "burn" more calories as they are metabolized. Many foods eaten away from home tend to be higher in fat than foods prepared at home.A weight-training program for building muscle mass does not need to be included in a weight loss program. Muscle weighs more and tends to increase weight in people who weight-train. Liquid dietary supplements cannot safely be substituted for solid food while attempting to lose weight. These types of liquid diets should be carefully supervised by a health care provider with special education in weight management.

ATI 54 A nurse is completing nutrition teaching for a client who has pancreatitis. Which of the following statements by the client indicates an understanding of the teaching? Select all A - "I plan to eat small, frequent meals." B - "I will eat easy-to-digest foods with limited spice." C - "I will use skim milk when cooking." D - "I plan to drink regular cola." E - "I will limit alcohol intake to two drinks per day."

A - "I plan to eat small, frequent meals." B - "I will eat easy-to-digest foods with limited spice." C - "I will use skim milk when cooking." Rational D - caffeine free beverages are recommended E - Should avoid any alcohol intake

ATI A nurse is providing discharge teaching for a client who has peptic ulcer disease and a new prescription for famotidine. Which of the following statements by the client indicates an understanding of the teaching? A - "I should take this medication at bedtime." B - "I should expect this medication to discolor my stools." C - "I will drink iced tea with my meals and snacks." D - "I will monitor my blood glucose level regularly while taking this medication."

A - "I should take this medication at bedtime." Rational A - The nurse should instruct the client to take the medication at bedtime to inhibit the action of histamine at the H2-receptor site in the stomach. B - Famotidine can cause constipation, anorexia, and cramps but does not discolor stools. Bismuth, another medication prescribed for peptic ulcer disease, can turn the client's stools black. C - The nurse should instruct the client to avoid excessive intake of caffeinated beverages, such as coffee and tea. D - There is no indication that famotidine therapy has an adverse effect on blood glucose levels. Therefore, it is unnecessary for the client to monitor daily glucose levels.

ATI A nurse is providing discharge teaching for a client who has GERD. Which of the following statements by the client indicates an understanding of the teaching? A - "I will decrease the amount of carbonated beverages I drink." B - "I will avoid drinking liquids for 30 minutes after taking a chewable antacid tablet." C - "I will eat a snack before going to bed." D - "I will lie down for at least 30 minutes after eating each meal."

A - "I will decrease the amount of carbonated beverages I drink." Rational A - The nurse should instruct the client to limit or eliminate fatty foods, coffee, tea, carbonated beverages, and chocolate from the diet because they irritate the lining of the stomach. B - The nurse should instruct the client to drink a glass of water immediately after taking an antacid tablet. C - The nurse should instruct the client to eat four to six small meals per day and avoid snacking before bed. D - The nurse should instruct the client to sit upright for 1 to 2 hr after meals.

ATI 52 A nurse in a clinic is teaching a client who has ulcerative colitis. Which of the following statements by the client indicated understanding of the teaching? A - "I will plan to limit fiber in my diet." B - "I will restrict fluid intake during meals." C - "I will switch to black tea instead of drinking coffee." D - "I will try to eat cold foods rather than warm when my stomach feels upset."

A - "I will plan to limit fiber in my diet." Rational A - low fiber to reduce inflammation B - dumping syndrome should avoid fluids with meals C - Caffeine can increase diarrhea and cramping D - Avoid cold foods because they can increase intestinal motility and cause exacerbations

ATI 29 A nurse is teaching a client about probiotic supplements. Which of the following information should the nurse include? Select all A - "Probiotics are micro-organism that are normally found in the GI tract." B - "Probiotics are used to treat C-Diff." C - "Probiotics are used to treat benign prostatic hyperplasia." D - "You can experience bloating while taking probiotic supplements." E - "If you are prescribed antibiotics, you should take ti at the same time you take your probiotic supplement."

A - "Probiotics are micro-organism that are normally found in the GI tract." B - "Probiotics are used to treat C-Diff." D - "You can experience bloating while taking probiotic supplements." Rational A - Probiotics consists of lactobacilli, bifidobacteria, and saccharomyces boulardii, which are found in the digestive tract B - Probiotics are used to treat a number of GI conditions, included IBS, diarrhea associated with C-Diff and UC C - Saw palmetto is a supplement that clients might use to treat benign prostatic hyperplasia D - The client should take the probiotic supplement at least 2 hr after taking an antibiotic or antifungal med. Antibiotics and antifungal med destroy bacteria and yeast found in probiotic supplements.

E - 53 The family of a client who has hepatic encephalopathy asks why the client is restricted to moderate amounts of dietary protein and has to take lactulose. What is an appropriate response by the nurse? A - "These interventions help to reduce the ammonia level." B - "These interventions help to prevent heart failure." C - "These interventions help the client's jaundice improve." D - "These interventions help to prevent nausea and vomiting."

A - "These interventions help to reduce the ammonia level." The client's high ammonia level has caused encephalopathy which can become so severe that it causes death. These interventions help to reduce ammonia in the body so that this condition does not worsen.

ATI - P A nurse is teaching a client who has a new prescription for sulfasalazine. Which of the following statements should the nurse make? A - "Use sunscreen and protective clothing while taking sulfasalazine to prevent sunburn." B - "The medication can stain your contact lenses green." C - "The medication can color your urine dark brown." "D - Take an iron supplement when you take sulfasalazine to prevent anemia."

A - "Use sunscreen and protective clothing while taking sulfasalazine to prevent sunburn." Rational A - Photosensitivity is a possible adverse effect of sulfasalazine that makes the skin sensitive to light. The nurse should instruct the client to wear sunscreen and protective clothing when outdoors to prevent burning. B - Sulfasalazine can stain contact lenses a yellow color. C -Sulfasalazine can color the urine orange-yellow. D - Sulfasalazine interacts with iron and folic acid supplements and will reduce their absorption. The client should take iron and folic acid supplements separate from sulfasalazine.

E - 55 The nurse is performing a health assessment on a client with obesity who states, "I have tried many diets in an effort to lose weight, but have been unsuccessful." How does the nurse assess whether the client's response to stress is related to the client's obesity? A - "What do you usually do that helps to relieve stress?" B - "What is it about your size that causes you to feel uncomfortable?" C - "Do you usually use alcohol or drugs when you feel stressed?" D - "Do you have a history of mental health concerns?"

A - "What do you usually do that helps to relieve stress?" The appropriate way to assess the client's response to stress is to ask an open-ended type of question because it cannot be answered with a "yes" or "no." From that answer, the nurse can better determine if the client eats in response to stress.Asking the client about mental health problems may cause the client to feel uncomfortable with the assessment and shut down. This subject can be more gently introduced later, if needed, based on the nurse's initial assessment. More effective methods can be used to determine the client's alcohol and drug habits. Having the client tell you what makes him or her uncomfortable about size will only cause the client to restate the obvious. It does not determine the effect that stress has on the client.

E - 48 While working in the outpatient procedure unit, the RN is assigned to four clients. Which client will the nurse assess first? A - A 51 year old who just had an endoscopic retrograde cholangiopancreatography (ERCP). B - A 58 year old who has just arrived for a sigmoidoscopy. C - A 60 year old with questions about an endoscopic ultrasound examination. D - A 54 year old who is ready for discharge following a colonoscopy.

A - A 51 year old who just had an endoscopic retrograde ERCP requires conscious sedation, so the client needs immediate assessment of respiratory and cardiovascular status. The endoscopic procedure and nursing care for a client having an ERCP are similar to those for the EGD procedure, except that the endoscope is advanced farther into the duodenum and into the biliary tract.A 54-year-old client being discharged after a colonoscopy, a 58-year-old client who is going to have a sigmoidoscopy, and a 60-year-old client with questions about an endoscopic ultrasound examination are not at risk for depressed respiratory status. They can all be seen following the client who just had an ERCP.

E - 55 An RN receives the change-of-shift report about these four clients. Which client will the nurse assess first? A - A 75 year old with dementia on nasogastric feedings with a respiratory rate of 38 breaths/min B - A 50 year old receiving total parenteral nutrition (TPN) with a blood glucose (BG) level of 300 mg/dL (16.7 mmol/L) C - A 30 year old admitted 2 hours ago with malnutrition associated with malabsorption syndrome D - A 45 year old who had gastric bypass surgery and is reporting severe incisional pain

A - A 75 year old with dementia on nasogastric feedings with a respiratory rate of 38 breaths/min The nurse first assesses the client with dementia who has a respiratory rate of 38 breaths/min. This client needs immediate respiratory assessment and interventions. Aspiration is a major complication in clients receiving tube feedings, especially in clients with an altered level of consciousness.The client with malnutrition associated with malabsorption syndrome, the client with incisional pain from gastric bypass surgery, and the client receiving TPN with a BG of 300 mg/dL (16.7 mmol/L) all need assessments and/or interventions by the RN, but maintaining respiratory function in the client with tachypnea is the highest priority.

ATI 28 A nurse is caring for four client who have peptic ulcer disease. The nurse should identify misoprostol is contraindicated for which of the following clients? A - A client who is pregnant B - A client who has osteoarthritis C - A client who has a kidney stone D - A client who has a urinary tract infection

A - A client who is pregnant Rational A - Misoprostol can induce labor B - osteoarthritis is not a contraindication C - kidney stones is not a contraindication D - UTI is not a contraindication

E - 52 The nurse is instructing a client with recently diagnosed diverticular disease about diet. What food does the nurse suggest the client include? A - A slice of 5-grain bread B - Strawberries (1 cup [160 g]) C - Tomato (1 medium) D - Chuck steak patty (6 ounces [170 g])

A - A slice of 5-grain bread The nurse suggests to the client with recently diagnosed diverticular disease to include a slice of 5-grain bread in the diet. Whole-grain breads are recommended to be included in the diet of clients with diverticular disease because cellulose and hemicellulose types of fiber are found in them. Dietary fat would be reduced in clients with diverticular disease.If the client wants to eat beef, it must be of a leaner cut. Foods containing seeds, such as strawberries, must be avoided. Tomatoes would also be avoided unless the seeds are removed. The seeds may block diverticula in the patient and present problems leading to diverticulitis.

ATI - P A nurse is caring for a client who is taking allopurinol to treat gout and has a new prescription for azathioprine to treat ulcerative colitis. For which of the following reasons should the nurse clarify these prescriptions with the provider? A - Allopurinol delays the conversion of azathioprine and can lead to toxicity. B - Azathioprine increases the effectiveness of allopurinol. C - Allopurinol increases the metabolism of azathioprine and can require an increased dosage. D - Azathioprine decreases the effectiveness of allopurinol.

A - Allopurinol delays the conversion of azathioprine and can lead to toxicity. Rational A - Allopurinol delays the conversion of azathioprine to an inactive form and can lead to toxicity. If used concurrently, the dose of azathioprine must be reduced. B - Azathioprine does not increase the effectiveness of allopurinol. It can increase the risk of bone marrow suppression, which is an adverse effect of allopurinol. C - Allopurinol decreases the rate of metabolism of azathioprine, which can cause the drug levels to build up in the body. If used concurrently, the dose of azathioprine must be reduced. D - Azathioprine does not decrease the effectiveness of allopurinol.

ATI 48 A nurse is caring for a client who has a new diagnosis of GERD. The nurse should expect prescription for which of the following meds select all A - Antacids B - Histamine 2 receptor antagonist C - opioid analgesic D - Fiber laxative E - Proton pump inhibitor

A - Antacids B - Histamine 2 receptor antagonist E - Proton pump inhibitor Rational B - Opioid analgesics are not effective in treating GERD C - Fiber laxatives are not effective in treating

E - 48 The nurse is assessing an alert client who had abdominal surgery yesterday. Which assessment method will the nurse use to most accurately determine whether peristalsis has resumed? A - Ask if the client has passed flatus (gas) within the previous 12 to 24 hours. B - Perform auscultation with the diaphragm of the stethoscope. C - Listen for bowel sounds in all abdominal quadrants. D - Count the number of bowel sounds in each abdominal quadrant over 1 minute.

A - Ask if the client has passed flatus (gas) within the previous 12 to 24 hours. The best and most reliable method for assessing the return of peristalsis following abdominal surgery is the client's report of passing flatus within the past 8 hours or stool within the past 12 hours.Although auscultation and counting the number of sounds can help to assess for bowel activity, it is not the most reliable method.

ATI 46 A nurse is having difficulty arousing a client following an EGD. Which of the following is the priority action by then nurse A - Assess the client's airway B - Allow the client to sleep C - Prepare to administer an antidote to the sedative D - Evaluate pre procedural laboratory findings.

A - Assess the client's airway Rational A- ABC's B - continue to allow the client to rest. However, another action is the priority. C - Prepare to admin an antidote however, another action is the priority D - Evaluate the pre procedure lab finding. However, another actoni is the priority.

E - 54 The nurse is caring for a client who had a Whipple surgical procedure yesterday. For what serum laboratory test results would the nurse want to monitor frequently and carefully? A - Blood glucose B - Blood urea nitrogen C - Phosphorus D - Platelet count

A - Blood glucose During a Whipple procedure, most or all of the pancreas is manipulated, stressed, and possibly removed. Therefore, the client is at risk for hyperglycemia or hypoglycemia and blood glucose would need careful monitoring with a possible need for treatment.

ATI - 46 A nurse is reveiwing bowel prep using polyethylene glycol with a client scheduled for a colonoscopy. Which of the following instructions should the nurse include A - Check with the provider about taking current medications when consuming bowel prep B - Consume a normal diet until starting the bowel prep C - Expect the bowel prep to not begin acting until the day after all the rep is consumed D - Discontinue the bowel prep once ddeces starts to be expelled.

A - Check with the provider about taking current medications when consuming bowel prep Rational A - some meds can be withheld when taking polyethylene glycol due to their lack of absorption B - clear liquid diet prior to starting the bowel prep C - actions of polyethylene glycol begin within 2-3 hours after consumption D - Consume the full amount prescribed

E - 52 A client with ulcerative colitis (UC) is prescribed sulfasalazine and corticosteroid therapy. As the disease improves, what change does the nurse expect in the client's medication regimen? A - Corticosteroid therapy will be tapered. B - Corticosteroid therapy will be stopped. C - Sulfasalazine will be stopped. D - Sulfasalazine will be tapered.

A - Corticosteroid therapy will be tapered. The nurse expects that corticosteroid therapy will be tapered as the UC improves in the client who was taking both sulfasalazine and corticosteroids. Once clinical improvement has been established, corticosteroids are tapered over a 2- to 3-month period.Stopping corticosteroid therapy abruptly is unsafe—steroids must be gradually decreased in patients. Usually the amount that they have been taking dictates how quickly or slowly they can be stopped. Sulfasalazine therapy will be taken on a long-term basis. It may be increased or decreased, depending on the patient's symptoms, but will likely never be stopped. These decisions are made over a long period of therapy.

ATI 55 A nurse is caring for a client who has cirrhosis. Which of the following medications can thenurse expect to administer to this client? Select all A - Diuretic B - Beta-blocking agent C - Opioid analgesic D - Lactulose E - Sedative

A - Diuretic B - Beta-blocking agent D - Lactulose Rational A - Diuretics facilitate excretion of excess fluid B - prevent bleeding from varices C - Opioid are metabolized in the liver D - Laculure is prescribed for a client who has cirrhosis to aid in the elimination of ammonia in the stool E - sedative are metabolized in the liver

ATI - P A nurse is assessing a client who was administered ondansetron IV 1 hr ago. Which of the following findings should the nurse recognize as an adverse effect of this drug? A - Dizziness B - Bradycardia C - Tardive dyskinesia D - Dyspepsia

A - Dizziness Rational A - Dizziness and lightheadedness are the most common adverse effects of ondansetron. B - Torsade De Pointes, a form of tachycardia, is a potential adverse effect of ondansetron, not bradycardia. C - Tardive dyskinesia is an adverse effect of metoclopramide. D - Dyspepsia is an adverse effect of misoprostol.

ATI 51 A nurse is planning care for a client who has a small bowel obstruction and a nasogastric (NG) tube in place. Which of the following interventions should the nurse include? Select all A - Document the NG drainage with the client's output B - Irrigate the NG tube every 8 hrs C - Assess bowel sounds D - Provide oral hygiene every 2 hr E - Monitor NG tube for placement

A - Document the NG drainage with the client's output C - Assess bowel sounds D - Provide oral hygiene every 2 hr E - Monitor NG tube for placement Rational B - irrigate every 4 hrs to maintain patency

E - 54 The nurse is reviewing laboratory results of a client recently admitted with a diagnosis of acute pancreatitis. Which values would the nurse expect to be elevated? Select all that apply. A - Elastase B - Amylase C - Glucose D - Lipase E - Trypsin F - Calcium

A - Elastase B - Amylase C - Glucose D - Lipase E - Trypsin The client who has acute pancreatitis experiences elevation of all pancreatic enzymes and glucose. The serum calcium level is usually decreased (rather than elevated) because the release of fatty acids combined with available calcium.

ATI 51 A nurse is caring for a client who has a small bowel obstruction from adhesions. Which of the following findings are consistent with this diagnosis? Select all A - Emesis greater than 500 mL with a fecal odor B - Report of spasmodic abdominal pain C - High-pitched bowel sounds D - Abdomen flat with rebound tenderness to palpation E - Laboratory findings indicating metabolic acidosis

A - Emesis greater than 500 mL with a fecal odor B - Report of spasmodic abdominal pain C - High-pitched bowel sounds Rational D - abdominal distention is common E - Metabolic alkalosis

E - 53 When caring for a client with portal hypertension, the nurse assesses for which potential complications? Select all that apply. A - Esophageal varices B - Ascites C - Hematuria D - Hemorrhoids E - Fever

A - Esophageal varices B - Ascites D - Hemorrhoids Potential complications of portal hypertension include esophageal varices, ascites, and hemorrhoids. Portal hypertension results from increased resistance to or obstruction (blockage) of the flow of blood through the portal vein and its branches. The blood meets resistance to flow and seeks collateral (alternative) venous channels around the high-pressure area. Veins become dilated in the esophagus (esophageal varices), rectum (hemorrhoids), and abdomen (ascites due to excessive abdominal [peritoneal] fluid).Hematuria may indicate insufficient production of clotting factors in the liver and decreased absorption of vitamin K. Fever indicates an inflammatory process.

ATI 50 A nurse is planning care for a client who has acute gastritis. Which of the following nursing interventions should the nurse include in the plan of care? select all A - Exualute intake and output B - Monitor laboratory reports of electrolytes C - Provide three large meals a day D - Administer ibuprofen for pain E - Observe stool characteristics.

A - Exualute intake and output B - Monitor laboratory reports of electrolytes E - Observe stool characteristics. Rational A - to prevent electrolyte loss and dehydration B - from fluid loss and dehydration C - eat small, frequent meals D - Avoid ibuprofen and NSAID because of it erosive capabilities E - report indicates of blood in stool, can indicate GI bleeding

ATI A nurse is assessing a client who has Crohn's disease. Which of the following findings should the nurse expect? A - Fatty diarrheal stools B - Hyperkalemia C - Weight gain D - Sharp epigastric pain

A - Fatty diarrheal stools Rational A - Steatorrhea, or fatty stool, is an expected finding in a client who has Crohn's disease. B - Hypokalemia is an expected finding in a client who has Crohn's disease. C - Weight loss is an expected finding in a client who has Crohn's disease. D - Abdominal pain in the right lower quadrant is an expected finding in a client who has Crohn's disease.

ATI 22 A nurse is assessing a child who has a rotavirus infection. Which of the following are expected findings? Select all A - Fever B - Vomiting C - Watery stools D - Bloody stools E - Confusion

A - Fever B - Vomiting C - Watery stools Rational A - fever is a manifestation of rotavirus infection B - Vomiting for approximately 2 days is a manifestation of rotavirus infection C - Foul smelling, water stool is a manifestation of rotavirus infection D - Blood stool are manifestation of E. coli E - Confusion is a manifestation of Salmonella

ATI - P A nurse is teaching a client who has a new prescription for methotrexate. The nurse should instruct the client to monitor for manifestations of which of the following conditions? A - Gout B - Constipation C - Insomnia D - Hirsutism

A - Gout Rational A - An adverse effect of methotrexate is hyperuricemia, which causes gout. The nurse should instruct the client to drink plenty of fluids to minimize this effect, and to report edema or pain in the joints. B - An adverse effect of methotrexate is diarrhea, not constipation. The nurse should instruct the client to monitor for diarrhea as well as bloody stools. C - An adverse effect of methotrexate is drowsiness, not insomnia. The nurse should also warn the client that methotrexate can cause seizures. D - An adverse effect of methotrexate is alopecia, not hirsutism. The nurse should inform the client about possible hair loss and assist the client with coping strategies.

ATI 29 A nurse is planning to administer ondansetron to a client. For which of the following adverse effects of ondansetron should the nurse monitor? Select all A - Headache B - Diarrhea C - Shortened PR interval D - Hyperglycemia E - Prolonged QT interval

A - Headache B - Diarrhea E - Prolonged QT interval Rational A - Headache is a common adverse effect B - diarrhea or constipation are both adverse effects C - Shortened PR interval is NOT an adverse effect D - ondansetron does not affect blood glucose E - A prolonged QT interval is a possible adverse effect of ondansetron that can lead to torsades de pointes, a serious dysrhythmia.

ATI A nurse is assessing a client who is postoperative following a gastrectomy. The nurse should identify which of the following findings as an indication of abdominal distension? A - Hiccups B - Hypertension C - Bradycardia D - Chest pain

A - Hiccups Rational A- Following surgery, hiccups can be caused by irritation of the phrenic nerve due to abdominal distension. If the hiccups are intractable, the nurse should anticipate a prescription for chlorpromazine because persistent hiccups are distressful to the client and can lead to complications, such as vomiting. B - The nurse should identify hypotension as an indication of abdominal distension. C - The nurse should identify tachycardia as an indication of abdominal distension. D - The nurse should identify abdominal pain as an indication of abdominal distension.

E - 50 The nurse is caring for a client with peptic ulcer disease who has been vomiting profusely at home before coming to the emergency department. For which vital sign change will the nurse expect for this client? A - Hypotension B - Tachypnea C - Oxygen desaturation D - Bradycardia

A - Hypotension The client who is vomiting profusely is losing fluids from the body causing dehydration. A client who is dehydrated has hypovolemia resulting in hypotension and tachycardia.

ATI 46 A nurse is teaching a client who will undergo a sigmoidoscopy. Which of the following information about the procedure should the nurse include? Select all A - Increase flatulence can occur following the procedure B - NPO status should be maintained preprocedure C - Conscious sedation is used D - Repositioning will occur throughout the procedure E - Fluid intake is limited the day after the procedure.

A - Increase flatulence can occur following the procedure B - NPO status should be maintained preprocedure Rational C - sedations is not indicated D - lie on left side E - no limit following

E - 53 The nurse is caring for a client who has been diagnosed with cirrhosis. Which laboratory result(s) would the nurse expect for this client? Select all that apply. A - Increased serum bilirubin B - Increased lactate dehydrogenase C - Decreased serum albumin D - Increased serum alanine E - aminotransferase F - Increased aspartate aminotransferase G - Increased serum ammonia

A - Increased serum bilirubin B - Increased lactate dehydrogenase C - Decreased serum albumin D - Increased serum alanine E - aminotransferase F - Increased aspartate aminotransferase G - Increased serum ammonia Cirrhosis is a chronic disease in which the liver progressively degenerates. As a result, liver enzymes and bilirubin increase. Additionally, the liver is unable to synthesize protein leading to decreased serum albumin. Elevated serum ammonia results from the inability of the liver to detoxify protein by-products.

ATI 30 A nurse is preparing to administer potassium chloride IV to a client who has hypokalemia. Which of the following actions should the nurse take? Select all A - Infuse medication through a large-bore needle B - Monitor urine output to ensure at least 20 mL/hr C - Administer medication via direct IV bolus D - Implement cardiac monitoring E - Administer the infusion using an IV pump

A - Infuse medication through a large-bore needle D - Implement cardiac monitoring E - Administer the infusion using an IV pump Rational A - infuse potassium through a large-born needle to prevent vein irritation, phlebitis, and infiltration B - Monitor urine output to ensure at least 30 mL/h for adequate kidney function C - Admin IV potassium slowly, no faster than 10 mEq/hr. Rapdu admin can result in fatal hyperkalemia D - implement cardiac monitor to detect cardiac dysrhythmias in the client receiving IV potassium E - Admin IV potassium using an infusion pump to prevent fatal hyperkalemia due to rapid infusion rate.

AIT - P A nurse is planning care for a client who is receiving chemotherapy and has a new prescription for ondansetron. Which of the following actions should the nurse plan to take? (Select all that apply.) A - Infuse the drug 30 min prior to chemotherapy. B - Administer the drug when the client reports nausea. C - Infuse the drug slowly over 15 min. D - Administer the drug immediately following chemotherapy. E - Repeat the dose 4 hr after chemotherapy.

A - Infuse the drug 30 min prior to chemotherapy. C - Infuse the drug slowly over 15 min. E - Repeat the dose 4 hr after chemotherapy. Rational A - Infuse the drug 30 min prior to chemotherapy is correct. The nurse should administer IV ondansetron, a serotonin antagonist, 30 min prior to chemotherapy to prevent chemotherapy-induced nausea and vomiting. When treating radiation-induced nausea and vomiting, the nurse should give the drug orally 1 to 2 hr prior to radiation therapy and again every 8 hr. B - Administer the drug when the client reports nausea is incorrect. Ondansetron prevents nausea and vomiting from chemotherapy, radiation therapy, and anesthesia. The nurse should not wait until the client reports nausea to administer the drug. C - Infuse the drug slowly over 15 min is correct. The nurse should infuse ondansetron slowly over 15 min to prevent chemotherapy-induced nausea. D - Administer the drug immediately following chemotherapy is incorrect. Administering ondansetron immediately following chemotherapy might not allow enough time for the drug to exert its antiemetic actions before the client begins to feel chemotherapy-induced nausea. E - Repeat the dose 4 hr after chemotherapy is correct. For maximum effectiveness, the nurse should administer ondansetron 4 hr after chemotherapy, and again 8 hr after chemotherapy. When treating anesthesia-induced nausea and vomiting, the nurse can give the drug 1 hr before anesthesia induction.

ATI A nurse is assessing a client who has acute hepatitis B. Which of the following findings should the nurse expect? A - Joint pain B - Obstipation C - Abdominal distention D - Periumbilical discoloration

A - Joint pain Rational A - Joint pain is an expected finding in a client who has acute hepatitis B. B - Obstipation, or failure to pass stools, is an expected finding in a client who has a complete bowel obstruction. C - Abdominal distention is an expected finding in a client who has a small bowel obstruction. D - Periumbilical discoloration is an expected finding in a client who has intraperitoneal bleeding.

E - 55 A client with obesity has been taking orlistat as prescribed for 4 weeks but has only lost 10 lb (4.5 kg). The health care provider doubles the dosage and asks the nurse to provide further teaching. What behavioral change does the nurse include in the teaching plan? Select all that apply. A - Keep a daily food diary. B - Set daily reasonable goals for eating. C - Increase exercise. D - Identify emotional and situational factors that stimulate eating. E - Identify a healthy individual that can be role-modeled.

A - Keep a daily food diary. B - Set daily reasonable goals for eating. C - Increase exercise. D - Identify emotional and situational factors that stimulate eating. Techniques the nurse includes in the teaching plan are keeping a record of foods eaten (food diary) (to look at daily trends), identifying emotional and situational factors that stimulate eating (which can be modified after identification), and increasing exercise (to burn calories). Setting daily reasonable goals helps the client focus on how to be healthy now instead of setting unreasonable future goals that cause the client to give up if not quickly achieved.The client should not look to role-model other people, as what works for them may not work for the client. The client should focus on his or her own behaviors in order to identify trends and make reasonable changes.

ATI 51 A nurse is completing discharge teaching with a client who has IBS. Which of the following instructions should the nurse include? A - Keep a food diary to identify triggers to exacerbation B - Consume 15 to 20 g of fiber C - Plan three moderate to large meals per day D - Limit fluid intake to 1 L each day

A - Keep a food diary to identify triggers to exacerbation Rational A - helps id triggers B - Increase fiber intake to 30-40 g C - should eat small frequent meals D - Should drink 2-3 L Fluid per day to promote a consistent bowel pattern

E - 54 The nurse is caring for a client who recently had an external percutaneous transhepatic biliary catheter placed for severe biliary obstruction. What is the nurse's priority intervention when caring for this client? A - Keeping the biliary drainage bag below the level of the catheter-insertion site B - Checking the client's blood glucose frequently to monitor for diabetes C - Managing pain with continuous opioid patient-controlled analgesia (PCA) D - Capping the catheter if it starts to leak around the insertion site

A - Keeping the biliary drainage bag below the level of the catheter-insertion site The client who has an external percutaneous transhepatic biliary catheter drains by gravity and therefore needs to have the drainage bag placed lower that the catheter-insertion site. The catheter is not capped if jaundice or leakage around the catheter site occurs. Opioids are not needed while the client has the catheter; however, if it is in place for an extended period of time, it needs to be changed.

ATI - P A nurse is caring for an older adult client who has kidney disease and a new prescription for cimetidine. The nurse should instruct the client to report which of the following manifestations? A - Lethargy B - Cellulitis C - Dry mouth D - Myalgia

A - Lethargy Rational A - Cimetidine is a histamine2 receptor antagonist, which can potentially cross the blood-brain barrier. Older adult clients who have kidney impairments are at risk for central nervous system (CNS) effects such as lethargy, depression, confusion, and seizures. The nurse should instruct the client to report these manifestations to their provider. B - Cimetidine, a histamine2 receptor antagonist, is unlikely to cause cellulitis. It can, however, cause other integumentary manifestations, such as rash, alopecia, and Stevens-Johnson syndrome. C - Cimetidine, a histamine2 receptor antagonist, is unlikely to cause dry mouth. It can, however, cause other adverse GI effects, such as fungal overgrowth in the stomach and mild diarrhea. D - Cimetidine, a histamine2 receptor antagonist, is unlikely to cause myalgia. It can, however, exacerbate joint pain in clients who have arthritis.

ATI 55 A nurse is teaching a client who has hepatitis B about home care. Which of the following instruction should the nurse include in the teaching? Select all A - Limit physical activity B - Avoid alcohol C - Take acetaminophen for comfort D - Wear a mask when in public places E - Eat small frequent meals.

A - Limit physical activity B - Avoid alcohol E - Eat small frequent meals. Rational A - frequent rest breaks conserves energy and assists in the recovery process B - Alcohol is metabolized in the liver C - Acetaminophen is metabolized in the liver D - Hep B is blood borne E - A client with Hep B should eat small frequent meals to promote improved nutrition due to the presence of anorexia

E - 51 The nurse is reviewing medications that can be used for female clients who have constipation-predominant irritable bowel syndrome (IBS). Which drugs are available for this health problem? Select all that apply. A - Lubiprostone B - Cetuximab C - 5-fluorouracil D - Psyllium hydrophilic mucilloid E - Linaclotide

A - Lubiprostone D - Psyllium hydrophilic mucilloid E - Linaclotide Cetuximab and 5-fluorouracil are chemotherapeutic drugs used for clients who have colorectal cancer. The other drugs are available for female clients who have constipation-predominant IBS.

ATI A nurse is developing a plan of care for a client who has cirrhosis and ascites. Which of the following interventions should the nurse include in the plan? A - Measure the client's abdominal girth daily. B - Check mental status once daily. C - Provide a daily intake of 4 g of sodium for the client. D - Assess the client's breath sounds every 12 hr.

A - Measure the client's abdominal girth daily. Rational A - The nurse should measure the client's abdominal girth and weigh the client daily to monitor the amount of fluid accumulation in the abdomen and the effectiveness of treatment measures. B - A client who has cirrhosis is at risk for hepatic encephalopathy. The nurse should assess the client's mental status every 4 to 8 hr. C - A client who has cirrhosis can have edema and ascites and is usually prescribed a 1 to 2 g sodium-restricted diet to prevent ascites. D - A client who has cirrhosis is at risk for dyspnea due to ascites. The nurse should monitor the client's breath sounds every 4 to 8 hr.

E - 55 Which nursing care activity for an undernourished client does the nurse safely delegate to an assistive personnel (AP)? A - Measuring current height and weight B - Determining body mass index (BMI) C - Estimating body fat using skinfold measurements D - Completing the Mini Nutritional Assessment

A - Measuring current height and weight Determining height and weight is the only activity that the nurse can safely delegated to an AP.The nurse is responsible for completing the Mini Nutritional Assessment, determining the client's BMI, and estimating body fat using skinfold measurements, as these assessments fall within the scope of practice of a registered nurse.

The nurse is caring for a client who was recently diagnosed with Helicobacter. pylori infection. Which drugs does the nurse and anticipate would be used for this client to manage the infection? Select all that apply. A - Metronidazole B - Lansoprazole C - Azithromycin D - Tetracycline E - Hydroxychloroquine

A - Metronidazole B - Lansoprazole D - Tetracycline Most clients who have this type of infection are prescribed to take a proton pump inhibitor, such as lansoprazole, and two antimicrobial drugs, such as metronidazole and tetracycline. Clarithromycin and amoxicillin may be used as alternative antibiotics.

The nurse is teaching a client with a newly created colostomy about foods to limit or avoid because of flatulence or odors. Which foods should be avoided? Select all that apply. A - Mushrooms B - Peas C - Onions D - Broccoli E - Buttermilk F - Yogurt

A - Mushrooms B - Peas C - Onions D - Broccoli Foods the patient with a newly created colostomy needs to limit or avoid because of flatulence or odors include: broccoli, mushrooms, onions, and peas. Buttermilk will help prevent odors. Yogurt can help prevent flatus.

ATI - P A nurse is caring for a client who is taking lubiprostone. The nurse should tell the client that lubiprostone can cause which of the following adverse drug reactions? A - Nausea B - Constipation C - Urinary retention D - Sore throat

A - Nausea Rational A - Lubiprostone, a chloride channel activator, can cause nausea. Clients who develop this effect do not need to discontinue the drug but should take it with food and water to minimize GI upset. B - Lubiprostone, a chloride channel activator, increases peristalsis and activates the chloride channels in the intestinal wall, which then increases the secretion of sodium and water. These actions treat constipation and can cause diarrhea. C - Lubiprostone, a chloride channel activator, is unlikely to cause or worsen urinary retention. Dimenhydrinate, an antihistamine, is a GI-system drug that can cause urinary retention. D - Lubiprostone, a chloride channel activator, is unlikely to cause or worsen a sore throat or respiratory infection. Sulfasalazine is a GI-system drug that can cause agranulocytosis, which could cause or worsen a sore throat.

E - 54 The nurse is caring for a client who states that her mother had "gallbladder problems" and wonders if she is at risk for this disorder. What major risk factor places women most at risk for gallbladder disease? A - Obesity B - Birth control pills C - Infertility D - Advanced age

A - Obesity Obese women who are middle age and have had multiple children are at the highest risk for gallbladder disease, although it can occur in anyone.

ATI 53 A nurse is reviewing risk factors with a client who has cholecystitis. The nurse should identify that which of the following as a risk factor for cholecystitis? A - Obesity B - Rapid weight gain C - Decreased blood triglyceride level D - Male sex

A - Obesity Rational B - rapid weight loss is a risk C - increase in cholesterol is a risk D - female is risk

ATI 47 A nurse is planning care for a client who has a new prescription for TPN. Which of the following interventions should be included in the plan of care Select all A - Obtain a capillary blood glucose four times daily B - Administer prescribed medications through a secondary port on the TPN IV tubing C - Monitor vital signs three times during the 12-hour shift D - CHange the TPN IV tubing every 24 hr E - Ensure a daily a PTT is obtained.

A - Obtain a capillary blood glucose four times daily C - Monitor vital signs three times during the 12-hour shift D - CHange the TPN IV tubing every 24 hr Rational B - no other meds/fluids should be admin through the IV - increase risk of infection and disruption of rate of TPN E - not necessary - aPTT measures coagulability of blood

ATI 23 A nurse is teaching a parent of an infant about gastrointestinal reflux disease. Which of the following should the nurse include in the teaching? Select all A - Offer frequent feedings B - Thicken formula with rice cereal C - Use a bottle with a one-way valve D - Positions baby upright after feedings E - Use a wide-base nipple for feedings

A - Offer frequent feedings B - Thicken formula with rice cereal D - Positions baby upright after feedings Rational A - Frequent feedings will assist in decreasing the amount of vomiting episodes B - Thickened formula will assist in decreasing the amount of vomiting episodes C - A bottle with a one-way valve is used for an infant who has cleft lip and palate D - Positioning the infant upright position following feedings will assist in decreasing the amount of vomiting episodes. E - A wide-based nipple iis used for an infant who has cleft li and palate

ATI - P A nurse is caring for a client who has a prescription for alosetron. The nurse should recognize that alosetron therapy is effective when the client reports which of the following? A - One formed stool per day B - Urination without burning C - Cessation of nausea or vomiting D - Reduced GI reflux effects

A - One formed stool per day Rational A - Alosetron, a serotonin 5-HT3 receptor antagonist, treats the diarrhea and pain of severe irritable bowel syndrome. One formed stool per day indicates effective therapy. B - Alosetron, a serotonin 5-HT3 receptor antagonist, is unlikely to improve burning or pain with urination. It can, however, cause urinary frequency. C - Alosetron, a serotonin 5-HT3 receptor antagonist, is unlikely to affect nausea and vomiting. Ondansetron is a GI-system drug that treats nausea and vomiting. D - Alosetron, a serotonin 5-HT3 receptor antagonist, is unlikely to affect gastrointestinal reflux. Metoclopramide is a GI-system drug that treats GERD.

ATI A nurse is assessing a client who has appendicitis. Which of the following findings should the nurse expect? (Select all that apply.) A - Oral temperature 38.4° C (101.1° F) B - WBC count 6,000/mm3 C - Bloody diarrhea D - Nausea and vomiting E - Right lower quadrant pain

A - Oral temperature 38.4° C (101.1° F) D - Nausea and vomiting E - Right lower quadrant pain Rational A - Oral temperature 38.4° C (101.1° F) is correct. A low-grade temperature is an expected finding in a client who has appendicitis. B - WBC count 6,000/mm3 is incorrect. A WBC count of 10,000 to 18,000/mm3 is an expected finding in a client who has appendicitis. C - Bloody diarrhea is incorrect. Bloody diarrhea is an expected finding in a client who has colorectal cancer. D - Nausea and vomiting is correct. Nausea and vomiting are expected findings in a client who has appendicitis. E - Right lower quadrant pain is correct. Right lower quadrant pain is an expected finding in a client who has appendicitis.

ATI 22 A nurse is caring for a child who is suspected to have Enterobius vermicularis. Which of the following actions should the nurse take? A - Perform a tape test B - Collect stool specimen for culture C - Test the stool for occult blood D - Initiate IV fluids

A - Perform a tape test Rational A - Used to diagnosis Enterobius vermicularis B - Stool culture are obtained to diagnoses Salmonella or C. Diff infections C - A manifestation of E. Coli is blood stools D - IV fluids are initiated for children who are dehydrated

E - 51 A 67-year-old male client with no surgical history reports pain in the inguinal area that occurs when he coughs. A bulge that can be pushed back into the abdomen is found in his inguinal area. What type of hernia does he have? A - Reducible B - Strangulated C - Incarcerated D - Femoral

A - Reducible The hernia is reducible because its contents can be pushed back into the abdominal cavity.Femoral hernias tend to occur more frequently in obese and pregnant women. A hernia is considered to be strangulated when the blood supply to the herniated segment of the bowel is cut off. An incarcerated or irreducible hernia cannot be reduced or placed back into the abdominal cavity. Any hernia that is not reducible requires immediate surgical evaluation.

A nurse is caring for a client who is receiving TPN solution. The current bag of solution was hung 24 hrs ago, and 400 mL remain to infuse. Which of the following is the appropriate action for the nurse to take? A - Remove the current bag and hang a new bag B - Infuse the remaining solution at the current rate and then hand a new bag C - Increase the infusion rate so the remaining solution is administered within the hour and hang a new bag D - Remove the current bag and hang a bag of lactated Ringer's

A - Remove the current bag and hang a new bag Rational A - TPN should not hand more than 24 hr due to risk of infection B - TPN should not infuse more than 24 hr due to infection C - Rate of TPN should never be increased abruptly due to risk of hyperglycemia D - use dextrose 10% in water

E - 54 A client is admitted to the emergency department with possible acute pancreatitis. What is the nurse's priority assessment at this time? A - Respiratory assessment B - Cardiovascular assessment C - Abdominal assessment D - Pain intensity assessment

A - Respiratory assessment As for any client, the nurse would want to continually assess for airway, breathing, and circulation. Clients who have acute pancreatitis often develop pleural infusions, atelectasis, or pneumonia. Necrotizing hemorrhagic pancreatitis places the client at risk for acute respiratory distress syndrome (ARDS).

E - 53 When assessing a client with hepatitis B, the nurse anticipates which assessment findings? Select all that apply. A - Right upper quadrant tenderness B - Itching C -Recent influenza infection D - Brown stool E - Tea-colored urine

A - Right upper quadrant tenderness B - Itching E - Tea-colored urine Assessment findings the nurse expects to find in a client with hepatitis B include brown, tea-, or cola-colored urine; right upper quadrant pain due to inflammation of the liver; and itching, irritating skin caused by deposits of bilirubin on the skin secondary to high bilirubin levels and jaundice.Hepatitis B virus, not the influenza virus, causes hepatitis B, which is spread by blood and body fluids. The stool in hepatitis may be tan or clay-colored, not typically brown

E - 52 A client returns to the unit after having an exploratory abdominal laparotomy. How does the nurse position this client after being situated in bed? A - Semi-Fowler B - Lateral Sims' (side-lying) C - High Fowler D - Supine

A - Semi-Fowler The nurse places the postoperative abdominal laparotomy client in the semi-Fowler position in bed. The client is maintained in this position to facilitate the drainage of peritoneal contents into the lower region of the abdominal cavity after an abdominal laparotomy. This position also helps increase lung expansion.High-Fowler position would be too high for the client postoperatively. It would place strain on the abdominal incision(s), and, if the client was still drowsy from anesthesia, this position would not enhance the client's ability to rest. Sims' position does not promote drainage to the lower abdomen. The supine position does not facilitate drainage to the abdomen or increased lung expansion. The client would be more likely to develop complications (wound drainage stasis and atelectasis) in the supine position.

ATI 22 A nurse is teaching a group of caregivers about E. coli. Which of the following information should the nurse include in the teaching? Select all A - Severe abdominal cramping occurs B - Watery diarrhea is present for more than 5 days C- It can lead to hemolytic uremic syndrome D - It is a foodborne pathogen E - Antibiotics are given for treatment

A - Severe abdominal cramping occurs C- It can lead to hemolytic uremic syndrome D - It is a foodborne pathogen Rational A - Severe abdominal cramping is. manifestation of E. Coli B - Watery diarrhea lasts 1 - 2 days, then advances to bloody diarrhea C - E. Coli can lead to hemolytic uremic syndrome D - E. coli is a foodborne pathogen E - Antibiotics can worsen an E. coli infection

E - 52 The nurse is caring for a client who has an enterocutaneous fistula. For what complications will the nurse monitor? Select all that apply. A - Skin breakdown B - Hyperkalemia C - Malnutrition D - Hypernatremia E - Dehydration F - Bowel obstruction

A - Skin breakdown C - Malnutrition E - Dehydration The client has an abnormal tunneling between the small intestines and the skin causing spillage of the GI contents onto the skin. Enzymes in the intestines can break down skin and underlying tissues. The intestinal contents are also rich in fluids and electrolytes, especially potassium, such that the client would likely develop hypokalemia rather than hyperkalemia. Loss of fluids could lead to dehydration if the client is not carefully monitored and managed.

E - 53 The nurse teaches the client who has cirrhosis about foods and other substances that should be avoided to prevent worsening of the disease. Which substance(s) will the nurse include in that health teaching? Select all that apply. A - Smoking B - Alcohol C - Illicit drugs D - Acetaminophen E - Sodium F - Protein

A - Smoking B - Alcohol C - Illicit drugs D - Acetaminophen Protein and sodium should be moderately restricted but not completely avoided. The other substances can worsen the disease process, especially drugs and alcohol which are normally metabolized by the liver.

E - 48 Which factor does the nurse identify that places a client at risk for gastrointestinal (GI) problems? Select all that apply. A - Smoking a half-pack of cigarettes per day B - Taking nonsteroidal anti-inflammatory drugs (NSAIDs) C - Financial concerns D - Eating a high-fiber diet E - Use of herbal preparations

A - Smoking a half-pack of cigarettes per day B - Taking nonsteroidal anti-inflammatory drugs (NSAIDs) C - Financial concerns E - Use of herbal preparations Smoking or any tobacco use places a client in a higher-risk category for GI problems. Financial concerns can also influence the risk for GI problems; clients may not be able to afford to seek care or treatment and may put off seeking help. Some herbal preparations contribute to GI problems, such as Ayurvedic herbs, which can affect appetite, absorption, and elimination. NSAIDs can predispose clients to peptic ulcer disease or GI bleeding.High-fiber diets are generally believed to be healthy for most clients.

ATI 29 A nurse is providing instruction about the use of laxatives to a client who has heart failure. The nurse should tell the client to avoid which of the following laxatives? A - Sodium phosphate B - Psyllium C - Bisacodyl D - Polyethylene glycol

A - Sodium phosphate Rational A- Absorption of sodium form sodium phosphate causes fluid retention which can exacerbate heart failure. B - Psyllium in not absorbed by the intestine C - Bisacodyl does not appear to have systemic effects D - polyethylene glycol is contraindicated in a number of GI conditions, but it is not contraindicated for clients who have heart failure.

ATI 49 A nurse is teaching a client who has a duodenal ulcer and a new prescription for esomeprazole. Which of the following information should the nurse include in the teaching? Select all A - Take the medication 1 hr before a meal B - Limit NSAIDs when taking this medication C - Expect skin flushing when taking this medication D - Increased fiber intake when taking this medication E - Chew the medication thoroughly before swallowing

A - Take the medication 1 hr before a meal B - Limit NSAIDs when taking this medication Rational C - not expected D - fiber does not need to be increased E - swallow whole

ATI - P A nurse is caring for a male client who asks the nurse about taking alosetron for irritable bowel syndrome with diarrhea (IBS-D) lasting 3 months. Which of the following information should the nurse provide the client about alosetron? A - The drug is prescribed to female clients who have IBS-D lasting more than 6 months. B - The drug is prescribed to clients who have chronic diarrhea lasting more than 12 months. C - The drug is most beneficial for male clients who have inflammatory bowel disease. D - The drug is most beneficial in treating clients who have constipation-predominant IBS (IBS-C).

A - The drug is prescribed to female clients who have IBS-D lasting more than 6 months. Rational A - Alosetron is approved only to be prescribed to females who have IBS-D lasting 6 months or longer that has not been controlled by conventional treatment. B - Alosetron can be prescribed to clients who have certain types of chronic diarrhea caused by IBS, but it is not prescribed to treat all causes of chronic diarrhea. Furthermore, it can be prescribed sooner than 12 months. C - Alosetron is not prescribed to treat inflammatory bowel disease. 5-aminosalicylate medications are one class of drugs used to treat this disorder. D - Alosetron is approved only to be prescribed to women who have IBS-D, not IBS-C.

ATI A nurse is caring for a client who has ulcerative colitis. The client has had several exacerbations over the past 3 years. Which of the following instructions should the nurse include in the plan of care to minimize the risk of further exacerbations? (Select all that apply.) A - Use progressive relaxation techniques. B - Increase dietary fiber intake. C - Drink two 240 mL (8 oz) glasses of milk per day. D - Arrange activities to allow for daily rest periods. E - Restrict intake of carbonated beverages.

A - Use progressive relaxation techniques. D - Arrange activities to allow for daily rest periods. E - Restrict intake of carbonated beverages. Rational A - Use progressive relaxation techniques is correct. Progressive relaxation techniques, a form of biofeedback, are recommended to help the client minimize stress, which can precipitate an exacerbation. B - Increase dietary fiber intake is incorrect. The client should restrict intake of dietary fiber, which can cause diarrhea and cramping. C - Drink two 240 mL (8 oz) glasses of milk per day is incorrect. Dairy products, such as milk, are poorly tolerated by clients who have ulcerative colitis and should be avoided. D - Arrange activities to allow for daily rest periods is correct. Daily rest periods decrease stress and reduce intestinal motility. E - Restrict intake of carbonated beverages is correct. The client should avoid gastrointestinal stimulants, such as carbonated beverages, nuts, peppers, and smoking.

E - 55 A client who has undergone a bariatric surgical procedure is recuperating after surgery. Which nursing intervention most effectively prevents a client injury during repositioning? A - Using a weight-rated extra-wide bed for the client B - Administering pain medication C - Monitoring skinfold areas and keeping them clean and dry D - Making sure not to move the client's nasogastric (NG) tube

A - Using a weight-rated extra-wide bed for the client The most effective way to reposition a postoperative bariatric client and prevent injury is to use a special weight-related extra wide bed. This will allow adequate room for repositioning the client comfortably without causing the bed rails to touch his or her body, causing pressure and injury.Pain medication and monitoring skinfold areas will not prevent injury to the client that might occur during repositioning. Not moving the client's NG tube will prevent disruption of the suture line, but will not prevent repositioning injuries.

E- 54 The nurse is preparing to instruct a client with chronic pancreatitis who is to begin taking pancrelipase. Which instruction does the nurse include when teaching the client about this medication? A - Wipe your lips after taking pancrelipase. B - Take pancrelipase before meals. C - Administer pancrelipase before taking an antacid. D - Chew tablets before swallowing.

A - Wipe your lips after taking pancrelipase. The nurse will instruct the client to wipe the lips after taking pancrelipase. Pancrelipase is a pancreatic enzyme used for enzyme replacement for clients with chronic pancreatitis. To avoid skin irritation and breakdown from residual enzymes, the lips should be wiped.Pancrelipase should be administered after, and not before, antacids or histamine2 blockers are taken. It should not be chewed to minimize oral irritation and allow the drug to be released more slowly. It should be taken with meals and snacks, and not before, and followed with a glass of water.

E - 51 The nurse is teaching a group of clients with irritable bowel syndrome (IBS) about complementary and alternative therapies. What does the nurse suggest as possible treatment modalities? Select all that apply. A - Yoga B - Acupuncture C - Peppermint oil capsules D - Decreasing physical activities E - Meditation

A - Yoga B - Acupuncture C - Peppermint oil capsules E - Meditation Possible treatment modalities the nurse suggests for a client with IBS include: acupuncture, meditation, peppermint oil capsules, and yoga. Acupuncture is recommended as a complementary therapy for IBS. Meditation, yoga, and other relaxation techniques help many patients manage stress and their IBS symptoms. Research has shown that peppermint oil capsules may be effective in reducing symptoms of IBS. Regular exercise is important for managing stress and promoting bowel elimination.

ATI - P A nurse should recognize that diphenoxylate/atropine should be used with caution for a client who has which of the following conditions? A -Inflammatory bowel disease B - Thrombophlebitis C - Agranulocytosis D - Immunization with a live virus vaccine within the last 6 weeks

A -Inflammatory bowel disease Rational A - Diphenoxylate is an opioid, which can cause the severe complication of toxic megacolon in clients who have inflammatory bowel disease. The nurse should have the client monitor the consistency and frequency of stools throughout therapy. B - Clients who have thrombophlebitis can take diphenoxylate/atropine, an opioid antidiarrheal combination. Thrombophlebitis is a contraindication for the use of alosetron, another GI-system drug. C - Clients who have agranulocytosis can take diphenoxylate/atropine, an opioid antidiarrheal combination. Agranulocytosis is a potential adverse drug reaction of sulfasalazine, another GI-system drug, and the drug should be used with caution for clients who have existing blood dyscrasias. D - Clients who have had an immunization with a live virus vaccine within the last 6 weeks can take diphenoxylate/atropine, an opioid antidiarrheal combination. For clients who have recently received a live virus vaccine, azathioprine can reduce the antibody response and increase the risk of adverse reactions.

ATI 23 A nurse is caring for a child who has Meckel's diverticulum. Which of the following manifestations should the nurse expect? Select all A- Abdominal pain B - Fever C - Mucus and blood in stool D - Vomiting E - Rapid, shallow breathing

A- Abdominal pain C - Mucus and blood in stool Rational A - abdominal pain is a manifestation of Meckel's diverticulum B - Fever is a manifestation of appendicitis C - Mucus and bloody stool are a manifestation of Meckel's diverticulum D - Vomiting is a manifestation of appendicitis E - Rapid, shallow breathing is a manifestation of appendicitis

ATI 23 A nurse is assessing an infant who has hypertrophic pyloric stenosis. Which of the following manifestations should the nurse expect? Select all A- Projectile vomiting B - Dry mucous membranes C - Currant jelly stools D - Sausage-shaped abdominal mass E - Constant hunger

A- Projectile vomiting B - Dry mucous membranes E - Constant hunger Rational A - pyloric stricture has thickening of the pyloric sphincter, resulting in projectile vomiting B - Client with pyloric stricture is unable to consume adequate food and fluids, resulting in dehydration. Dry mucous membranes is a manifestation of hypertrophic pyloric stenosis C - a client who has intussusception has blood mucus stools, resulting in currant jelly stools D - A client who has intussusception has telescoping intestine, resulting in sausage shaped abdominal mass E - A client who has pyloric stricture is unable to consume adequate flood and fluid result in constant hunger

ATI 49 A nurse in the ED is completing an assessment of a client who has suspected stomach perforation due to a peptic ulcer. Which of the following findings should the nurse expect? Select all A- Rigid abdomen B - Tachycardia C - Elevated Blood pressure D - Circumoral cyanosis E - Rebound tenderness

A- Rigid abdomen B - Tachycardia E - Rebound tenderness Rational C - hypotension D - not a manifestation of perforation

Mastery - 48-2 Which teaching will the nurse include when educating a client who is scheduled to have an Esophagogastroduodenoscopy (EGD)? Select all that apply. A. "Anesthesia will be used for sedation." B. "The procedure takes about 20-30 minutes to complete." C. "Informed consent will be needed prior to the procedure." D. "A separate test will be required to obtain any needed biopsies." E. "You will need to refrain from eating for at least 6-8 hours before the EGD."

A. "Anesthesia will be used for sedation." B. "The procedure takes about 20-30 minutes to complete." D. "A separate test will be required to obtain any needed biopsies." E. "You will need to refrain from eating for at least 6-8 hours before the EGD." Rational The nurse will teach the patient undergoing an EGD that informed consent is required; anesthesia will be used for purposes of sedation during the procedure; the procedure lasts 20-30 minutes; and to refrain from eating 6-8 hours before the procedure. Specimens for biopsy and cell studies can be obtained through the endoscope, so a separate procedure is not needed.

Mastery - 53-1 A client is receiving adefovir for management of hepatitis B. What health teaching will the nurse provide for the client about this drug? Select all that apply. A. "Avoid places with crowds and individuals who have infection." B. "Report increased bruising to your doctor because the drug can cause bleeding." C. "Get your lab work done regularly because the drug can affect your kidneys." D. "Be careful and avoid falls because the drug can cause fractures." E. "Follow up with the dietitian to ensure that you adhere to your special diet."

A. "Avoid places with crowds and individuals who have infection." C. "Get your lab work done regularly because the drug can affect your kidneys." Rationales: Adefovir is an immune modulating drug and therefore can weaken a client's immune system. Therefore, teaching the client to avoid sources of infection is a correct response (Choice A). Almost all immune modulating drugs are excreted via the kidneys and kidney function may be impaired. Regular follow-up with lab work for monitor renal function is essential (Choice C). The other choices are not associated with this medication.

Master - 49-2 A client who had the Stretta procedure to treat severe GERD is being discharged. Which client statement requires further nursing teaching? Select all that apply. A. "Dysphagia after this procedure is normal". B. "It's important to stop my proton pump inhibitor." C. "I will not take NSAIDs and aspirin for at least 10 days." D. "I might cough up some blood following this procedure." E. "Today I will drink clear liquids and tomorrow I can eat soft food."

A. "Dysphagia after this procedure is normal". B. "It's important to stop my proton pump D. "I might cough up some blood following this procedure." Rational The nurse will provide further teaching by educating the patient to report dysphagia and bleeding after the procedure, as these are not normal post-procedure findings. The nurse will also clarify that PPI therapy should continue post-procedure. Continued use can be reevaluated after the efficacy of the procedure is determined in follow-up visits. The client has demonstrated understanding by saying that NSAIDs and aspirin should be avoided for at least 10 days, as this increases the risk of bleeding. Additionally, these drugs can be irritating to the gastric mucosa, so avoidance even longer might be recommended by the primary health care provider. The client also has demonstrated understanding by remaining on a clear liquid diet for at least 24 hours post-procedure before advancing to a soft diet the following day.

55-3 A client with obesity tells the nurse, "My genes are the only thing that have made me obese." What is the appropriate nursing response? Select all that apply. A. "Genes can contribute to obesity." B. "Tell me about your family history." C. "Let's talk about your nutrition intake." D. "Have you considered bariatric surgery?" E. "How do you feel about physical activity?" F. "What lifestyle modifications have you tried?"

A. "Genes can contribute to obesity." B. "Tell me about your family history." C. "Let's talk about your nutrition intake." E. "How do you feel about physical activity?" F. "What lifestyle modifications have you tried?" Rationale: The nurse can affirm that there is evidence to show that genes can contribute to obesity and asking about the family history. However, the nurse will also introduce lifestyle and environmental factors that exist, by asking about nutrition intake, physical activity, and lifestyle modifications that have been tried. At this time, it is not appropriate to ask about consideration of bariatric surgery; other historical information needs to be gathered before a plan of care is considered.

50-2 Which client statement regarding diet and nutrition after a total gastrectomy requires further teaching by the nurse? A. "I should stay sitting up for an hour after I eat." B. "I will avoid liquids with my meals." C. "I need to eat small frequent meals." D. "I need to stay away from concentrated sweets."

A. "I should stay sitting up for an hour after I eat." Rational Clients who have a total gastrectomy are at risk for dumping syndrome which occurs when food moves quickly into the small intestine. To slow the movement of food, the client should lie down rather than sit up after eating because gravity is less likely in that position to move the food though the GI tract. Therefore, the client needs more teaching to correct this statement. The other Choices B, C, and D help move the food more slowly and a smaller amount of food will be better tolerated than a larger one.

Mastery 50-2 What health teaching will the nurse include to promote gastric health for an adult client? Select all that apply. A. "Stop smoking or using tobacco of any form." B. "Do not drink excessive amounts of alcohol." C. "Consume high-fat protein foods and decrease carbohydrates." D. "Avoid excessive amounts of pickled or smoked food." E. "Avoid taking large amounts of NSAIDs."

A. "Stop smoking or using tobacco of any form." B. "Do not drink excessive amounts of alcohol." D. "Avoid excessive amounts of pickled or smoked food." E. "Avoid taking large amounts of NSAIDs." Rational Smoking/tobacco, alcohol, pickled or smoked foods, and NSAIDs can all irritate the lining of the stomach and should be avoided or used in small amounts. Therefore, Choices A, B, D, and E are correct. Choice C is an incorrect response because adults should eat a well-balanced diet that consists of all groups of foods following the MyPlate recommendations

49-2 A nurse is caring for four clients. Which individual does the nurse identify at the highest risk for development of oral cancer? A. 28-year-old with human papilloma virus (HPV) infection B. 30-year-old with recurrent aphthous stomatitis (RAS) C. 55-year-old who quit chewing tobacco 5 years ago. D. 76-year-old who is sometimes negligent in denture care

A. 28-year-old with human papilloma virus (HPV) infection Rational HPV is directly correlated to development of oral cancer, so the individual with this infection is at highest risk. Recurrent aphthous stomatitis and intermittent negligence in denture care do not pose a higher risk for oral cancer than HPV. The individual who stopped chewing tobacco 5 years ago have already decreased their risk.

53-2 The nurse is caring for a client in end-stage liver failure. Which interventions should implemented when observing for hepatic encephalopathy? Select all that apply. A. Assess the client's neurologic status as prescribed. B. Monitor the client's hemoglobin and hematocrit levels. C. Monitor the client's serum ammonia level. D. Monitor the client's electrolyte values daily. E. Prepare to insert an esophageal balloon tamponade tube. F. Make sure the client's fingernails are short.

A. Assess the client's neurologic status as prescribed. C. Monitor the client's serum ammonia level. Rationales: The client experiencing encephalopathy has chronic confusion and other mental status changes caused by increased ammonia levels. Therefore, monitoring the serum ammonia levels and the client's neurologic status during treatment can help determine the effectiveness of the treatment plan (Choices A and C). Monitoring electrolytes is appropriate for patients who have ascites (Choice D). Preparing to insert an esophageal tamponade tube and monitoring the client's hemoglobin and hematocrit are interventions for clients experiencing bleeding varices (Choices B and E). Choice F is appropriate to prevent tissue damage when clients have pruritus as a result of jaundice.

Master 55-1 The nurse is caring for four clients who have been recommended to consider bariatric surgery. Which assessment data requires immediate nursing intervention? A. BMI of 23 with gastrointestinal reflux B. BMI of 36 with hypertension C. BMI of 40 with type II diabetes D. BMI of 43 with sleep apnea

A. BMI of 23 with gastrointestinal reflux Rationale: Clients who may benefit from weight loss surgery include those who have not responded to traditional interventions, those with a BMI of 40 or greater, or those with a BMI of 35 or greater with other health risk factors. The client who has a BMI of 23, regardless of a coexisting health problem, has a normal BMI. Therefore the nurse will immediately intervene to determine that this individual does not need bariatric surgery.

Mastery 55-2 What discharge teaching will the nurse provide to a patient who had gastric bypass surgery? Select all that apply. A. Be certain to stay hydrated by drinking water. B. Solid food can be introduced back into the diet in a week. C. Report any back, shoulder, or abdominal pain to the surgeon. D. You are likely to have little urine output for the first few weeks. E.Each of your meals should initially contain about 5 tablespoons of food

A. Be certain to stay hydrated by drinking water. C. Report any back, shoulder, or abdominal pain to the surgeon. E.Each of your meals should initially contain about 5 tablespoons of food Rationale: The nurse will teach that hydration by drinking water is important; that back shoulder, or abdominal pain, or little urine output, should be reported to the surgeon immediately, as these are signs of anastomotic leakage; and that each of the initial meals at home should contain about 5 tablespoons of food. Solid food cannot be introduced back into the diet for about 8 weeks.

Mastery 54-2 The nurse is planning care for a client who had a laparoscopic Whipple surgery. For which complications will the nurse assess? Select all that apply. A. Bleeding B. Wound infection C. Intestinal obstruction D. Diabetes mellitus E. Abdominal abscess

A. Bleeding B. Wound infection C. Intestinal obstruction D. Diabetes mellitus E. Abdominal abscess Rationales: Clients having this complex surgery can develop any or all of the postoperative complications listed in the test item.

52-3 The nurse is teaching a client about nutrition and diverticulosis. Which food will the nurse teach the client to avoid? A. Cucumber B. Beans C. Carrot D. Radish

A. Cucumber Rationales: Clients who have diverticulosis should avoid any food with seeds, and cucumbers have seeds (Choice A). The other foods do not have seeds.

51-1 The community nurse is talking with a group of older clients about colorectal cancer (CRC) risk factors. Which of the following factors are considered to be CRC risk factors? Select all that apply. A. High-fat diet B. Crohn's disease C. Smoking D. Alcoholism E. Family history of cancer F. Obesity

A. High-fat diet B. Crohn's disease C. Smoking D. Alcoholism E. Family history of cancer F. Obesity Rationales: All of the choices have been found to be risk factors for the development of colorectal cancer.

48-3 Which teaching will the nurse provide to a community group about early detection of colorectal cancer? Select all that apply. A. Home testing kits are available with a prescription. B. A sigmoidoscopy should be performed every 10 years. C. Individuals over 40 years old should be testing for colon cancer. D. Bowel preparation is necessary prior to performance of a colonscopy E. Virtual colonoscopies (CT colonography) can be performed every 5 years

A. Home testing kits are available with a prescription. D. Bowel preparation is necessary prior to performance of a colonscopy E. Virtual colonoscopies (CT colonography) can be performed every 5 years Rational The nurse will teach that (1) home testing kits are available with a prescription, (2) bowel preparation is necessary prior to undergoing a traditional colonscopy to ensure precise visualization of the colon; and (3) virtual colonoscopies (CT colonography) can be performed every 5 years, per the American Cancer Society (2019). Sigmoidoscopies should be performed every 5 years, and individuals over the age of 45 should be tested for colon cancer.

52-2 The nurse is caring for an older adult client who experiences an exacerbation of ulcerative colitis with severe diarrhea and rectal bleeding that have lasted a week. For which complication(s) will the nurse assess? Select all that apply. A. Increased BUN B. Hypokalemia C. Leukocytosis D. Anemia E. Hyponatremia

A. Increased BUN B. Hypokalemia C. Leukocytosis D. Anemia E. Hyponatremia Rationales: The older client who has severe diarrhea and rectal bleeding is expected to be dehydrated, as evidenced by an increased blood urea nitrogen (BUN) (Choice A) and hyponatremia (Choice E). The client would have a decreased serum potassium as a result of loss of intestinal contents (Choice B), have an increase in WBCs as a result of inflammation Choice C), and have a low hemoglobin and hematocrit (anemia) from bleeding (Choice D). Therefore, all choices are correct responses.

Master 49-1 A nurse is caring for a 34-year-old client newly diagnosed with GERD. Which lifestyle change will the nurse suggest? Select all that apply A. Lose weight if needed B. Do not eat before bed C. Elevate the foot of your bed by 6-12 inches D. Avoid pants with a tight waistband or belt E. Eat fatty foods to minimize ongoing hunger

A. Lose weight if needed B. Do not eat before bed D. Avoid pants with a tight waistband or belt Rational Clients with GERD benefit from weight loss (if needed), avoidance of eating before bed (so that reflux is minimized), and by avoiding constrictive clothing which increase abdominal pressure. The head - not the foot - of the bed should be elevated to optimize gravity. Fatty foods should be avoided by clients with GERD, as these can increase reflux.

49-3 When providing discharge teaching about mouth care, which substance will the nurse teach the client with oral cancer to avoid? Select all that apply. A. Mouthwash B. Lip lubricant C. Warm saline rinses D. Ultra-soft toothbrush E. Disposable foam brushes F. Bicarbonate mouth rinse

A. Mouthwash E. Disposable foam brushes Rational The nurse will teach the client with oral cancer to avoid mouthwash (alcohol can irritate mucous membranes), and disposable foam brushes (which do not control plaque, and dry the mouth). Lip lubricant, warm saline or bicarbonate rinses, and use of an ultra-soft toothbrush can be encourage.

Mastery 51-1 The nurse is caring for a client with a complete large bowel obstruction. What assessment findings will the nurse expect? Select all that apply. A. Obstipation B. Dehydration C. Metabolic alkalosis D. Abdominal distention E. Abdominal pain F. Profuse vomiting

A. Obstipation D. Abdominal distention E. Abdominal pain Rationales: The client who has a large bowel obstruction does not usually have dehydration (Choice B), profuse vomiting (Choice F), or metabolic alkalosis (Choice C). These assessment findings are more common in clients who have a small bowel obstruction. However, obstipation (no stool)(Choice A) and pain/distention are very common in the lower abdomen (Choices A, D, E).

53-1 The nurse is caring for a client who is diagnosed with cirrhosis? Which serum laboratory values will the nurse expect to be abnormal? Select all that apply. A. Prothrombin time B. Serum bilirubin C. Albumin D. Aspartate aminotransferase (AST) E. Lactate dehydrogenase (LDH) F. Acid phosphatase

A. Prothrombin time B. Serum bilirubin C. Albumin D. Aspartate aminotransferase (AST) E. Lactate dehydrogenase (LDH) Rationales: Acid phosphatase is not usually affected by liver disease (Choice F). However, serum liver enzymes including AST and LDH are elevated (Choices D and E). Serum bilirubin increase due to the inability of the liver to excrete bile (Choice B). Prothrombin time is prolonged because the diseased liver decreases prothrombin production (Choice A). Serum albumin decreases due to the inability of the diseased liver to produce it (Choice C).

51-2 A client had an open partial colectomy and colostomy placement 6 hours ago. Which assessment would concern the nurse? A. Purple, moist stoma B. Stoma edema C. Liquid stool collecting in the drainage bag D. Serosanguineous fluid draining from the drain(s)

A. Purple, moist stoma Rationales: The stoma in the immediate postoperative period is expected to be swollen (Choice B), but reddish-pink and moist, not purple, which may indicate lack of blood flow to the stoma (Choice A). The stool is expected to be liquid at first (Choice C) and serosanguinous drainage is expected from the wound drain(s) (Choice D).

54-2 A client is admitted to the hospital yesterday with a diagnosis of acute pancreatitis. What assessment findings will the nurse expect for this client? Select all that apply. A. Severe boring abdominal pain B. Jaundice C. Nausea and/or vomiting D. Decreased serum amylase level E. Leukocytosis F. Dyspnea

A. Severe boring abdominal pain B. Jaundice C. Nausea and/or vomiting E. Leukocytosis F. Dyspnea Rationales: Clients who have acute pancreatitis usually have an elevated serum amylase level, not a decreased level. Therefore, Choice D is an incorrect response. All of the other choices are typical signs and symptoms of acute pancreatitis.

48-1 Which daily behavior of a client with GI problems requires further nursing assessment? Select all that apply. A. Smokes a pack of cigarettes B. Uses Fleet's enemas frequently to assist with bowel movements C. Practices intentional relaxation D. Eats multiple servings of fruits E. Takes 325 mg of aspirin at night for arthritic pain F. Exercises at 30 minutes three times weekly G. Travels extensively across the world

A. Smokes a pack of cigarettes* B. Uses Fleet's enemas frequently to assist with bowel movements* E. Takes 325 mg of aspirin at night for arthritic pain* G. Travels extensively across the world* Rational Smoking has been linked with an increased risk for most GI cancers; the nurse will need to obtain a full smoking history. If a client must use an enema frequently to assist with having bowel movements, further assessment is indicated. Aspirin and NSAID use can contribute to rectal bleeding, so this should be further assessed. Water and food variations from around the globe can impact GI health; therefore, the nurse will need to further assess the client's travel and nutrition history. Client behaviors of practicing intentional relaxation, eating multiple servings of fruit, and exercising are healthy behaviors that do not require further assessment.

E - 52 The nurse is teaching a client with Crohn disease about managing the disease with the adalimumab Which instruction does the nurse emphasize to the client? A - "Do not take the medication if you are allergic to foods with fatty acids." B - "Avoid large crowds and anyone who is sick." C - "Monitor your blood pressure and report any significant decrease in it." D - "Expect difficulty with wound healing while you are taking this drug."

B - "Avoid large crowds and anyone who is sick." The nurse emphasizes that the client taking adalimumab for Crohn disease needs to avoid being around large crowds to prevent developing an infection. Adalimumab (Humira), a biological response modifier (BRM), also known as a monoclonal antibody drug, has been approved for use in Crohn disease when other drugs have been ineffective. BRMs are approved for refractory (not responsive to other therapies) cases. These drugs cause immunosuppression and should be used with caution. Clients must be taught to report any signs of a beginning infection, including a cold, and to also avoid others who are sick.The client would not take the medication if he or she is allergic to certain proteins. Although immune suppression may occur to some degree, the client would not experience difficulty with wound healing while taking adalimumab. Also, the client would not experience a decrease in blood pressure from taking this drug.

E - 52 The nurse is teaching a client about dietary methods to help manage exacerbations (flare-ups) of diverticulitis. What does the nurse advise the client? A - "Maintain a high-fiber diet to prevent the development of hemorrhoids that frequently accompany this condition." B - "Consume a low-fiber diet while your diverticulitis is active. When inflammation resolves, consume a high-fiber diet." C - "Make sure you consume a high-fiber diet while diverticulitis is active. When inflammation resolves, consume a low-fiber diet." D - "Be sure to maintain an exclusively low-fiber diet to prevent pain on defecation."

B - "Consume a low-fiber diet while your diverticulitis is active. When inflammation resolves, consume a high-fiber diet." The nurse teaches the client that the most effective way to manage diverticulitis is to consume a low-fiber diet while inflammation is present, followed by a high-fiber diet once the inflammation has subsided.Neither an exclusively low-fiber diet nor an exclusively high-fiber diet will effectively manage diverticulitis. A high-fiber diet while diverticulitis is active will only worsen the disease and its symptoms.

E - 51 The nurse is teaching a client with irritable bowel syndrome (IBS) who has frequent constipation. Which statement by the client shows an accurate understanding of the nurse's teaching? A - "Maintaining a low-fiber diet will manage my constipation." B - "I need to go for a walk every day if possible." C - "Limiting the amount of fluid that I drink with meals is very important." D - "A cup of caffeinated coffee with cream & sugar at dinner is OK for me."

B - "I need to go for a walk every day if possible." The client statement, "I need to go for a walk every evening," shows that the client accurately understands the nurse's teaching plan to treat IBS. Walking every day is an excellent exercise for promoting intestinal motility. Increased ambulation is part of the management plan for IBS, along with increased fluids and fiber and avoiding caffeinated beverages.

E - 54 The nurse is teaching a preoperative client who is scheduled for a laparoscopic cholecystectomy ("lap chole"). What statement by the client indicates a need for further teaching? A - "I will likely need oral pain medications for the first few days after my surgery." B - "I should only be hospitalized for 2 to 3 days after my surgery." C - "I will probably not be at risk for complications from this surgery." D - "I should be able to go back to work in the next week or so."

B - "I should only be hospitalized for 2 to 3 days after my surgery." A "lap chole" surgery has many advantages over the open traditional surgical method, including a short hospital stay, usually same-day surgery, minimal risk for complications, and the ability to achieve pain control by using oral analgesics.

ATI A nurse is providing discharge teaching for a client who has chronic hepatitis C. Which of the following statements by the client indicates an understanding of the teaching? A - "I will avoid alcohol until I'm no longer contagious." B - "I will avoid medications that contain acetaminophen." C - "I will decrease my intake of calories." D - "I can donate blood once when I am in remission."

B - "I will avoid medications that contain acetaminophen." Rational A - A client who has hepatitis C should avoid alcohol consumption at all times due to the client's increased risk for cirrhosis. B - A client who has hepatitis C should avoid medications that contain acetaminophen, which can cause additional liver damage. C - A client who has hepatitis C should eat small, frequent meals that are high in carbohydrates and calories. D - Clients who have hepatitis C should never donate blood or organs, regardless of whether the virus is in the remission phase.

E - 52 A client has been newly diagnosed with ulcerative colitis (UC). What does the nurse teach the client about diet and lifestyle choices? A - "Raw vegetables and high-fiber foods may help to diminish your symptoms." B - "Lactose-containing foods should be reduced or eliminated from your diet." C - "Drinking carbonated beverages will help with your abdominal distress." D - "It's OK to smoke cigarettes, but you should limit them to ½ pack per day."

B - "Lactose-containing foods should be reduced or eliminated from your diet." The nurse teaches the newly diagnosed client with ulcerative colitis that lactose-containing foods are often poorly tolerated and need to be reduced or eliminated from the diet.Carbonated beverages are GI stimulants that can cause discomfort and must be used rarely or completely eliminated from the diet. Cigarette smoking is a stimulant that can cause GI distress symptoms. Nurses would never advise patients that any amount of cigarette smoking is "OK." Raw vegetables and high-fiber foods can cause GI symptoms in patients with UC.

E - 53 Which statement by a client with cirrhosis indicates that further instruction is needed about the disease? A - "The scars on my liver create problems with blood circulation." B - "My liver is scarred, but the cells can regenerate themselves and repair the damage." C - "Because of the scars on my liver, blood clotting and blood pressure are affected." D - "Cirrhosis is a chronic disease that has scarred my liver."

B - "My liver is scarred, but the cells can regenerate themselves and repair the damage." The client's statement that, although his liver is scarred, the cells can regenerate and repair the damage indicates that further instruction is needed. Although cells and tissues will attempt to regenerate, destroyed liver cells will result in permanent scarring and irreparable damage.

E - 48 A client is preparing to undergo a stool DNA (sDNA) test to screen for colon cancer. What preprocedure teaching does the nurse provide? A - "Do not eat or drink anything for 12 hours before the test." B - "No special preparation is needed prior to completing this test." C - "Give yourself tap water enemas until the fluid returns are clear." D - "Begin a clear liquid diet at least 24 hours before the test."

B - "No special preparation is needed prior to completing this test." The nurse will teach the client that no special preparation is needed prior to completing the Cologuard test. Cologuard is a home screening test that the client can perform at any time, with no traditional bowel cleaning preparation or fasting necessary.

ATI 48 A nurse is teaching a client who has hiatal hernia. Which of the following client statements indicates an understanding of the teaching A - "I can take my medication with soda." B - "Peppermint tea will increase my indigestion." C - "Wearing an abdominal binder will limit my manifestations." D - "I will drink hot chocolate at bedtime to help me sleep." E - "I can lift weight as a way to exercise."

B - "Peppermint tea will increase my indigestion." Rational A - Carbonated beverages decrease LES pressure and should be avoided by the client who has a hiatal hernia B - Peppermint decreases LES pressure and should be avoided b the client who has a hiatal hernia C - Tight restrictive clothing or abdominal binders should be avoided by the client who has hiatal hernia, as this increases intra-abdominal pressure and causes the protrusion of the stomach into the thoracic cavity. D - The client should avoid consuming anything immediately prior to bedtime. Additionally, chocolate relaxes the lower esophageal sphincter and should be avoided by the client who has hiatal hernia E - Heavy lifting and vigorous activities are to be avoided in the client who has hiatal hernia

ATI 28 A nurse is teaching a client who will begin taking aluminum hydroxide. Which of the following information should the nurse include in the teaching? A - "If constipation develops, switch to a calcium-based antacid" B - "Take this medication two hours before or after other medications." C - "This medication increases the risk for pneumonia." D - "Have your magnesium level monitored while taking this medication."

B - "Take this medication two hours before or after other medications." Rational A -alternate the aluminum antacid with a magnesium-bases one to achieve a normal bowel patterns if constipation occurs B - Cimetidine alters the absorption of many medications C - Omeprazole can increase risk for pneumonia D - Omeprazole can cause hypomagnesemia

ATI A nurse is caring for a client who has colorectal cancer and is receiving chemotherapy. The client asks the nurse why blood is being drawn for a carcinoembryonic antigen (CEA) level. Which of the following responses should the nurse make? A - "The CEA determines the current stage of your colon cancer." B - "The CEA determines the efficacy of your chemotherapy." C - "The CEA determines if the neutrophil count is below the expected reference range." D - "The CEA determines if you are experiencing occult bleeding from the gastrointestinal tract."

B - "The CEA determines the efficacy of your chemotherapy." Rational A - A provider uses a colonoscopy to determine the diagnosis of colorectal cancer. B - A provider uses the CEA level to determine the efficacy of the chemotherapy. The client's CEA levels will decrease if the chemotherapy is effective. C - A provider uses a CBC to determine if the neutrophil count is within the expected reference range. D - A provider uses a fecal occult blood test to determine if there is bleeding in the gastrointestinal tract.

ATI 50 A charge nurse is teaching a group of nurses about a client who has chronic gastritis and is scheduled for a selective vagotomy. Which of the following statements by a unit nurse indicated understanding of the purpose of the procedure? A - "The client will have increased duodenal gastric emptying." B - "The client will have a reduction of gastric acid secretions." C - "The client will have an increase of gastric mucus secretion." D - "The client will have an increased secretion of hydrogen/potassium ATPase enzyme."

B - "The client will have a reduction of gastric acid secretions." Rational A - Pyloroplasty will increase gastric emptying, which is performed to widen the opening from the stomach to the duodenum B - Selective vagotomy will reduce gastric acid secretions C - Prostaglandin analog medications will stimulate mucosal protection and decrease gastric acid secretion D - Histamine 2 antagonist med will inhibit gastric secretion by inhibiting the hydrogen/potassium ATPase enzyme system in the gastric parietal cells

ATI 53 A nurse is completing preoperative teaching for a client who is scheduled for a laparoscopic cholecystectomy. Which of the following should be included in the teaching? A - "The scope will be passed through your rectum." B - "You might have shoulder pain after surgery." C - "You will have a Jackson-Pratt drain in place after surgery." D - "You should limit how often you walk for 1 to 2 weeks

B - "You might have shoulder pain after surgery." Rational A - natural orifice surgery is NOTES B - is expect post op due to free air that is introduced into the abdomen during laparoscopic surgery C - Jackson-Pratt is placed during open surgery D - instructed to ambulate frequently to minimize the free air

ATI 30 A nurse is evaluating a group of client at a health fair to identify the need for folic acid therapy. Which of the following clients require folic acid therapy? Select all A - 12-year-old child who has iron deficiency anemia B - 24-year-old female who has no health problems C - 44-year-old male who has hypertension D - 55-year-old female who has alcohol use disorder E - 35-year-old male who has type 2 diabetes mellitus

B - 24-year-old female who has no health problems D - 55-year-old female who has alcohol use disorder Rational A - A client with iron deficiency anemia requires treatment with iron supplements B - A client of childbearing age should take folic acid to prevent neural tube defects in the fetus C - A client who has hypertension requires treatment with diet, exercise, and antihypertensive meds D -The client who has alcohol use disorder ca requires folic acid therapy. Excess alcohol consumption leads to poor dietary intake of folic acid and injury to the liver. E - T2DM is treated with diet, exercise, and hyperglycemia meds

E - 54 A client is experiencing an attack of acute pancreatitis. Which nursing intervention is the highest priority for this client? A - Assist the client to assume a position of comfort. B - Administer opioid analgesic medication. C - Do not administer food or fluids by mouth. D - Measure intake and output every shift.

B - Administer opioid analgesic medication. Pain relief is the highest priority for the client with acute pancreatitis.Although measuring intake and output, NPO status, and positioning for comfort are all important, they are not the highest priority.

ATI - P A nurse is caring for a client who has peptic ulcer disease. The nurse should monitor the client's phosphorus levels when administering which of the following drugs? A - Omeprazole B - Aluminum hydroxide C - Sucralfate D - Famotidine

B - Aluminum hydroxide Rational A - Administering omeprazole, a proton pump inhibitor, does not require monitoring of phosphorus levels. However, long-term therapy with the drug can cause bone loss, so it requires monitoring of bone density. B - Antacids that contain aluminum, such as aluminum hydroxide, can cause hypophosphatemia because of aluminum's ability to bind with phosphate and decrease its absorption. The nurse should monitor the client's phosphorus levels while administering this drug. C - Administering sucralfate, a mucosal protectant, does not require monitoring of phosphorus levels. However, sucralfate therapy does require monitoring of bowel function. D - Administering famotidine, a histamine2-receptor antagonist, does not require monitoring of phosphorus levels. However, famotidine therapy does require monitoring of the client's complete blood count periodically during therapy because it can cause anemia.

ATI 47 A nurse is providing care to a client who is 1 day postoperative following a paracentesis. The nurse observes clear, pale, yellow fluid leaking from the operative site. Which of the following is an appropriate nursing intervention A - Place a clean towel near the drainage site B - Apply a dry, sterile dressing C - Apply direct pressure to the site D - Place the client in a supine position.

B - Apply a dry, sterile dressing Rational A - Cover the op site to prevent infection and allow for assessment of drainage B - application of a sterile dressing will contain the drainage and allow continuous assessment of color and quantity C - Direct pressure can cause discomfort and potential harm D - Place the client with the head of the bed elevated to promote lung expansion

ATI - P A nurse should recognize that sulfasalazine is contraindicated for clients who have which of the following conditions? A - Pancreatitis B - Aspirin sensitivity C - Bronchitis D - GERD

B - Aspirin sensitivity Rational A - Clients who have pancreatitis may take sulfasalazine, a 5-aminosalicylate. Pancreatitis is a potential adverse drug reaction of azathioprine, another GI-system drug. B - Any sensitivity to salicylates, sulfonamides, or trimethoprim is a contraindication for the use of sulfasalazine, a 5-aminosalicylate. This is because intestinal bacteria metabolize the drug into 5-aminosalicylic acid, a salicylate. Aspirin is also a salicylate. C - Clients who have bronchitis may take sulfasalazine, a 5-aminosalicylate. However, this drug is contraindicated for clients who have an intestinal obstruction. D - Clients who have GERD may take sulfasalazine, a 5-aminosalicylate. However, this drug is contraindicated for clients who have a bowel obstruction.

E - 51 A client has a nasogastric tube (NGT) connected to low continuous suction. What is the nurse's priority to ensure client safety? A - Assess for peristalsis at least once every 8 to 12 hours. B - Assess placement of the NGT for placement every 4 hours. C - Measure the gastric drainage every 8 to 12 hours and document. D - Monitor the nasal skin and membranes around the tube for irritation.

B - Assess placement of the NGT for placement every 4 hours. Assessing the NGT for placement every 4 hours can help prevent aspiration which could lead to pneumonia. The other actions are appropriate for some clients, checking tube placement is the priority for care.

ATI A nurse is assessing a client who has peritonitis. Which of the following findings should the nurse expect? A - Bloody diarrhea B - Board-like abdomen C - Periumbilical cyanosis D - Increased bowel sounds

B - Board-like abdomen Rational A - Bloody diarrhea is an expected finding for a client who has colorectal cancer. B - A board-like, distended abdomen, accompanied by extreme pain and tenderness, is an expected finding for a client who has peritonitis. C - Periumbilical cyanosis is an expected finding for a client who has pancreatitis. D - Diminished bowel sounds is an expected finding for a client who has peritonitis.

E - 52 The nurse is preparing to administer natalizumab for a client who has Crohn disease (CD). What is the most important client assessment for the nurse to perform before giving this drug? A - Skin integrity B - Body temperature C - Peripheral pulses D - Breath sounds

B - Body temperature Because this drug may cause a deadly infection that affects the brain (progressive multifocal leukencephalopathy [PML]), the nurse would want to ensure that the client does not have any type of infection. Assessing body temperature is one way to determine the presence of infection.

ATI 55 A nurse is assessing a client who has advanced cirrhosis. The nurse should identify which of the following findings as indicators of hepatic encephalopathy? Select all A - Anorexia B - Change in orientation C - Asterixis D - Ascites E - Fetor hepaticus

B - Change in orientation C - Asterixis E - Fetor hepaticus Rational A - anorexia is present in a client who has liver dysnfruncoint, not an indication of hepatic encephalopathy B - change in orientation indicated hepatic encephalopathy in advanced cirrhosis C - Asterixis a coarse tremor of the wrists and fingers, is observed as a late complication D - Ascites can be present in a client who has liver dysfunction, but is not an indication of hepatic encephalopathy E - Fetor hepaticus (a fruit, musty breath odor), is a finding of hepatic encephalology in the client with advanced cirrhosis.

E - 53 The nurse is teaching a client and family about home care following a transjugular intrahepatic portal-systemic shunt (TIPS) procedure. Which client finding would the nurse teach the family to report to the primary health care provider immediately? A - Decreased ascitic fluid B - Changes in consciousness or behavior C - Fatigue and weakness D - Decreased pulse rate

B - Changes in consciousness or behavior Although serious complications of the TIPS are not common, the client needs to be monitored for hepatic encephalopathy. This complication is manifested by changes in consciousness, mental status, and/or behavior. A decreased pulse rate and ascitic fluid are expected and clients with cirrhosis are usually fatigued and weak.

E -54 A client has undergone the Whipple procedure (radical pancreaticoduodenectomy) for pancreatic cancer. Which nursing actions would the nurse implement to prevent potential complications? Select all that apply. A - Ensure that drainage color is clear. B - Check blood glucose often. C - Place the client in the supine position. D - Check bowel sounds and stools. E - Monitor mental status.

B - Check blood glucose often. D - Check bowel sounds and stools. E - Monitor mental status To prevent potential complications after a Whipple procedure, the nurse would check the client's glucose often to monitor for diabetes mellitus. Bowels sounds and stools would be checked to monitor for bowel obstruction. A change in mental status or level of consciousness could be indicative of hemorrhage.Clear, colorless, bile-tinged drainage or frank blood with increased output may indicate disruption or leakage of a site of anastomosis but is not a precautionary action for the nurse to implement. The client should be placed in semi-Fowler and not supine position to reduce tension on the suture line and the anastomosis site and to optimize lung expansion.

E - 52 The nurse is reinforcing teaching provided by the registered dietitian nutritionist about dietary restrictions needed for a client who has a new ileostomy. Although each client can tolerate different foods, what food would the nurse suggest that the client avoid? A - Potatoes B - Corn C - Bread D - Green beans

B - Corn The client should avoid gas-forming foods like cabbage and foods that contain indigestible fiber such as nuts and corn.

E - 52 The nurse is assessing an older client who has had frequent vomiting and diarrhea for the last 24 hours. Which vital sign change would be of most concern to the nurse? A - Increased oxygen saturation B - Decreased blood pressure C - Increased temperature D - Decreased pulse rate

B - Decreased blood pressure Older clients are most at risk for dehydration from loss of fluids. Older clients who have dehydration usually have an increased pulse and decreased blood pressure (BP). When BP decreases, the client is at risk for orthostatic hypotension which can cause dizziness and subsequent falls. The client may also experience an elevated temperature, but this change is less common in older adults when compared to their younger counterparts.

E - 48 The nurse is teaching an older adult client. Which gastrointestinal problem does the nurse discuss that takes place during the normal aging process? A - Increased peristalsis B - Decreased hydrochloric acid levels C - Increased liver size D - Excess lipase production

B - Decreased hydrochloric acid levels In older adults, decreased hydrochloric acid levels (hypochlorhydria) results from atrophy of the gastric mucosa.A decrease in lipase production results from calcification of pancreatic vessels. A decrease in the number and size of hepatic cells leads to decreased liver weight and mass. Peristalsis decreases, and nerve impulses are dulled.

E - 53 The nurse is caring for a client who just had a paracentesis. Which client finding indicates that the procedure was effective? A - Increased blood pressure B - Decreased weight C - Increased pulse D - Decreased pain

B - Decreased weight A paracentesis is performed to remove ascitic fluid from the abdomen. Therefore, the client should weigh less after the procedure than before. Blood pressure should decrease due to less fluid volume and the pulse rate may not be affected. The client may report less abdominal discomfort or ease in breathing, but pain is not a common problem for cirrhotic clients.

E - 55 An older adult with severe rheumatoid arthritis in the upper extremities is undernourished. What does the nurse anticipate may be a contributing factor? A - Inadequate income to purchase sufficient food B - Diminishing ability to manipulate eating utensils C - A decrease in appetite D - Metabolic requirements that are increased due to immobility

B - Diminishing ability to manipulate eating utensils The client's severe rheumatoid arthritis in the hands and arms would produce a decrease in the client's ability to manipulate utensils.No evidence suggests that the client is experiencing a decrease in appetite or is financially unable to purchase adequate food. No evidence suggests that the client is immobile. Metabolic requirements would decrease, not increase, with less mobility.

E - 55 The nurse is monitoring a client who is receiving an intravenous fat emulsion (IVFE) nutritional supplement. What action does the nurse take after reviewing the client's laboratory report, and seeing an increase in triglycerides? Select all that apply. A - Document the findings and continues to monitor. B - Discontinue the IVFE infusion. C - Slow the rate of flow of the IVFE infusion. D - Offer small bites of oral foods. E - Notify the health care provider.

B - Discontinue the IVFE infusion. E - Notify the health care provider. If a client receiving an IVFE nutritional supplement develops fever, increased triglycerides, clotting problems, or symptoms of multi-system organ failure, the nurse must discontinue the IVFE and notify the HCP. These symptoms may indicate fat overload syndrome, especially in a critically ill patient.Only documenting the findings and continuing to monitor could have serious repercussions for this client. Slowing the rate of flow of the IVFE infusion, or offering small bites of oral foods, can also present a serious safety risk.

ATI 52 A nurse is completing discharge teaching with client who has Crohn's disease. Which of the following instructions should the nurse include in the teaching? A - Decrease intake of calorie-dense foods B - Drink canned protein supplements C - Increase intake of high fiber foods D - Eat high-residue foods

B - Drink canned protein supplements Rational A - high-protein diet is recommended C - Low fiber is recommended for Crohn's D - East low-residue foods to reduce inflammation

ATI 30 A nurse is caring for a client who is receiving IV potassium. The nurse should monitor the client for which of the following manifestations as an indication of hyperkalemia? A - Tachycardia B - Dyspnea C - Lethargy D - Increased thirst

B - Dyspnea Rational A - Bradycardia is a manifestation of hyperkalemia B - Monitor the client for dyspnea as a manifestation of hyperkalemia C - Anxiety is a manifestation of hyperkalemia D - Vomiting is a manifestation of hyperkalemia

E - 53 The nurse is assessing a client who is diagnosed as having Hepatitis A and asks how someone gets this disease. What is the most likely cause of the client's Hepatitis A? A - Being exposed to blood or blood products B - Eating contaminated food or water C - Having unprotected sex D - Sharing needles for illicit drugs

B - Eating contaminated food or water Hepatitis A is transmitted through the fecal-oral route rather than via blood. Therefore, contaminated food or water with Escherichia coli or other microbes can cause this liver infection.

ATI A nurse is providing dietary teaching for a client who is postoperative following a gastrectomy. Which of the following foods should the nurse encourage the client to include in their diet to reduce the risk for dumping syndrome? A - Ice cream B - Eggs C - Grape juice D - Honey

B - Eggs Rational A - The nurse should instruct the client to avoid eating ice cream, a food that is high in sugar, due to the client's increased risk for manifestations of dumping syndrome. B - The nurse should instruct the client to increase dietary intake of protein-containing foods, such as eggs, to decrease the risk for manifestations of dumping syndrome. The client should eat some form of protein at each meal. C - The nurse should instruct the client to avoid drinking sweetened fruit juice due to the client's increased risk for manifestations of dumping syndrome. D - The nurse should instruct the client to avoid consuming honey and other simple sugars due to the client's increased risk for manifestations of dumping syndrome.

E - 51 A client with a family history of colorectal cancer (CRC) regularly sees a primary health care provider for early detection of any signs of cancer. Which laboratory result may be an indication of CRC in this client? A - Decrease in liver function test results B - Elevated carcinoembryonic antigen C - Negative test for occult blood D - Elevated hemoglobin levels

B - Elevated carcinoembryonic antigen Carcinoembryonic antigen may be elevated in many patients diagnosed with CRC. Liver involvement may or may not occur in CRC. Hemoglobin will likely be decreased with CRC, not increased. An occult blood test is not reliable to affirm or rule out CRC.

ATI 52 A nurse is reveiwing the laboratory data of a client who has an acute exacerbation of Crohn's disease. Which of the following blood laboratory results should the nurse expect to be elevated? Select all A - Hematocrit B - Erythrocyte sedimentation rate C - WBC D - Folic acid E - Albumin

B - Erythrocyte sedimentation rate C - WBC Rational A - Hematocrit will decrease D - Folic acid levels decrease = malabsorption E - Albumin decrease = malabsorption

ATI 50 A nurse is teaching about pernicious anemia with a client who has chronic gastritis. Which of the following information should the nurse include in the teaching A - Pernicious anemia is caused when the cells producing gastric acid are damaged B - Expect a monthly injection of vitamin B12 C - Plan to take vitamin K supplements D - Pernicious anemia is caused by an increase production of intrinsic factor.

B - Expect a monthly injection of vitamin B12 Rational A - Damage to parietal cells have occurred, which leads to pernicious anemia and causes a decrease of the intrinsic factor by the stomach parietal cells B - monthly injections of vitamin B12 to treat pernicious anemia due to. decrease of the intrinsic factor by the stomach parietal cells C - Vitamin K supplements are given to client who have bleeding disorder D - Parietal cells damage results in insufficient production of intrinsic factor by the stomach parietal cells

ATI A nurse is providing teaching for a client who has cirrhosis and a new prescription for lactulose. The nurse should include which of the following instructions in the teaching? A - Notify the provider if bloating occurs. B - Expect to have two to three soft stools per day. C - Restrict carbohydrates in the diet. D - Limit oral fluid intake to 1,000 mL per day of clear liquids.

B - Expect to have two to three soft stools per day. Rational A - The nurse should instruct the client that bloating, flatulence, and belching are adverse effects of lactulose and that it is not necessary to notify the provider about these adverse effects. B - The purpose of administering lactulose is to promote the excretion of ammonia in the stool. The nurse should instruct the client to take the medication every day and inform the client that two to three bowel movements every day is the treatment goal. C - The nurse should instruct the client to follow a diet that is high in carbohydrates and protein because clients who have cirrhosis are at risk for malnutrition. D -Lactulose can cause increased stool frequency, which results in dehydration. The nurse should instruct the client to maintain an adequate fluid intake to offset the dehydrating effects of the medication.

ATI A nurse is providing discharge teaching for a client who has a new prescription for medications to treat peptic ulcer disease. The nurse should inform the client that which of the following medications inhibits gastric acid secretion? A - Calcium carbonate B - Famotidine C - Aluminum hydroxide D - Sucralfate

B - Famotidine Rational A - The nurse should inform the client that calcium carbonate is an antacid that neutralizes gastric acid but does not inhibit its secretion. B - The nurse should inform the client that famotidine is an H2-receptor antagonist that is prescribed for the treatment of peptic ulcer disease to inhibit the secretion of gastric acid. C - The nurse should inform the client that aluminum hydroxide is an antacid that neutralizes gastric acid but does not inhibit its secretion. D - The nurse should inform the client that sucralfate is a mucosal barrier fortifier that forms a protective coating over the ulcer but does not inhibit gastric acid secretion.

E - 52 The nurse is teaching a group of senior citizens in a residential facility about how to prevent gastrointestinal (GI) infectious outbreaks, such as norovirus. What information will the nurse include as a priority intervention for the group? A - Keeping at least 6 feet apart B - Handwashing and hand sanitizing C - Avoiding group dining D - Cooking all food and boiling water

B - Handwashing and hand sanitizing GI infections like norovirus are typically transmitted via the fecal-oral route. Therefore, handwashing and using hand sanitizers frequently is the best method to promote health and prevent infection.

ATI A nurse is reviewing the laboratory results of a client who has acute pancreatitis. Which of the following findings should the nurse expect? A - Blood glucose 110 mg/dL B - Increased amylase C - WBC count 9,000/mm3 D - Decreased bilirubin

B - Increased amylase Rational A- A blood glucose level of 110 mg/dL is within the expected reference range. Elevated blood glucose is an expected finding for a client who has acute pancreatitis. B - Serum amylase levels are increased in a client who has acute pancreatitis due to pancreatic cell injury. C - A WBC count of 9,000/mm3 is within the expected reference range of 5,000-10,000/mm3. An elevated WBC is an expected finding in a client who has acute pancreatitis. D - Increased bilirubin is an expected finding for a client who has acute pancreatitis due to the hepatobiliary obstructive process.

E - 52 A client developed gastroenteritis while traveling outside the country. What is the most likely cause of the client's symptoms? A - Overcooked food B - Ingestion of parasites in the water C - Insufficient vaccinations D - Bacteria on the patient's hands

B - Ingestion of parasites in the water The likely cause of gastroenteritis when a client travels outside the country is ingestion of water that is infested with parasites. Bacteria on the client's hands will not produce gastroenteritis unless food or water is contaminated with the bacteria. Insufficient vaccinations may cause other disease processes, but not gastroenteritis. Undercooked, not overcooked, food may produce gastroenteritis.

ATI 22 A nurse is caring for a child who has had watery diarrhea for the past 3 days. Which of the following is an action for the nurse to take? A - Offer chicken broth B - Initiate oral rehydration therapy C - State hypertonic IV solution D - Keep NPO until the diarrhea subsides.

B - Initiate oral rehydration therapy Rational A- Chicken broth is avoided for children who have diarrhea because of its increases sodium and inadequate carbohydrates B - Oral rehydration iis recommended to replace lost electrolytes C - Isotonic IV solution are recommended for children who experience severe dehydration D - Children who experience diarrhea are at risk for dehydration

E - 48 The nurse and health care provider are discussing a client who has pernicious anemia. The nurse anticipates that the client has which deficiency? A - Hydrochloric acid B - Intrinsic factor C - Glucagon D - Pepsinogen

B - Intrinsic factor Intrinsic cells are produced by the parietal cells in the stomach. This substance facilitates the absorption of vitamin B12. Absence of intrinsic factor causes pernicious anemia.Glucagon, which is produced by the alpha cells in the pancreas, is essential for the regulation of metabolism. Parietal cells secrete hydrochloric acid, but this does not facilitate the absorption of vitamin B12. Pepsinogen is secreted by the chief cells; pepsinogen is a precursor to pepsin, a digestive enzyme.

ATI 22 A nurse is teaching a group of parents about Salmonella. Which of the following information should thenurse include in the teaching? Select all A - Incubation period is nonspecific B - It is a bacterial infection C - Bloody diarrhea is common D - Transmission can be form house pets E - ANtibiotics are used for treatment.

B - It is a bacterial infection C - Bloody diarrhea is common D - Transmission can be form house pets Rational A - Incubation for Salmonella is 6 to 27 hr B - Salmonella is classified as bacterial infection C - Salmonella manifestations include bloody diarrhea, nausea, vomiting, and abdominal cramping D - Salmonella can be transmitted to children form household pets, cats, dogs, hamsters, turtles E - Salmonella is a bacterial infection. Antibiotics are prescribed unless complications occur

ATI - P A nurse is reviewing the medical record of a client who has a new prescription for famotidine. The nurse should recognize that which of the following drugs interacts with famotidine? A - Phenobarbital sodium B - Ketoconazole C - Lisinopril D - Hydrochlorothiazide

B - Ketoconazole Rational A - Phenobarbital sodium does not interact with famotidine. However, it interacts with several types of drugs, including anticoagulants, oral contraceptives, and anticonvulsants. B - Famotidine reduces the absorption of ketoconazole. C - Lisinopril does not interact with famotidine. However, lisinopril interacts with lithium carbonate and can cause lithium toxicity. D - Hydrochlorothiazide does not interact with famotidine. However, it acts by promoting potassium loss and increases the risk of digoxin toxicity.

E - 52 The nurse is caring for a client admitted with a long-term diagnosis of ulcerative colitis (UC). For what potentially life-threatening complication would the nurse monitor? A - Chronic kidney disease B - Lower gastrointestinal (GI) bleeding C - Metabolic acidosis D - Hyperkalemia

B - Lower gastrointestinal (GI) bleeding The client who has UC is at most risk for lower GI bleeding due to inflammation and diarrhea. The client with UC is also at risk for hypokalemia and metabolic alkalosis as a result of losing intestinal contents through diarrhea.

ATI A nurse is reviewing the prescriptions for a client who has Campylobacter enteritis. Which of the following prescriptions should the nurse clarify with the provider? A - 0.45% sodium chloride IV B - Magnesium hydroxide C - Ciprofloxacin D - Potassium

B - Magnesium hydroxide Rational A - Nausea, vomiting, and diarrhea are manifestations of enteritis. The nurse should recognize that a prescription for an IV administration of 0.45% sodium chloride, a hypotonic solution, is prescribed to reverse the effects of dehydration and does not require clarification. B - Nausea, vomiting, and diarrhea are manifestations of enteritis. The nurse should clarify a prescription for magnesium hydroxide, also known as milk of magnesia, with the provider. This medication increases gastrointestinal motility, which can increase the client's risk for an electrolyte imbalance and contribute to dehydration. C - The nurse should recognize that a prescription for ciprofloxacin, an antibiotic, is expected and does not require clarification because Campylobacter enteritis is a bacterial form of gastroenteritis. D - Nausea, vomiting, and diarrhea are manifestations of enteritis, which places the client at risk for hypokalemia. The nurse should recognize that a prescription for potassium is expected and does not require clarification.

ATI 29 A nurse is caring for a client who has diabetes and is experience nausea due to gastroparesis. The nurse should expect a prescription for which of the following medications? A - Lubiprostone B - Metoclopramide C - Bisacodyl D - Loperamide

B - Metoclopramide Rational A -Lubiprostone is a med used to treat IBS-C in women B - Metoclopramide is a dopamine antagonist that is used to treat nausea and also increases gastric motility. It can relieve the bloating and nausea of diabetic gastroparesis. C - Bisacodyl is a stimulant laxative used for short-term constipation D - Loperamide is an antidiarrheal agent that decreases GI peristalsis

ATI 54 A nurse is preparing to administer pancrelipase to a client who has pancreatitis. Which of the following actions should the nurse take? A - Instruct the client to chew the medication before swallowing B - Offer a glass of water following medication administration C - Administer the medication 30 min before meals D - Sprinkle the contents on peanut butter

B - Offer a glass of water following medication administration Rational A - should be swallowed w/o chewing to reduce irritations and slow the release C - admin with every meal and snack D - can be sprinkled on non protein foods - peanut butter is a protein

ATI 48 A nurse is completing an assessment of a client who has GERD. Which of the following is an expected finding? A - Absence of saliva B - Painful swallowing C - Sweetin taste in mouth D - Absence of eructation

B - Painful swallowing Rational A - Hypersalivation is an expected findings B - Painful swallowing is a manifestation of GERD due to esophageal stricture or inflammation C - Clients with GERD report bitter taste D - Increased burping is an expected findings

ATI 23 A nurse is caring for an infant who has just returned from PACU following cleft lip and palate repair. Which of the following actions should the nurse take? A - Remove the packing in the mouth B - Place the infant in an upright position C - Offer a pacifier with sucrose D - Assess the mouth with a tongue blade.

B - Place the infant in an upright position Rational A - Packing ini mouth should stay in place for 2-3 days B - placing the infant in an upright position will facilitate drainage and prevent aspiration C - Objects in the mouth could injury the surgical site and should be avoided D - Objects in mouth could injury the surgical site and should be avoided

ATI 55 A nurse is caring for a client who has a new diagnosis of Hepatitis C. Which of the following laboratory findings should the nurse expect? A - Presence of immunoglobulin G antibodies (IgG) B - Positive EIA test C - Aspartate aminotransferase (AST) 35 units/L D - Alanine aminotransferase (ALT) 15 IU/L

B - Positive EIA test Rational A - presence of IgG is expect for Hep A B - positive EIA is exped for new diagnosis of hep C C - AST is elevated in client who have Hep C infections. 35 units/L is WNL D - ALT is elevated in client who have Hep C. 15 units/L isi WLN

ATI 23 A nurse is caring for a child who has Hirschsprung's disease. Which of the following actions should the nurse take? A - Encourage a high-fiber, low-protein, low-calorie diet B - Prepare the family for surgery C -Place an NG tube for decompression D - Initiate bed rest

B - Prepare the family for surgery Rational A - Hirschsprung's - low-fiber, high protein , high calorie diet B - Hirschsprung's requires surgery to remove the affected segment of the intestine C - Hirschsprung's disease is managed nutritionally D - Meckel's' diverticulum is placed on bed rest to prevent bleeding

E - 48 A client is being observed after a routine sigmoidoscopy with a tissue biopsy. Which assessment finding will the nurse report to the health care provider? A - Flatulence B - Rectal bleeding C - Mild abdominal pain D - Borborygmi

B - Rectal bleeding Bleeding is a possible complication following a sigmoidoscopy. It must be reported immediately to the health care provider.Mild abdominal pain (usually gas pain) and flatulence are expected findings after a sigmoidoscopy. Borborygmi may be heard, especially if the client is hungry if they have followed a clear liquid diet prep before the procedure.

E - 55 A client who is receiving total enteral nutrition exhibits acute confusion and shallow breathing and says, "I feel weak." As the client begin to experience a seizure, how does the nurse interpret this client's signs and symptoms? A - Abdominal distention is present. B - Refeeding syndrome may be occurring. C - Severe hyperglycemia is present. D - The enteral tube is dislodged.

B - Refeeding syndrome may be occurring. Refeeding syndrome is a syndrome consisting of metabolic disturbances that occur as a result of reinstitution of nutrition to clients who are starved, severely malnourished, or metabolically stressed due to severe illness. Symptoms of refeeding syndrome include heart failure, peripheral edema, rhabdomyolysis, seizures, and hemolysis.If the enteral tube becomes misplaced or dislodged, the client may develop aspiration pneumonia displayed by increased temperature, increased pulse, dehydration, diminished breath sounds, and shortness of breath. Abdominal distention is most frequently accompanied by nausea and vomiting. In refeeding syndrome, insulin secretion decreases in response to the physiologic changes in the body, so hyperglycemia is not present. When refeeding begins, insulin production resumes and the cells take up glucose and electrolytes from the bloodstream, thus depleting serum levels, resulting in hypoglycemia.

ATI 53 A nurse is providing discharge teaching to a client who is postoperative following a laparoscopic cholecystectomy. Which of the following instructions should the nurse include in the teaching? Select all A - Take baths rather than showers B - Resume a diet of choice C - Cleanse the puncture site using mild soap and water D -Remove adhesive stritps form the puncture site in 24 hrs E - Report nausea and vomiting to the surgeon

B - Resume a diet of choice C - Cleanse the puncture site using mild soap and water E - Report nausea and vomiting to the surgeon Rational A - can take a bath or shower 1-2 days following surgery D - strips should remain in place until they fall off naturally

E - 50 The nurse is caring for a client who is diagnosed with a perforated duodenal ulcer. Which assessment finding would the nurse expect? A - Positive McBurney point B - Rigid, board-like and tender abdomen C - Nausea and profuse vomiting D - Absent bowel sounds in all four quadrants

B - Rigid, board-like and tender abdomen Perforation allows intestinal contents to escape into the peritoneal cavity causing peritonitis. The classic assessment finding for a client who has peritonitis is a rigid, board-like abdomen that is tender or painful.

ATI - P A nurse is administering sulfasalazine to a client. Which of the following data should the nurse collect to help identify an adverse drug reaction? (Select all that apply.) A - Level of consciousness B - Skin integrity C - Temperature D - Urine output E - CBC

B - Skin integrity C - Temperature E - CBC Rational A - Level of consciousness is incorrect. Sulfasalazine, a 5-aminosalicylate, is unlikely to affect level of consciousness. Metoclopramide, a dopamine antagonist, is a GI-system drug that can cause sedation. B - Skin integrity is correct. Sulfasalazine can cause a skin rash, so the nurse should check the client's skin for rashes. The drug can also cause nausea. If the client reports nausea, the nurse should suggest taking the drug with food or water. C - Temperature is correct. Sulfasalazine can cause a fever, so the nurse should check the client's temperature and treat fever with an antipyretic. D - Urine output is incorrect. Sulfasalazine is unlikely to affect urine output, although it can cause an orange-yellow discoloration of urine and skin. E - CBC is correct. Sulfasalazine can cause hematologic disorders, such as agranulocytosis and hemolytic and macrocytic anemia. The nurse should check the client's CBC periodically during therapy and tell the client to report sore throat or fatigue.

E - 52 An older client with a 2-day history of myalgia, nausea, vomiting, and diarrhea is admitted to the medical-surgical unit with a diagnosis of gastroenteritis. Which primary health care provider request does the nurse implement first? A - Obtain a stool specimen for culture and sensitivity. B - Start an IV solution of 5% dextrose in 0.45 normal saline. C - Draw blood for a complete blood count and serum electrolytes. D - Administer acetaminophen 650 mg rectally.

B - Start an IV solution of 5% dextrose in 0.45 normal saline. The request the nurse implements first is to start an IV solution of 5% dextrose in 0.45 normal saline at 125 mL/hr. Although the dextrose 5% in 0.45% sodium chloride is hypertonic in the IV bag, once it is infused, the glucose is rapidly metabolized and the fluid is really hypotonic. Fluid therapy is the focus of treatment for clients with gastroenteritis. Older clients are at increased risk for the complications of dehydration such as hypovolemia and acute kidney failure.Acetaminophen 650 mg should be administered rectally soon, and blood draws and stool specimen collection would also be implemented soon, but prevention and treatment of dehydration are the priorities for this client.

ATI - P A nurse is providing teaching to a client who has a new prescription for omeprazole to treat a duodenal ulcer. Which of the following instructions should the nurse include? A - Take the medication with food. B - Swallow the capsules whole. C - Dissolve the tablets in water. D - Take the medication at bedtime.

B - Swallow the capsules whole. Rational A - Food can reduce the absorption of omeprazole, a proton pump inhibitor. Clients should take omeprazole prior to meals. B - Omeprazole, a proton pump inhibitor, is unstable in stomach acid. The nurse should instruct the client to swallow the capsules or tablets whole and not to crush or chew the capsules or tablets. C - Omeprazole, a proton pump inhibitor, is available in delayed-release tablets, capsules, or powder for oral suspension. If taking the tablet formulation, the client should swallow them whole because gastric acid can decrease the effectiveness of the medication. D - Omeprazole, a proton pump inhibitor, decreases the secretion of gastric acid. It is most effective when clients take it in the morning prior to the first meal of the day.

E - 53 When caring for a client with Laennec cirrhosis, which of these findings does the nurse expect to find on assessment? Select all that apply. A - Elevated magnesium B - Swollen abdomen C - Prolonged partial thromboplastin time D - Elevated amylase level E - Currant jelly stool F - Icterus of skin

B - Swollen abdomen C - Prolonged partial thromboplastin time F - Icterus of skin Clients with Laennec cirrhosis have damaged clotting factors, so prolonged coagulation times and bleeding may result. Icterus, or jaundice, results from cirrhosis. The client with cirrhosis may develop ascites, or fluid in the abdominal cavity.Elevated magnesium is not related to cirrhosis. Amylase is typically elevated in pancreatitis. Currant jelly stool is consistent with intussusception, a type of bowel obstruction. The client with cirrhosis may develop hypocalcemia and/or hypokalemia. It is also consistent with elevations of aspartate aminotransferase (AST), alanine aminotransferase (ALT), and alkaline phosphatase.

ATI 51 A nurse is assessing a client in an extended care facility. The nurse should recognize which of the following findings is a manifestation of an obstruction of the large intestine due to a fecal impaction? A - The client reports one bowel movement yesterday B - The client is having small, frequent liquid stools C - The client is flatulent D - The client indicated vomiting once this morning.

B - The client is having small, frequent liquid stools Rational A - a bowel movement yesterday does not indicate a mechanical obstruction of the large intestine due to a fecal impaction B - Small, frequent liquid stools can be passed around a fecal impaction C - The presence of flatus does not indicate a mechanical obstruction D - A report of a single episode of vomiting does not indicate a mechanical obstruction of the large intestine due to a fecal impaction

ATI 28 A nurse is teaching a client who has a new prescription for omeprazole. Which of the following information should the nurse include in the teaching? A - Take this medication at bedtime. B - This medication decreases the production of gastric acid C - Take this medication 2 hr after eating D - This medication can cause hyperkalemia

B - This medication decreases the production of gastric acid Rational A - Omeprazole is admin in the morning B - Omeprazole reduces gastric acid secretion by inhibiting the enzyme that produces gastric acid C - Omeprazole is amin before meals with a glass of water D - Omeprazole can cause hypomagnesemia

E - 55 The nurse obtains assessment data on a client who had bariatric surgery today. Which finding does the nurse report to the surgeon immediately? A - Reports pain of "6" on 0-10 scale when being repositioned. B - Urine output total is 15 mL for the past 2 hours. C - Skin under the panniculus is excoriated. D - Bowel sounds are not audible in all quadrants.

B - Urine output total is 15 mL for the past 2 hours. The nurse reports a urine output total of 15 mL for the past 2 hours. Normal urine output needs to be at least 30 mL/hr. Oliguria (scant urine output) may indicate severe postoperative complications such as anastomotic leaks or acute kidney failure.Inaudible bowel sounds would typically require intervention, but on the day of surgery, absent bowel sounds are an expected finding. The other findings, excoriated skin under the panniculus and subjective reports of pain, may require nursing interventions, but do not require an immediate report to the surgeon.

ATI 51 A nurse is completing an admission assessment for a client who has a small bowel obstruction. Which of the followings findings should the nurse report to the provider? Select all A - Emesis prior to insertion of the nasogastric tube B - Urine specific gravity 1.040 C - Hematocrit 60% D - Blood potassium 3.0 mEq/L E - WBC, 10,000/uL

B - Urine specific gravity 1.040 C - Hematocrit 60% D - Blood potassium 3.0 mEq/L Rational A - expected B - Urine specific gravity - 1.005 - 1.030 C - Hct males 42-52% Females 37-47% D - Potassium 3.5 - 5.0 mEq/L E - WBC's- 10,000/mm3

49-4 Which client statement about GERD triggers requires further nursing teaching? Select all that apply. A. "I will decrease my alcohol intake." B. "Smoking 1-2 cigarettes a day won't hurt." C. "My plan is to eat six small meals daily." D. "Tomato-based foods should be avoided."' E. "I love soda but I am going to stop drinking it." F. "Our family eats Mexican dishes several times weekly."

B. "Smoking 1-2 cigarettes a day won't hurt." F. "Our family eats Mexican dishes several times weekly." Rational The nurse will need to provide further teaching when the client states that continued smoking is acceptable, because smoking influences the tone of the lower esophageal sphincter (LES). The nurse will also provide further teaching regarding spicy dishes like Mexican foods, as spicy foods can increase episodes of GERD.

Mastery 50-1 The primary health care provider prescribes bismuth subsalicylate for a client as part of treating H.pylori infection. What health teaching will the nurse include for the client about this drug? A. "Do not crush this drug before taking." B. "The drug may cause your tongue and stool to turn black." C. "Take the drug at night only." D. "The drug may cause you to have diarrhea."

B. "The drug may cause your tongue and stool to turn black." Rational This medication is available as a tablet or liquid and can be crushed. Therefore, Choice A is not correct. Bismuth can cause the tongue and/or stool to turn black, which is a harmless side effect. Therefore, Choice B is correct.

Mastery 49-3 A public health nurse is assessing community clients for oral health disorders. Which client is identified at highest risk? A. 23-year old with 3 dental fillings B. 34-year-old with schizophrenia C. 55-year-old with stable angina D. 62-year-old with irritable bowel syndrome

B. 34-year-old with schizophrenia Rational The nurse identifies that the client with schizophrenia is at highest risk for oral health disorders. The population with mental health disorders is at high risk for this health deviation. The client who has dental fillings demonstrates receiving dental care, which decreases risk. Stable angina and irritable bowel syndrome do not raise risk for oral health disorders

49-5 A community health nurse is screening clients for esophageal cancer. Which client is identified at highest risk? A. 22-year-old who drinks a glass of beer weekly. B. 44-year-old who smokes a pack of cigarettes daily. C. 50-year-old who takes over-the-counter omeprazole. D. 63-year old who uses protein supplements regularly.

B. 44-year-old who smokes a pack of cigarettes daily. Rational The nurse identifies the 44-year-old patient who smokes a pack of cigarettes daily as at the highest risk for developing esophageal cancer. Drinking a glass of beer weekly is not high consumption; taking over-the-counter omeprazole reflects treatment of GERD; and using protein supplements daily demonstrates addressing the possibility of malnutrition.

Mastery 52-1 The nurse is caring for a client with peritonitis from a perforated appendix. Which abdominal assessment finding will the nurse most likely expect? A. Soft abdomen B. Board-like abdomen C. Slightly distended abdomen D. Absent bowel sounds

B. Board-like abdomen Rationales: A board-like abdomen is one of the cardinal signs of peritonitis (Choice B). Therefore, Choice A is an incorrect response. Bowel sounds may or may not be present and severe distention is common (Choice C and D).

49-1 A nurse is caring for a client with recurrent aphthous stomatitis (RAS) who asks about food choices while healing. Which food will the nurse suggest? A. Half of an orange B. Chocolate pudding C. Chips with hummus D. Glass of tomato juice

B. Chocolate pudding Rational Chocolate pudding is likely to be the most soothing food choice for a client with RAS. The orange and glass of tomato juice are acidic, which will inflict pain. Although plain hummus may be a good choice (as long as it is not flavored with spicy additives), the chips are hard and salty, which will cause oral discomfort.

55-2 Total enteral nutrition (TEN) has been prescribed for a client with terminal cancer. When the nurse notes that no advanced directives are in place yet a durable power of attorney exists, what is the appropriate action? A. Withhold TEN indefinitely B. Contact the durable power of attorney C. Begin administration of TEN immediately D. Turn over care to the interprofessional ethics committee

B. Contact the durable power of attorney Rationale: The nurse must attempt to locate the durable power of attorney (DPOA) to determine the patient's wishes. Life-preserving measures are usually implemented until (1) an advance directive is presented, (2) a DPOA is located, or (3) the interprofessional ethics committee meets to determine the way forward.

52-1 A client had an exploratory laparotomy to treat the cause of peritonitis, and has a large incision that is closed with staples and two abdominal drains. Which finding(s) would the nurse report immediately to the surgeon? Select all that apply. A. Serosanguineous drainage B. Increased abdominal distention C. Fever and chills D. Pain level 2 on a scale of 0 to 10 E. Passing flatus

B. Increased abdominal distention C. Fever and chills Rationales: Serosanguinous drainage (Choice A), a low pain level (Choice D), and passing flatus (Choice E) are normally expected after an exploratory laparotomy. Therefore, these findings are not the correct responses. However, an increase in abdominal distention (Choice B) and the presence of fever and chills (Choice C) could indicate bleeding or bowel perforation and need to be reported immediately to the surgeon.

50-1 The nurse is caring for a client diagnosed with peptic ulcer disease (PUD). For which potential complications will the nurse monitor? Select all that apply. A. Pneumonia B. Peritonitis C. Anemia D. Stroke E. Hypotension F. Cirrhosis

B. Peritonitis C. Anemia E. Hypotension Rational The client who has PUD is at risk for perforation in which GI contents enter the abdomen and cause peritonitis (Choice B). Bleeding occurs as a result causing decreased hematocrit (anemia) and blood pressure (hypotension) (Choices C and E). Choice A is an incorrect response because the lungs are located in the thoracic cavity and not in the abdominal cavity where the stomach and intestines are located. Therefore, Choice A is an incorrect response. A stroke results as a lack of perfusion to the brain which is not related to PUD. (Choice D). Cirrhosis is a primary liver disease and is not a complication of PUD (Choice F).

ATI A nurse is teaching a client how to prepare for a colonoscopy. Which of the following instructions should the nurse include in the teaching? A - "Refrain from eating or drinking for 2 hr prior to the procedure." B - "Stop taking aspirin the day before the procedure." C - "Drink clear liquids for 24 hr prior to the procedure, and then take nothing by mouth for 6 hr before the procedure." D - "Drink the oral liquid preparation for bowel cleansing slowly the night before the procedure."

C - "Drink clear liquids for 24 hr prior to the procedure, and then take nothing by mouth for 6 hr before the procedure." Rational A - The nurse should instruct the client to have nothing to eat or drink, except water, for 4 to 6 hr prior to the procedure. B - The nurses should instruct the client to discontinue the use of aspirin, as well as anticoagulants and antiplatelet medications, several days prior to the test to decrease the risk of bleeding. C - The nurse should instruct the client to drink clear liquids for 24 hr prior to the colonoscopy to promote adequate bowel cleansing. D - The nurse should instruct the client to drink the oral liquid preparation quickly to prevent nausea.

E - 50 The nurse is teaching a client about dietary choices to prevent dumping syndrome after gastric bypass surgery. Which statement by the client indicates a need for further teaching? A - "I cannot drink alcohol at all." B - "I will need to avoid sweetened fruit juice beverages." C - "I can eat ice cream in moderation." D - "It is okay to have a serving of sugar-free pudding."

C - "I can eat ice cream in moderation." A need for further teaching about dietary changes related to dumping syndrome is indicated when the client says that ice cream can be eaten in moderation. Milk products such as ice cream must be eliminated from the diet of a patient with dumping syndrome.

ATI A nurse is providing discharge teaching for a client who has mild diverticulitis. Which of the following statements by the client indicates an understanding of the teaching? A - "I may experience right lower quadrant pain." B - "I will remain active by working in my garden every day." C - "I should eat foods that are low in fiber." D - "I will use a mild laxative every day."

C - "I should eat foods that are low in fiber." Rational A - Left lower quadrant pain is an expected finding in a client who has diverticulitis. B - A client who has diverticulitis should refrain from any activity that increases intra-abdominal pressure because this can result in the perforation of the diverticula. C - The nurse should instruct a client who has diverticulitis to follow a low-fiber diet. When the inflammation subsides, the client should consume foods that are high in fiber. D - A client who has diverticulitis should avoid laxatives, which increase intestinal motility and can exacerbate the adverse effects of diverticulitis.

E - 51 The Certified Wound, Ostomy, and Continence Nurse (CWOCN) is teaching a client with colorectal cancer how to care for a newly created colostomy. Which statement by the client indicates a correct understanding of the necessary self-management skills? A - "If I have any leakage, I'll put a towel over it." B - "I can put aspirin tablets in the pouch in order to reduce odor" C - "I will apply a nonalcoholic skin sealant and let it dry before applying the bag." D - "I will have my spouse change the bag for me."

C - "I will apply a nonalcoholic skin sealant and let it dry before applying the bag." The nurse would teach the client and family to apply a skin sealant (preferably without alcohol) and allow it to dry before application of the appliance (colostomy bag) to facilitate less painful removal of the tape or adhesive. It is not realistic that the spouse will always change the patient's bag and does not reflect correct understanding of self-management skills. A towel is not an acceptable or effective way to cope with leakage. Putting an aspirin in the pouch will not reduce odor and can lead to ulcers in the stoma.offers reassurance and is a "pat" statement, making it nontherapeutic. "Why" questions place patients on the defense and are not therapeutic because they close the conversation.

E - 51 The nurse is teaching an older client how to prevent a stool impaction that can obstruct the intestines. Which statement by the client indicates a need for further teaching? A - "I will drink lots of fluids every day, especially water." B - "I will increase my exercise, especially walking, every day." C - "I will be sure to take a laxative every night to keep my bowels moving." D - "I will try to eat more high-fiber foods, such as raw vegetables and whole grains."

C - "I will be sure to take a laxative every night to keep my bowels moving." All of these statements are correct except that the client should not take laxatives because they can decrease the tone of the abdominal muscles.

ATI A nurse is providing dietary teaching for a client who has a new diagnosis of celiac disease. Which of the following statements by the client indicates an understanding of the teaching? A - "I can return to my regular diet when I am free of symptoms." B - "I will need to avoid taking vitamin supplements while on this diet." C - "I will eat beans to ensure I get enough fiber in my diet." D - "I need to avoid drinking liquids with my meals while on this diet."

C - "I will eat beans to ensure I get enough fiber in my diet." Rational A - Celiac disease is an autoimmune disorder that causes changes to the intestinal mucosa, resulting in an intolerance to gluten, which is found in wheat, barley, and rye. The client should continue to avoid eating foods that contain gluten even when free of symptoms. B - Clients who have celiac disease are at risk for malabsorption of vitamins and minerals; therefore, the client should continue taking vitamin and mineral supplements. C - Clients who have celiac disease must maintain a gluten-free diet, which eliminates fiber-rich whole wheat products. Clients should eat beans, nuts, fruits, and vegetables to ensure an adequate intake of fiber. D - Clients who have dumping syndrome should avoid drinking liquids with their meals to slow the movement of food through the intestinal tract. Clients who have celiac disease do not need to refrain from drinking liquids with meals.

ATI 48 A nurse is completing discharge teaching with a client who is postoperative following fundoplication. Which of the following statements by the client indicates understanding of the teaching A - "When sitting in my lounge chair after a meal, I will lower the back of it." B - "I will try to eat three large meals a day." C - "I will elevated the head of my bed on blocks." D - "I will avoid eating within 1 hour before bedtime."

C - "I will elevated the head of my bed on blocks." Rational A - Client is instructed to remain upright after eating following a fundoplication B - The client is instructed to avoid large meals after a fundoplication C - Elevated head to limit reflux D - Should avoid eating 2 hr before bedtime to reduce risk of reflux

E - 51 The nurse is teaching a client who has undergone a hemorrhoidectomy about a follow-up plan of care. Which statement by the client demonstrates a correct understanding of the nurse's instructions? A - "I will take laxatives after the surgery to 'keep things moving?" B - "To help with the pain, I'll apply ice to the surgical area." C - "I will need to eat a diet high in fiber, including raw vegetables." D - "Limiting my fluids will help me with constipation."

C - "I will need to eat a diet high in fiber, including raw vegetables." The statement that shows that the hemorrhoidectomy patient correctly understands the nurse's instruction is, "I will need to eat a diet high in fiber." A diet high in fiber serves as a natural stool softener and will prevent irritation to hemorrhoids caused by painful bowel movements.Laxatives are discouraged because they can be habit-forming and decrease abdominal muscle tone. Increased amounts of fluids are needed to prevent constipation. Moist heat (sitz baths) will be more effective with postoperative discomfort than cold applications.

E - 51 A male client is scheduled for a minimally invasive inguinal hernia repair (MIIHR). Which statement by the client indicates a need for further teaching about this procedure? A - "I may have trouble urinating immediately after the surgery." B - "My chances of having complications after this procedure are slim." C - "I will need to stay in the hospital overnight." D - "I will not eat after midnight the day of the surgery."

C - "I will need to stay in the hospital overnight." A need for further teaching about MIIHR is when the patient says, "I will need to stay in the hospital overnight." Usually, the patient is discharged 3 to 5 hours after MIIHR surgery.Male patients who have difficulty urinating after the procedure would be encouraged to force fluids and to assume a natural position when voiding. Patients undergoing MIIHR surgery must be NPO after midnight before the surgery. Most patients who have MIIHR surgery have an uneventful recovery.

ATI 49 A nurse is complete discharge teaching for a client who has an infection due to H. pylori. Which of the following statements by the client indicates understanding of the teaching A - "I will continue my prescription for corticosteroids." B - "I will schedule a CT scan to monitor improvement." C - "I will take a combination of medications for treatment." D - "I will have my throat swabbed to reach for this bacteria.

C - "I will take a combination of medications for treatment." Rational A - corticosteroids use is a contributing factor to an infection caused by H. pylori B - EGD is done to evaluate for the presences of H. pylori adn to evaluate effectiveness of treatment C - A combo of antibiotics and a histamine 2 receptor antagonist is used to treat infection caused by H. pylori D - H. pylori is evaluated by obtaining gastric samples, not throat swab

ATI 50 A nurse is teaching a client who has a new prescription for famotidine. Which of the following statements by the client indicates understanding of the teaching A - "The medicine coats the lining of my stomach." B - "The medication should stop the pain right away." C - "I will take my pill at bedtime." D - "I will monitor for bleeding for my nose."

C - "I will take my pill at bedtime." Rational A - Famotidine decrease gastric acid output. It does not have a protective coating action B - Needs to take for several days before pain relief occurs when starting this therapy C - Take a bedtime, which suppresses nocturnal acid production D - Monitor for GI bleeding

ATI 52 A nurse is teaching a client who has a new prescription for sulfasalazine. Which of the following instructions should the nurse include in the teaching? A - "Take the medication 2 hours after eating." B - "Discontinue this medication if you skin turns yellow-orange." C - "Notify the provider if you experience a sore throat." D - "Expect your stools to turn black."

C - "Notify the provider if you experience a sore throat." Rational A - take right after meals with full glass of water B - yellow-orange coloring of skin and urine is harmless C - Sulfasalazine can cause blood dyscrasias - report manifestation of infection D - Can cause thrombocytopenia and bleeding. Black stools are a manifestation of GI bleeding and should be reported

E - 51 A male client's sister was recently diagnosed with colorectal cancer (CRC), and his brother died of CRC 5 years ago. The client asks whether he will inherit the disease. How would the nurse respond? A - "Have you asked your primary health care provider about your chances ?" B - "It is hard to know what can predispose a person to develop a certain disease." C - "The only way to know whether you are predisposed to CRC is by genetic testing." D - "No. Just because they both had CRC doesn't mean that you will have it, too."

C - "The only way to know whether you are predisposed to CRC is by genetic testing." The nurse's best response to the client who asks if he will inherit CRC is "the only way to know whether you are predisposed to CRC is by genetic testing." Genetic testing is the only definitive way to determine whether the patient has a predisposition to develop CRC.

E - 54 A client is preparing to have a hepatobiliary scan (HIDA scan). What health teaching would the nurse include about what the client can expect during the test? A - "This test measures how inflamed your gallbladder and liver may be." B - "You may eat and drink as much as you'd like before you have this test." C - "You will have to lie still for some time while the camera is very close to your body." D - "I need to know if you are allergic to shellfish because the contrast will be iodine-based."

C - "You will have to lie still for some time while the camera is very close to your body." The HIDA scan requires the injection of radioactive medium which is given about 20 minutes before a large camera is positioned very close to the body. The camera moves to assess for biliary flow and to determine if any obstruction is present.

ATI - P A nurse should recognize that misoprostol is contraindicated for a client who has which of the following conditions? A - A seizure disorder B - Rheumatoid arthritis C - A positive pregnancy test D - Heart failure

C - A positive pregnancy test Rational A - Clients who have a seizure disorder can take misoprostol, a prostaglandin E1 analog. Drugs that contain caffeine are contraindicated for clients who have a seizure disorder. B - Clients who have rheumatoid arthritis can take misoprostol, a prostaglandin E1 analog, especially because the drug prevents gastric ulcers that can occur with long-term NSAID use. C -Misoprostol, a prostaglandin E1 analog, is a teratogenic drug. It can cause uterine contractions and induce spontaneous abortion; therefore, providers must confirm that clients are not pregnant before prescribing the drug; and clients who take misoprostol must use contraception. D - Clients who have heart failure can take misoprostol, a prostaglandin E1 analog. Metoclopramide, another GI-system drug, is contraindicated for clients who have heart failure and hypertension.

ATI - P A nurse is caring for a client who has a new prescription for alosetron to treat irritable bowel syndrome. The nurse should instruct the client to report which of the following adverse effects of the drug? A - Headache B - Drowsiness C - Abdominal pain D - Sore throat

C - Abdominal pain Rational A - Headache is not an adverse effect of alosetron, a serotonin 5-HT3 receptor antagonist. Ondansetron, a serotonin antagonist, is a GI-system drug that can cause headaches. B - Drowsiness is not an adverse effect of alosetron, a serotonin 5-HT3 receptor antagonist. Alosetron is more likely to cause anxiety than drowsiness. C - Alosetron, a serotonin 5-HT3 receptor antagonist, can cause ischemic colitis. The nurse should tell the client to report abdominal pain, bloody diarrhea, or rectal bleeding, and to stop taking the drug if these manifestations occur. D - Alosetron, a serotonin 5-HT3 receptor antagonist, is unlikely to cause a sore throat or infections. Sulfasalazine, a 5-aminosalicylate, is a GI-system drug that can cause agranulocytosis and has reportable adverse effects including sore throat and fatigue.

E - 53 Which action by the nurse would most likely help to relieve symptoms associated with ascites? A - Monitoring serum albumin levels B - Lowering the head of the bed C - Administering oxygen therapy D - Administering intravenous fluids

C - Administering oxygen therapy The best action by the nurse caring for a client with ascites is to elevate the head of the bed and provide supplemental oxygen. The enlarged abdomen of ascites limits respiratory excursion. Fowler position will increase excursion and reduce shortness of breath.Monitoring serum albumin levels will detect anticipated decreased levels associated with cirrhosis and hepatic failure but does not relieve the symptoms of ascites. Administering IV fluids will contribute to fluid volume excess and fluid shifts into the peritoneal cavity, worsening ascites.

ATI 28 A nurse is teaching a client who takes phenytoin and has a new prescription for sucralfate tablets. Which of the following instructions should the nurse include. A - Take an antacid with the sucralfate B - Take sucralfate with a glass of milk C - Allow a 2 hr interval between these medications D - Chew the curalate thoroughly before swallowing.

C - Allow a 2 hr interval between these medications Rational A - Antacids can interfere with the effects of sucralfate. pt should allow 30 min interval between sucralfate and the antacid B - Sucralfate should be taken on an empty stomach, 1 hr before meals C - Sucralfate can interfere with the absorption of phenytoin, so the client should allow a 2 rh interval between teh sucralfate and phenytoin D - Should swallow whole

ATI - P A nurse is providing teaching to a client who has a new prescription for loperamide. Which of the following instructions should the nurse include? A - Dissolve the powder thoroughly in 8 oz (237 mL) of water. B - Take with diphenhydramine to prevent extrapyramidal effects. C - Avoid activities that require alertness. D - Take 30 min before activities that trigger nausea.

C - Avoid activities that require alertness. Rational A - Loperamide, an opioid agonist, is available in tablets, capsules, or liquid form. Psyllium is a GI-system drug that comes in a powder to mix in 8 oz of water. B - Loperamide, an opioid agonist, is unlikely to cause extrapyramidal effects. Metoclopramide is a GI-system drug that can cause extrapyramidal effects and can require taking diphenhydramine to help reverse these effects. C - Loperamide, an opioid agonist, can cause sedation and dizziness. The nurse should instruct the client to avoid taking it before activities that require alertness. D -Loperamide, an opioid agonist, treats diarrhea, not nausea. Dimenhydrinate is a GI-system drug that requires taking it 30 min before activities that trigger nausea.

E - 55 How does the nurse accurately calculate a client's body mass index (BMI)? A - BMI = weight (lb)/height (in inches)2 B - BMI = weight (kg)/height (in meters) C - BMI = weight (kg)/height (in meters)2 D - BMI = weight (lb)/height (in meters)

C - BMI = weight (kg)/height (in meters)2 The correct formula to accurately calculate a client's body mass index (BMI) is: BMI = weight (kg)/height (in meters)2.

ATI A nurse is assessing a client who has cirrhosis. Which of the following findings is the priority for the nurse to report to the provider? A - Spider angiomas B - Peripheral edema C - Bloody stools D - Jaundice

C - Bloody stools Rational A - Spider angiomas are an expected finding for a client who has cirrhosis. Therefore, there is another finding that is the priority for the nurse to report to the provider. B - Peripheral edema is an expected finding for a client who has cirrhosis. Therefore, there is another finding that is the priority for the nurse to report to the provider. C - The greatest risk to the client who has cirrhosis of the liver is hemorrhagic shock due to bleeding in the esophageal varices. Therefore, bloody stools is the priority finding to report to the provider. D - Jaundice is an expected finding for a client who has cirrhosis. Therefore, there is another finding that is the priority for the nurse to report to the provider.

ATI 30 A nurse is assessing a client who is receiving magnesium sulfate and notes the client has decreased deep tendon reflexes. The nurse should expect to administer which of the following medications? A - Potassium chloride B - Folic acid C - Calcium gluconate D - Cyanocobalamin

C - Calcium gluconate Rational A - Potassium is giving with hypokalemia B - Folic acid is given with alcohol use disorders C - Expect to admin calcium gluconate to a client who is experiencing magnesium toxicity as evidenced by depressed or absent deep tendon reflexes. D - Cyanocobalamin is given with megaloblastic anemia

E - 51 A client who has colorectal cancer is scheduled for a colostomy. Which referral is initially the most important for this client? A - Home health nursing agency B - Social worker C - Certified Wound, Ostomy, and Continence Nurse (CWOCN) D - Hospital chaplain

C - Certified Wound, Ostomy, and Continence Nurse (CWOCN) A CWOCN (or an enterostomal therapist) will be of greatest value to the client with colorectal cancer because the client is scheduled to receive a colostomy.The client is newly diagnosed, so it is not yet known whether home health nursing will be needed. A referral to hospice may be helpful for a terminally ill client. Referral to a chaplain may be helpful later in the process of adjusting to the disease.

ATI 49 A nurse is completing an assessment of a client who has gastric ulcer. Which of the following findings should the nurse expect? Select all A - Client reprots pain relieved by eating B - Client states that pani often occurs at night C - Client reports a sensation of bloating D - Client states that pain occurs 30 - 1 hour after a meal E - Client experiences pain upon palpation of the epigastric region.

C - Client reports a sensation of bloating D - Client states that pain occurs 30 - 1 hour after a meal E - Client experiences pain upon palpation of the epigastric region. Rational A - duodenal ulcer will report pain is relieved by eating B - pain that rarely occurs at night is an expected finding

ATI - 52 A nurse is assessing a client who has been taking prednisone following an exacerbation of inflammatory bowel disease. The nurse should recognize which of the following findings as the priority? A - Client reports difficulty sleeping B - The client's urine isi positive for glucose C - Client reprots having an elevated body temperature D - Client reports gaining 4 lb in the last 6 months

C - Client reprots having an elevated body temperature Rational A - prednisone can cause anxiety and insomnia B - risk for hyperglycemia but not priority C - greatest risk to the client in infection because of immunosuppression D - at risk for weight gain due to prednisone can cause fluid retention

ATI A nurse is providing discharge teaching for a client following an ileostomy. The nurse should instruct the client to report which of the following findings to the provider? A - Intolerance to high-fiber foods B - Liquid ileostomy output C - Dark purple stoma D - Sensation of burning during bowel elimination

C - Dark purple stoma Rational A - Clients who have an ileostomy often experience intolerance to high-fiber foods. The client might need to eliminate these foods from their diet. However, the client does not need to report this intolerance to the provider B - Clients who have an ileostomy are expected to have loose liquid output. The nurse should provide the client with information about how to avoid dehydration due to excessive liquid output, including recommending the consumption of oral replacement solutions. C - The nurse should instruct the client to contact the provider if the stoma is a dark purple color, which is an indication of bowel ischemia. D - Clients who have an ileostomy are expected to experience a burning sensation during bowel elimination due to decreased absorption of gastric acid in the ileum. The nurse should provide the client with instructions about skin care, such as washing the area around the ostomy with warm soap and water after each bowel movement, drying the area gently, and applying a thin coat of ointment to the area.

ATI 53 A nurse in a client is reveiwing the laboratory reports of a client who has suspected cholelithiasis. Which of the following is an expected finding? A - Blood amylase 80 units/L B - WBC 9,000/mm3 C - Direct bilirubin 2.1 mg/dL D - Alkaline phosphatase 25 units/L

C - Direct bilirubin 2.1 mg/dL Rational A - a pt with cholelithiasis has elevated blood amylase. Blood amylase of 80 units/L is WLN B - expect the client who has cholelithiassi to hav ean eleveated WBC due to inflammation. 9,000/mms is WLN C - Expected client who has cholelithiassi to have elevated bilirubin level if the bile duct is obstructed. A direct bilirubin level of 2.1 mg/dL is above expected reference range D - Expect a client who has cholelithiasis to have an elevated phosphate (ALP) level if the common bile duct is obstructed. An ALP of 25 units/L is less than expected reference range.

ATI 28 A nurse is teaching a client about cimetidine. Which of the following adverse effects of cimetidine? Select all A - Increased libido B - Insomnia C - Enlarged of breast tissue D - Confusion E - Decreased sperm count

C - Enlarged of breast tissue D - Confusion E - Decreased sperm count Rational A - decrease libido B - lethargy is an adverse CNS effect C - Gynecomastia is an adverse effect D -Confusion is an adverse CNS effect E - Impotence is an adverse CNS effect

E - 55 A client receiving total parenteral nutrition (TPN) exhibits symptoms of heart failure (CHF) and pulmonary edema. Which complication of TPN does the nurse recognize that the client is experiencing? A - Potassium imbalance B - Fluid volume deficit C - Fluid volume overload D - Calcium imbalance

C - Fluid volume overload This client is most likely experiencing fluid volume overload. Heart failure and pulmonary edema are symptoms of this condition.Calcium imbalance, fluid volume deficit, and potassium imbalance do not manifest with heart failure and pulmonary edema.The nurse tells the client to expect loose stools, abdominal cramps, and nausea. These are side effects unique to orlistat (Xenical).Dry mouth, constipation, and insomnia are not side effects of orlistat. Insomnia, dry mouth, blurred vision, palpitations, constipation, and restlessness are all side effects of short-term therapy drugs such as phentermine (Adipex-P), diethylpropion (Tenuate), and phendimetrazine (Bontril).

ATI 46 A nurse is reviewing the health record of a client who has suspected tumor of the jejunum. The nurse should expect a prescription for which of the following tests? Select all A - Blood alpha-fetoprotein B - Endoscopic retrograde cholangiopancreatography (ERCP) C - Gastrointestinal x-ray with contract D - Small bowel capsule endoscopy (M2A) E - Colonoscopy

C - Gastrointestinal x-ray with contract D - Small bowel capsule endoscopy (M2A) Rational A - used for suspected liver cancer B - used to visualize duodenum, biliary ducts, gallbladder, liver, and pancreas E - Used to visualize rectum, sigmoid, descending, transverse and ascending colon

ATI 54 A nurse is assessing a client who has pancreatitis. Which of the following findings should the nurse identify as a manifestation of pancreatitis? A - Generalized cyanosis B - Hyperactive bowel sounds C - Gray-blue discoloration of the skin around the umbilicus D - Wheezing in the lower lung fields

C - Gray-blue discoloration of the skin around the umbilicus Rational A - expect jaundice B - expect absent or decreased bowel sounds C - Expect to find diminished breath sounds as wella s dyspnea or orthopnea

ATI A nurse is assessing a client who has upper gastrointestinal bleeding. Which of the following findings should the nurse expect? A - Bradycardia B - Bounding peripheral pulses C - Hypotension D - Increased hematocrit levels

C - Hypotension Rational A - A client who has upper gastrointestinal bleeding is at risk for hemorrhagic shock. Tachycardia is a manifestation of hemorrhagic shock. B - A client who has upper gastrointestinal bleeding is at risk for hemorrhagic shock. Weak peripheral pulses are a manifestation of hemorrhagic shock. C - A client who has upper gastrointestinal bleeding is at risk for hemorrhagic shock. Hypotension is a manifestation of hemorrhagic shock. D - A client who has upper gastrointestinal bleeding is at risk for hemorrhagic shock. Decreased hematocrit and hemoglobin levels are manifestations of hemorrhagic shock.

ATI A nurse is admitting a client who has acute pancreatitis. Which of the following actions should the nurse take first? A - Insert a nasogastric tube for the client. B - Administer ceftazidime to the client. C - Identify the client's current level of pain. D - Instruct the client to remain NPO.

C - Identify the client's current level of pain. Rational A - Clients who have acute pancreatitis are at risk for paralytic ileus and might require gastric decompression. The nurse should insert a nasogastric tube, if needed. However, there is another action the nurse should perform first. B - Clients who have acute pancreatitis are at risk for infection. The nurse should administer prescribed antibiotics. However, there is another action the nurse should perform first. C - The first action the nurse should take when using the nursing process is to assess the client. Clients who have acute pancreatitis often have severe abdominal pain. By assessing the client's level of pain, the nurse can identify the need for and implement interventions to alleviate the client's pain. D - Clients who have acute pancreatitis are often placed on NPO status to decrease stress on the pancreas. The nurse should explain this intervention to the client. However, there is another action the nurse should perform first.

E - 53 What teaching does the home health nurse give the family of a client with hepatitis C to prevent the spread of the infection? A - Drink only bottled water and avoid ice. B - Avoid sharing the bathroom with the client. C - Members of the household must not share toothbrushes. D - The client must not consume alcohol.

C - Members of the household must not share toothbrushes. The nurse teaches the family of a client with hepatitis C that toothbrushes, razors, towels, and any other items may spread blood and body fluids and must not be shared.The client should not consume alcohol, but abstention will not prevent spread of the virus. The client may share a bathroom if he or she is continent. To prevent hepatitis A when traveling to foreign countries, bottled water should be consumed and ice made from tap water needs to be avoided.

E - 53 When providing community education, the nurse emphasizes that which group needs to receive immunization for hepatitis B? A - Clients who work with shellfish. B - Clients with elevations of aspartate aminotransferase and alanine aminotransferase. C - Men who engage in sex with men. D - Clients traveling to a third-world country.

C - Men who engage in sex with men. Men who prefer sex with men are at increased risk for hepatitis B, which is spread by the exchange of blood and body fluids during sexual activity.Consuming raw or undercooked shellfish may cause hepatitis A, not hepatitis B. Travel to third-world countries exposes the traveler to contaminated water and risk for hepatitis A. Hepatitis B is not of concern, unless the client is exposed to blood and body fluids during travel. Clients who have liver disease should receive the vaccine, but men who have sex with men are at higher risk for contracting hepatitis B.

E - 48 The nurse is assessing a very thin client who has come to the emergency department with acute abdominal pain. Upon assessment, visible peristaltic movements are noted. What is the appropriate nursing action? A - Prepare to administer antibiotics as prescribed. B - Report finding to the health care provider. C - Monitor laboratory values for possible pancreatitis. D - Toilet quickly as diarrhea is imminent.

C - Monitor laboratory values for possible pancreatitis. The nurse will report the finding to the health care provider, as it is possible that the client has an obstruction. Peristaltic movements are rarely seen except in thin clients. Acute diarrhea does not cause visible peristaltic movements. Pancreatitis is not characterized by visible peristaltic movement. The client likely has an obstruction, not an infection.

E - 55 A young adult man says that he cannot stay on a diet because of trouble finding one that will incorporate his food preferences. How does the nurse most effectively plan nutritional care for this client? A - Calculates his body mass index (BMI). B - Measures his accurate height and weight measurements. C - Obtains a 24-hour recall (diary) of his food intake. D - Records a 24-hour diary of his physical activities.

C - Obtains a 24-hour recall (diary) of his food intake. The most effective way to plan nutritional care for a client is to obtain a 24-hour recall of food intake. This will determine the client's food preferences and eating patterns so that they can be incorporated into the diet.Although calculating a BMI and measuring height and weight are important parts of a nutritional assessment, they do not address the issue of the client's food preferences. Keeping an activity diary will also not reveal any information related to the client's food preferences.

E - 54 A client who had a Whipple surgical procedure develops an internal fistula between the pancreas and stomach. For which complication would the nurse monitor? A - Cirrhosis B - Crohn disease C - Peritonitis D - Peptic ulcer disease

C - Peritonitis Leakage of pancreatic enzymes, bile, and/or gastric secretions into the abdomen (peritoneal cavity) often causes peritonitis, which requires IV antibiotic therapy to manage.

E - 51 The nurse is caring for a client who had an anterior-posterior surgical resection for colorectal cancer this morning. What will the nurse anticipate as the client's priority problem at this time? A - Intestinal obstruction B - Nausea and vomiting C - Severe pain D - Constipation

C - Severe pain The surgical incisions are in the perineal area and are very painful due to the number of nerves in that region of the body. Pain control is the biggest challenge for the nurse and health care team to promote client comfort.

E - 50 The nurse finds a client vomiting coffee-ground emesis. On assessment, the client has a blood pressure of 100/74 mm Hg, is acutely confused, and has a weak and thready pulse. Which intervention is the nurse's first priority? A - Administer anti anxiety medication. B - Initiate enteral nutrition. C - Start intravenous (IV) fluids, D - Administer histamine (H2) antagonist.

C - Start intravenous (IV) fluids, The nurse's first priority is to administer intravenous (IV) fluids. Administering IV fluids is necessary to treat the hypovolemia caused by acute gastrointestinal (GI) bleeding.

A nurse is completing discharge teaching with a client who is 3 days postoperative following a transverse colostomy. Which of the following should the nurse include in the teaching? A - Mucus will be present in stool for 5 to days after surgery B - Expect 500 to 1,000 mL of semi liquid stools after 2 weeks C - Stoma should be moist and pink D - Change the ostomy bag when it is full.

C - Stoma should be moist and pink Rationial A - Music and blood can be present for 2-3 days after surgery B - Output should become stool-like, semi-formed, or formed within days to weeks C - pink and moist stomas is expected D - Change when it is 1/4 to 1/2 full

ATI - P A nurse is providing teaching to a client who has a new prescription for dimenhydrinate to prevent motion sickness. Which of the following instructions should the nurse include? (Select all that apply.) A - Sit upright for 30 min after taking the drug. B - Avoid antacids. C - Take the drug 30 to 60 min before activities that trigger nausea. D - Avoid activities that require alertness. E - Increase fluid and fiber intake.

C - Take the drug 30 to 60 min before activities that trigger nausea. D - Avoid activities that require alertness. E - Increase fluid and fiber intake. Rational A - Sit upright for 30 min after taking the drug is incorrect. Dimenhydrinate, an antihistamine, is unlikely to cause esophagitis, so this precaution is unnecessary. Alendronate, a bisphosphonate that treats osteoporosis, is a drug that requires sitting upright for 30 min after taking it because it can cause esophagitis. B - Avoid antacids is incorrect. Dimenhydrinate does not interact specifically with antacids. Antacids can decrease the absorption of ranitidine, another GI-system drug. C - Take the drug 30 to 60 min before activities that trigger nausea is correct. The nurse should instruct the client to take dimenhydrinate 30 to 60 min before activities that trigger nausea, and again before meals and at bedtime. D - Avoid activities that require alertness is correct. Dimenhydrinate can cause sedation. The nurse should instruct the client to avoid activities that require alertness. E - Increase fluid and fiber intake is correct. Dimenhydrinate can cause anticholinergic effects, such as dry mouth and constipation. The nurse should instruct the client to increase activity level, and fluid and fiber intake.

ATI - P A nurse is providing teaching to a client who is about to start taking psyllium to treat constipation. Which of the following instructions should the nurse include? (Select all that apply.) A - Expect results in 6 to 12 hr. B - Urinate every 4 hr. C - Take the drug with at least 8 oz (237 mL) of fluid. D - Avoid activities that require alertness. E - Increase fluid and fiber intake.

C - Take the drug with at least 8 oz (237 mL) of fluid. E - Increase fluid and fiber intake. Rational A - Expect results in 6 to 12 hr is incorrect. Psyllium, a fiber supplement, typically results in soft, formed bowel movements 1 to 3 days after beginning therapy. B - Urinate every 4 hr is incorrect. Psyllium is unlikely to cause urinary retention. Atropine can cause anticholinergic effects, such as urinary retention, and can require scheduled urination to help prevent this effect. C - Take the drug with at least 8 oz (237 mL) of fluid is correct. To prevent esophageal obstruction, clients should take psyllium with at least 8 oz of fluid; and after mixing the powdered form, clients should drink it immediately. D - Avoid activities that require alertness is incorrect. Psyllium is unlikely to cause sedation or dizziness. Prochlorperazine is a GI-system drug that can cause sedation and requires avoiding activities that require alertness. E - Increase fluid and fiber intake is correct. The nurse should tell the client to increase activity, fluid intake, and fiber intake, and to keep track of bowel function.

E - 54 The nurse is teaching a client with gallbladder disease about diet modification. Which meal would the nurse suggest to the client? A - Sausage and scrambled eggs B - Steak and french fries C - Turkey sandwich on wheat bread D - Fried chicken and mashed potatoes

C - Turkey sandwich on wheat bread Turkey is an appropriate low-fat selection for this client. High fiber, from the wheat bread, also helps reduce the risk. Typically, diets high in fat, high in calories, low in fiber, and high in refined white carbohydrates place clients at higher risk for developing gallstones.Steak, french fries, fried chicken and mashed potatoes, and sausage are too fatty. Eggs are too high in cholesterol for a client with gallbladder disease.

E - 48 A hospitalized client with ongoing abdominal tenderness reports an increase in generalized abdominal pain today. Which assessment technique will the nurse perform? Select all that apply. A - Percuss to determine size of liver and spleen. B - Auscultate beginning in the RLQ. C - Visually observe for contour and symmetry. D - Ask for a pain scale rating on a scale of 0-10. E - Deeply palpate the area of tenderness.

C - Visually observe for contour and symmetry. D - Ask for a pain scale rating on a scale of 0-10. The abdominal assessment is performed in the order of inspection, auscultation, percussion, and palpation. The nurse will visually observe the abdomen for contour and symmetry, auscultate beginning in the RUQ (not the RLQ), lightly palpate for any large masses or areas of tenderness, ask the client to rate the pain level on a 0-10 scale, and document the findings.The nurse will not perform deep palpation nor percussion, as the health care provider conducts this portion of the examination.

ATI 50 A nurse is providing discharge teaching to a client who has a new prescription for aluminum hydroxide. Which of the following information should the nurse include in the teaching A - Take the medication with food B - Monitor for diarrhea C - Wait 1 hr before taking other oral medications D - Maintain a low-fiber diet.

C - Wait 1 hr before taking other oral medications Rational A - Advised to take aluminum hydroxide on an empty stomach B - Aluminum hydroxide can cause constipation C - Do not take oral meds within 1 hr of an antacid D - Increases five du to the constipation effect of the med

E - 55 Based on nutritional screening findings and assessments, which client does the nurse identify that meets criteria for surgical treatment of obesity? A - Woman with a BMI of 38, weight 50% above ideal body weight B - Man with a body mass index (BMI) of 40, weight 75% above ideal body weight C - Woman with a BMI of 42, weight 100% above ideal body weight D - Man with a BMI of 41, weight 80% above ideal body weight

C - Woman with a BMI of 42, weight 100% above ideal body weight The client who will be most successful with surgical intervention is the client with a BMI of 40 or more and a weight 100% above the ideal body weight.The other clients do not have a high enough BMI-to-weight ratio to be considered for surgical intervention.

ATI A nurse is providing discharge teaching for a client who has a new colostomy and is concerned about flatus and odor. Which of the following foods should the nurse recommend to the client? A - Eggs B - Fish C - Yogurt D - Broccoli

C - Yogurt Rational A - The nurse should inform the client that eggs, asparagus, and cabbage can contribute to odor when the colostomy pouch is open. B -The nurse should inform the client that fish can contribute to odor when the colostomy pouch is open. C - The nurse should recommend yogurt, crackers, and toast, which can prevent flatus and stool odor. D - The nurse should inform the client that broccoli, beans, and spicy foods can cause flatus.

E - 54 After receiving change-of-shift report on these clients, which client does the nurse plan to assess first? A - Older adult client who is receiving total parenteral nutrition after a Whipple procedure and has a glucose level of 235 mg/dL (13.1 mmol/L). B - Adult client admitted with cholecystitis who is experiencing severe right upper quadrant abdominal pain. C - Young adult client with acute pancreatitis who is dyspneic and has a respiratory rate of 34 to 38 breaths/min. D - Middle-age client who has an elevated temperature after undergoing endoscopic retrograde cholangiopancreatography.

C - Young adult client with acute pancreatitis who is dyspneic and has a respiratory rate of 34 to 38 breaths/min. The nurse would first assess the young adult client with acute pancreatitis who is dyspneic and has a respiratory rate of 34 to 38 breaths/min. Airway and breathing are the two most important criteria the nurse will use to determine which client to assess first. The dyspneic client is at greatest risk for rapid deterioration and requires immediate assessment and intervention. Acute respiratory distress syndrome is a possible complication of acute pancreatitis.The client with cholecystitis and the client with an elevated temperature will require further assessment and intervention, but these are not medical emergencies requiring the nurse's immediate attention. The older adult client's glucose level will require intervention but, again, is not a medical emergency.

E - 53 The nurse is caring for a client who has cirrhosis of the liver. The client's latest laboratory testing shows a prolonged prothrombin time. For what assessment finding would the nurse monitor: A - deep vein thrombosis. B - jaundice. C - hematemesis. D - pressure injury.

C - hematemesis. The client who has cirrhosis is at risk for bleeding due to decreased production of prothrombin by the liver. Portal hypertension that occurs in clients with cirrhosis causes esophageal blood veins to become fragile, distended, and tortuous. Therefore, these veins tend to bleed as evidenced by either hematemesis or melena.

Mastery - 51-2 A client has a new diagnosis of irritable bowel syndrome (IBS) with diarrhea. What health teaching by the nurse is appropriate for this client? A. "Take a stool softener every day to ease defecation." B. "Avoid high-fiber foods in your diet." C. "Avoid dairy products and caffeinated beverages." D. "Ask your primary health care provider for an antidepressant."

C. "Avoid dairy products and caffeinated beverages." Rationales: The client who presents with diarrhea as a result of IBS needs fiber to help provide bulk and does not need a stool softener as clients with constipation do. Therefore, Choices A and B are incorrect responses. The nurse should not suggest any medication to manage possible anxiety or depression as a result of IBS because not all clients with IBS have those mental health problems (Choice D). However, dairy products are often not well tolerated and should be avoided to help decrease diarrhea (Choice C).

51-3 A nurse working provides discharge teaching for a male client who had a minimally invasive hernia repair this morning. Which statement by the client indicates a need for further teaching? A. "I should avoid coughing if at all possible." B. "I can shower in day or two after I remove my surgical bandage." C. "I can't go back to work for at least 6 weeks." D. "I should use an ice pack to help relieve my pain."

C. "I can't go back to work for at least 6 weeks." Rationales: Minimally invasive surgery (MIS) allows the client to be discharged on the same day as surgery and return to work in 1-2 weeks rather than wait the usual 6 weeks after a conventional open surgical procedure. Therefore, Choice C is the correct answer indicating a need for additional health teaching. The other choices (A, B, and D) are all appropriate for an MIS surgical approach.

Mastery 48 - 1 Immediately following a colonscopy, which client behavior will the nurse report to the health care provider? Select all that apply. A. Passing of flatus B. Blood pressure 128/80 C. Abdominal guarding D. Change in mental status E. Report of mild abdominal cramping

C. Abdominal guarding D. Change in mental status Rational The nurse will report abdominal guarding, as this can be a sign of bowel perforation. The nurse will also report any changes in mental status, as this can be a sign of hypovolemic shock. In older adults, this is often the first sign. A blood pressure of 128/80, and reports of mild abdominal cramping, are considered normal findings that do not require the nurse to notify the healthcare provider.

54-1 A young adult client admitted with a diagnosis of cholecystitis from cholelithiasis has severe abdominal pain, nausea, and vomiting. Based on these assessment findings, which client problem is the highest priority for nursing intervention at this time? A. Anxiety B. Risk for dehydration C. Acute pain D. Malnutrition

C. Acute pain Rationale: The client's assessment does not indicate the presence of anxiety or malnutrition. Therefore, Choices A and D are incorrect responses. The client is at a risk for dehydration due to vomiting, but the actual client problem of acute pain (Choice C) is a higher priority than a potential problem that does not exist at the present time for a young adult (Choice B).

Mastery 52-2 A client had a colectomy with creation of an ileo-anal pouch and temporary ileostomy yesterday morning. The nurse assesses the ostomy and its functioning. Which assessment finding will the nurse report to the primary health care provider? A. Client's report of abdominal pain of 3 on a 0-10 pain intensity scale B. Slight abdominal distention C. No drainage from the ileostomy D. Serosanguinous effluent from the drain

C. No drainage from the ileostomy Rationales: The nurse would expect a report of pain at a 3 (Choice A), slight abdominal distention (Choice B), and serosanguinous drainage (Choice D). Therefore, these findings do not need to be reported. However, the ileostomy should drain within 24 hours after surgery. Therefore, Choice C needs to be reported to the primary health care provider.

48-2 While performing an abdominal assessment on a client, the nurse notes a bruit over the aorta. What is the appropriate nursing action? A. Consult another nurse to verify the bruit B. Auscultate each quadrant for 5 minutes each C. Notify the health care provider of the findings D. Perform light palpation to further assess the pulsation

C. Notify the health care provider of the findings Rational A bruit (a "swooshing" sounds) over the abdominal aorta usually indicates the presence of an aneurysm. If this sound is heard, the nurse should stop the assessment, and refrain from percussing or palpating the abdomen. It is not necessary to consult another nurse to verify the findings. It is of critical importance to notify the healthcare provider immediately of the findings.

Mastery 53-2 The nurse is caring for a patient with cirrhosis who has hepatic encephalopathy. Which assessment finding should the nurse report to the primary health care provider? A. Fatigue B. Difficulty sleeping C. Seizure D. Disorientation

C. Seizure Rationale: Choices A, B, and D are associated with early hepatic encephalopathy. However, seizures occur later and indicate that the complication is progressing. Therefore, Choice C is the most concerning and should be reported to the primary health care provider.

E - 52 The nurse is teaching a client about caring for a new ileostomy. What information is most important to include? A - "After surgery, output from your ileostomy may be a loose, dark-green liquid with some blood present." B - "Remember that you must wear a pouch system at all times." C - "Notify the primary health care provider if output from your stoma has a sweetish odor." D - "Call your primary health care provider if your stoma has a bluish or pale look."

D - "Call your primary health care provider if your stoma has a bluish or pale look." It is most important for the nurse to tell the client with a new ileostomy to call the primary health care provider if the stoma has a bluish or pale look. If the stoma has a bluish, pale, or dark look, its blood supply may be compromised and the primary health care provider must be notified immediately.

E - 52 A client who developed viral gastroenteritis with vomiting and diarrhea is scheduled to be seen in the clinic the following day. What intervention would the nurse recommend for the client to do? A - "Avoid all solid foods to allow complete bowel rest." B - "Take an over-the-counter antidiarrheal medication." C - "Contact your primary health care provider for an antibiotic medication." D - "Consume extra fluids to replace fluid losses."

D - "Consume extra fluids to replace fluid losses." The nurse tells the client to drink extra fluids to replace fluid lost through vomiting and diarrhea.It is not necessary to stop all solid food intake. Antidiarrheal medications are used if diarrhea is severe. Antibiotics are used if the infection is bacterial.

E - 51 The home health nurse is teaching a client about the care of a new colostomy. Which statement by the client demonstrates a correct understanding of the health teaching? A - "If the skin around the stoma is red or scratched, it will heal soon." B - "I need to strive for a very tight fit when applying the barrier around the stoma." C - "A dark or purplish-looking stoma is normal and would not concern me." D - "I need to check for leakage underneath my colostomy."

D - "I need to check for leakage underneath my colostomy." The client's statement, "I need to check for leakage underneath my colostomy" shows that the patient correctly understands the instructions about how to care for a new colostomy. The pouch system must be checked frequently for evidence of leakage to prevent excoriation.A purplish stoma is indicative of ischemia and necrosis. Redness or scratched skin around the stoma must be reported to prevent it from beginning to break down. An overly tight fit may lead to necrosis of the stoma.

E - 52 The nurse is preparing to provide health teaching for a client who is starting sulfasalazine. Which statement by the client indicates a need for further teaching? A - "I'll let my primary health care provider know if the drug upsets my stomach." B - "I will be sure to take a folic acid supplement while on this drug." C - "I will follow up with getting labs done to check my blood counts." D - "This drug can make me dehydrated because I'm already on a diuretic."

D - "This drug can make me dehydrated because I'm already on a diuretic." Sulfasalazine can cause nausea and vomiting, and can interfere with folic acid absorption. In high doses, it can also cause anemia and agranulocytosis, so blood work would be important for ongoing monitoring. However, the drug does not cause dehydration.

ATI 46 A nurse in a clinic is instructing a client about a fecal occult blood test, which requires mailing three specimens. Which of the following statements by the client indicates understanding? A - "I will continue taking my warfarin while i complete these tests." B - "I'm glad I don't have to follow any special diet at this time." C - "This test determines if I have parasites in my bowel." D - "This is an easy way to screen for colon cancer."

D - "This is an easy way to screen for colon cancer." Rational A - Stop taking anticoagulants prior to obtaining stool specimens for fecal occult blood testing because they can interfere with the results. B - Avoid consuming red, meat, chicken, and fish prior to obtaining stool specimens for fecal occult blood testing because this can interfere with the results. C - Fecal occult blood testing does not id parasites im stool

E - 51 The nurse is providing teaching on ways to promote bowel health and disease prevention. Which statement will the nurse include in this teaching? A - "You should start colorectal cancer screening when you are over 70 years of age." B - "You only need to have regular colonoscopies if there is colorectal cancer in your family.' C - "If you perform fecal occult blood tests every 5 years, you don't need a colonoscopy." D - "You should have a colonoscopy every 10 years starting at 45 years of age

D - "You should have a colonoscopy every 10 years starting at 45 years of age The American Cancer Society recommends that for individuals of average risk for colorectal cancer (CRC), a colonoscopy every 10 years or flexible sigmoidoscopy every 5 years is adequate. The screening should begin for adults of 45 years of age or older unless individuals are at high risk for CRC.

ATI A nurse is providing dietary teaching for a client who has chronic pancreatitis. Which of the following food selections by the client indicates an understanding of the teaching? A - 8 oz (0.24 L) whole milk B - One slice of beef bologna C - 1 oz (28.3 g) cheddar D - 1 cup (0.24 L) sliced banana

D - 1 cup (0.24 L) sliced banana Rational A - The nurse should inform the client that high-fat foods, such as whole milk, should be avoided because foods that are high in fat can cause diarrhea. A cup of whole milk contains 7.93 g of fat. The nurse should recommend fat-free milk. B - The nurse should inform the client that high-fat foods, such as bologna, should be avoided because foods that are high in fat can cause diarrhea. One slice of beef bologna contains 7.84 g of fat. C - The nurse should inform the client that high-fat foods, such as cheddar cheese, should be avoided because foods that are high in fat can cause diarrhea. An ounce of cheddar cheese contains 9.33 g of total fat. D - Foods that are high in fat can cause diarrhea for clients who have pancreatitis. One cup of sliced banana, which contains 0.49 g of fat, is a low-fat food option. Clients who have pancreatitis should consume a high-protein and low-fat diet with an adequate amount of carbohydrates and calories.

E - 48 Which client does the charge nurse assign to an experienced LPN/LVN working on the adult medical unit? A - A 40 year old who needs administration of IV midazolam hydrochloride during an upper endoscopy. B - A 36 year old who needs teaching about an endoscopic retrograde cholangiopancreatography. C - A 46 year old who is admitted with abdominal cramping and diarrhea of unknown causes. D - A 32 year old with constipation who has received a laxative.

D - A 32 year old with constipation who has received a laxative. The LPN/LVN can best assist the RN by monitoring the client with constipation who has received a laxative.Assessment, IV hypnotic medication administration, and client teaching must be done by an RN.

ATI - P Which of the following drugs has protocols that require clients to meet specific risk-management criteria and sign a treatment agreement before the nurse can administer the drug? A - Lubiprostone B - Azathioprine C - Sulfasalazine D - Alosetron

D - Alosetron Rational A - Although lubiprostone, a serotonin 5-HT4 receptor agonist, can cause potentially serious adverse effects, including nausea, vomiting, diarrhea, and headache, clients do not have to sign anything other than the standard consent forms prior to taking the drug. B - Although azathioprine, an immunosuppressant, can cause potentially serious adverse effects, such as bone marrow suppression, clients do not have to sign anything other than the standard consent forms prior to taking the drug. C - Although sulfasalazine, a 5-aminosalicylate, can cause potentially serious adverse effects, including hematologic disorders, clients do not have to sign anything other than the standard consent forms prior to taking the drug. D - Clients who take alosetron, a serotonin 5-HT3 receptor antagonist, can develop severe constipation that can lead to impaction, bowel obstruction, perforation, and potentially fatal ischemic colitis. Because of these risks, nurses must inform clients of the benefits and risks of the drug therapy, and clients must sign a treatment agreement.

ATI A nurse is reviewing the laboratory results of a client who has hepatic cirrhosis. Which of the following laboratory findings should the nurse report to the provider? A - Albumin 4.0 g/dL B - INR 1.0 C - Direct bilirubin 0.5 mg/dL D - Ammonia 180 mcg/dL

D - Ammonia 180 mcg/dL Rational A - An albumin level of 4.0 g/dL is within the expected reference range of 3.5 to 5 g/dL. B - An INR of 1.0 is within the expected reference range of 0.8 to 1.1 C - A direct bilirubin level of 0.5 mg/dL is within the expected reference range of 0.3 to 1.0 mg/dL. D - An ammonia level of 180 mcg/dL is above the expected reference range of 10 to 80 mcg/dL. The nurse should report an increased ammonia level because it can indicate portal-systemic encephalopathy.

ATI - P A nurse is caring for a client who has a new prescription for famotidine to treat GERD. The nurse should instruct the client to wait at least 1 hr between taking famotidine and which of the following over-the-counter drugs? A - Ginkgo biloba B - Antidiarrheals C - St. John's wort D - Antacids

D - Antacids Rational A - Ginkgo biloba is a natural or herbal medication that is classified as an antiplatlet or central nervous system stimulant. There is no interaction with famotidine, a histamine2-receptor antagonist. B - Antidiarrheal preparations do not specifically interact with famotidine, a histamine2-receptor antagonist. However, bismuth subsalicylate, which is used to treat diarrhea, can decrease the absorption of tetracycline and quinolones. C - St. John's wort is a natural or herbal medication that is classified as an antidepressant. It does not specifically interact with famotidine, a histamine2-receptor antagonist. D - Antacids can decrease the absorption of famotidine, a histamine2-receptor antagonist. The nurse should instruct the client to wait at least 1 hr between taking famotidine and taking an antacid.

E - 53 When preparing a client to undergo paracentesis, which action is necessary to reduce potential injury as a result of the procedure? A - Assist the provider to insert a trocar catheter into the abdomen. B - Position the client with the head of the bed flat. C - Encourage the client to take deep breaths and cough. D - Ask the client to void prior to the procedure.

D - Ask the client to void prior to the procedure. To avoid injury to the bladder during a paracentesis, the client would be asked to void prior to the procedure. Taking deep breaths and coughing does not prevent complications or injury as a result of paracentesis. Clients would be positioned with the head of the bed elevated. The trocar catheter is used to drain the ascetic fluid and does not reduce the risk of damage to the bladder.

E - 55 An older adult client is at risk for undernutrition. Which nursing intervention is appropriate to ensure optimum nutritional intake? A - Administering antiemetics and analgesics after meals B - Reminding APs to allow the client to remain in bed during meals C - Turning on the television during meals to provide distraction D - Assisting the client with toileting and oral care prior to meals

D - Assisting the client with toileting and oral care prior to meals The appropriate intervention to ensure optimum nutritional intake in an older adult client at risk for undernutrition is to assist the client with toileting and oral care prior to meals for comfort and to prevent these from distracting clients from meals.Antiemetics and analgesics should be provided prior to meals. Clients need to be free from distractions while eating. When possible, clients are placed in chairs for eating.

ATI A nurse is caring for a client who has GERD and a new prescription for metoclopramide. The nurse should plan to monitor for which of the following adverse effects? A - Thrombocytopenia B - Hearing loss C - Hypersalivation D - Ataxia

D - Ataxia Rational A - An adverse effect of low-molecular-weight heparins, such as enoxaparin, is thrombocytopenia. However, thrombocytopenia is not an adverse effect of metoclopramide. B - Gentamicin and many other medications are ototoxic. However, metoclopramide does not cause hearing loss. C - An adverse effect of metoclopramide is xerostomia (dry mouth), not hypersalivation. D - The nurse should plan to monitor the client for extrapyramidal symptoms, such as ataxia, and should report any of these findings to the provider.

E - 53 It is essential that the nurse monitor the client returning from hepatic artery embolization for hepatic cancer for which potential complication? A - Right shoulder pain B - Bone marrow suppression C - Polyuria D - Bleeding

D - Bleeding A potential complication of hepatic artery embolization for hepatic cancer is bleeding. Prompt detection of hemorrhage is the priority.Discomfort such as right shoulder pain may be present, but the priority is to assess for hemorrhage. The nurse must assess for signs of shock, not polyuria. Embolization does not suppress the bone marrow. If chemotherapy or immune modulators is used, the nurse then assesses for bone marrow suppression.

E - 50 The nurse is monitoring a client with gastric cancer for signs and symptoms of upper gastrointestinal bleeding. Which change in vital signs would the nurse expect? A - Temperature from 97.9° to 98.9° F (36.6° to 37.2° C) B - Respiratory rate from 24 to 20 breaths/min C - Apical pulse from 80 to 72 beats/min D - Blood pressure from 140/90 to 110/70 mm Hg

D - Blood pressure from 140/90 to 110/70 mm Hg A decrease in blood pressure from 140/90 to 110/70 indicates that the client has hypovolemia from loss of body fluid (in this case, blood).

E - 48 Which client does the charge nurse on the adult medical unit assign to an RN who has floated from the outpatient gastrointestinal (GI) clinic? A - Client admitted with nausea, abdominal pain, and abdominal distention. B - Client who needs discharge teaching after an endoscopic retrograde cholangiopancreatography (ERCP). C - Client with epigastric pain who needs conscious sedation during endoscopy. D - Client who has had laxatives administered and needs monitoring before a colonoscopy.

D - Client who has had laxatives administered and needs monitoring before a colonoscopy. The client who needs laxatives administered and effectiveness monitored before a colonoscopy is the least complicated client. This client would be assigned to the float nurse who would have the experience and education to adequately care for this client.Discharge instructions following an ERCP, assessment of an admitted acutely ill client, and monitoring a client who is receiving conscious sedation is accomplished best by a nurse with experience in caring for adults with acute GI problems.

E - 53 The nurse is caring for a client who was recently diagnosed with Laennec cirrhosis. What is the nurse's priority assessment during client care? A - Cardiovascular assessment B - Abdominal assessment, including bowel sounds C - Respiratory assessment D - Cognitive and neurologic assessment

D - Cognitive and neurologic assessment The type of cirrhosis that this client has is caused by alcoholism. Withdrawal from alcohol can cause cognitive and neurologic changes, such as confusion and delirium tremens (DTs).

ATI A nurse is assessing a client immediately following a paracentesis for the treatment of ascites. Which of the following findings indicates the procedure was effective? A - Presence of a fluid wave B - Increased heart rate C - Equal pre- and postprocedure weights D - Decreased shortness of breath

D - Decreased shortness of breath Rational A - The nurse should identify the presence of a fluid wave as an indication of ascites. This finding does not indicate the procedure was effective. B - The nurse should identify an increased heart rate as an indication of hypovolemia, which is a potential complication of a paracentesis. This finding does not indicate the procedure was effective. C - The nurse should expect the client's postprocedure weight to be less than the preprocedure weight due to the withdrawal of fluid from the peritoneal cavity. This finding does not indicate the procedure was effective. D - Increased abdominal fluid can limit the expansion of the diaphragm and prevent the client from taking a deep breath. After excess peritoneal fluid is removed, the diaphragm will expand more freely. The nurse should identify this finding as an indicator the procedure was effective.

ATI 29 A nurse is caring for a client who received prochlorperazine 4 hours ago. The client reports spasms of the face. The nurse should expect a prescription for which of the following medications? A - Fomepizole B - Naloxone C - Phytonadione D - Diphenhydramine

D - Diphenhydramine Rational A - Fomepizole is an antidote to treat ethylene glycol poisoning B - Naloxone is used to treat opioid toxicity C - Phytonadione - Vitamin K1 is used to treat warfarin toxicity D - An adverse effect of prochlorperazine is acute dystonia, which is evidence by spasms of the muscles in the face, neck and tongue. Diphenhydramine is used to suppress extrapyramidal effects of prochlorperazine.

ATI 54 A nurse is completing an admission assessment of a client who has pancreatitis. Which of the following findings should the nurse expect? A - Pain in right upper quadrant radiating to right shoulder B - Report of pain being worse than sitting upright C - Pain relieved with defecation D - Epigastric pain radiating to the left shoulder.

D - Epigastric pain radiating to the left shoulder. Rational A - A person who has cholecystitis will report RUQ radiating to the right shoulder B - Pain will be worse lying down C - relieved by assuming the fetal position D - will report severe, boring epigastric pain that radiates to back, left flank or left shoulder

E - 48 A client is admitted to the hospital with severe right upper quadrant (RUQ) abdominal pain. Which assessment technique does the nurse use for this client? A - Has the client lie in a supine position with legs straight and arms above the head. B - Assesses the following sequence: inspection, palpation, percussion, auscultation. C - Palpates any bulging mass very gently and documents findings. D - Examines the RUQ of the abdomen last following all other assessment techniques.

D - Examines the RUQ of the abdomen last following all other assessment techniques. If the client reports pain in the RUQ, the nurse examines this area last. This sequence prevents the client from tensing abdominal muscles because of the pain, which would make the assessment difficult.The sequence for examining the abdomen is inspection, auscultation, percussion, and then palpation. This sequence prevents the increase in intestinal activity and bowel sounds caused by palpation and percussion. The client would be positioned supine with the knees bent while keeping the arms at the sides to prevent tensing of the abdominal muscles. If a bulging, pulsating mass is present during assessment of the abdomen, the nurse must never touch the area because the client may have an abdominal aortic aneurysm, a life-threatening problem. The nurse would notify the health care provider of this finding immediately!

E - 50 The nurse is recovering a client who had an esophagogastroduodenoscopy (EGD). What assessment would the nurse perform before determining if the client can have fluids to drink? A - Bowel sounds B - Orientation C - Presence of bruit D - Gag reflex

D - Gag reflex The nurse would check for the return of the client's gag reflex before allowing the client to drink or eat to prevent aspiration.

ATI A nurse is reviewing the laboratory values of a client who has colorectal cancer. Which of the following findings should the nurse expect? A - Negative fecal occult blood test B - Decreased serum carcinoembryonic antigen (CEA) level C - Hematocrit 43% D - Hemoglobin 9.1 g/dL

D - Hemoglobin 9.1 g/dL Rational A - A positive fetal occult blood test is an expected finding for a client who has colorectal cancer because colorectal cancer causes bleeding in the gastrointestinal tract. B - An elevated CEA level is an expected finding in a client who has colorectal cancer. C - A hematocrit level of 43% is within the expected reference range. The nurse should expect a decreased hematocrit level for a client who has colorectal cancer due to occult intestinal bleeding. D - A hemoglobin level of 9.1 g/dL is below the expected reference range. Decreased hemoglobin is an expected finding in a client who has colorectal cancer due to occult intestinal bleeding.

ATI 49 A nurse is teaching a client who has a new diagnosis of dumping syndrome following gastric surgery. Which of the following informations should the nurse include in the teaching? A -Eat three moderate size meals a day B - Drink at least one glass of water with each meal C - Eat a bedtime snack that contains a milk product D - Increase protein in the diet

D - Increase protein in the diet Rational A - Consume small, frequent meals B - limited liquids with meals and for 1 hr prior to and following meals C - Avoid milk products D - high proteini, high fat, low fiber, and moderate to low carbs

ATI 54 A nurse is reviewing the admission laboratory results of a client who has acute pancreatitis. Which of the following findings should thenurse expect? A - Decreased blood lipase level B - Decreased blood amylase level C - Increase blood calcium level D - Increased blood glucose level

D - Increased blood glucose level Rational A - experience an elevated blood lipase level due to pancreatic cell injury B - Experience an elevsat4d blood amylase due to pancreatic cell injury C - Decreased calcium level due to fat necrosis D - Increased blood glucose due to pancreatic cell injury, which results in impaired metabolism of carbs due to a decrease in the release of insulin

E - 53 The nurse is caring for a client who had a liver transplant last week. For which complication will the nurse teach the client and family to monitor? A - Acute kidney injury B - Hypertension C - Pulmonary edema D - Infection

D - Infection The client is at the most risk for rejection of the transplant which can be the result of an infection if not identified and managed effectively. Therefore, the nurse would teach the client and family to report cough, fever, skin redness, and other signs of infection.

E- 51 A client with an intestinal obstruction has pain that changes from a "colicky" intermittent type to constant discomfort. After a complete assessment, what action would the nurse plan implement at this time? A - Change the nasogastric suction level from "intermittent" to "continuous." B - Administer medication for pain based on the client's pain level. C - Position the client in a semi- or high-Fowler position. D - Prepare the client for emergency surgery in collaboration with the health team.

D - Prepare the client for emergency surgery in collaboration with the health team. The appropriate nursing action for a client with intestinal obstruction whose pain changes from "colicky" intermittent type to constant discomfort is to prepare surgery because this change is most likely indicative of perforation or peritonitis and will require immediate surgical intervention.Pain medication may mask the client's symptoms but will not address the root cause. A change in the nasogastric suction rate will not resolve the cause of the client's pain and could be particularly ineffective if a nonvented tube is in use.

ATI - P A nurse is teaching a client who recently had a myocardial infarction and has a new prescription for docusate sodium. The nurse should inform the client that docusate sodium has which of the following therapeutic effects? A - Reduces inflammation B - Reduces gastric acid C - Prevents diarrhea D - Prevents straining

D - Prevents straining Rational A - Docusate sodium does not have anti-inflammatory effects. Sulfasalazine, a 5-aminosalicylate, is a GI-system drug that reduces the inflammation of inflammatory bowel disease. B - Docusate sodium does not reduce gastric acid secretion. Omeprazole, a proton pump inhibitor, is a GI-system drug that reduces gastric acid secretion. C - Docusate sodium does not prevent diarrhea. Loperamide, an opioid agonist, is a GI-system drug that prevents or treats diarrhea. D - Docusate sodium, a stool softener, prevents straining during defecation and prevents the elevation in blood pressure that can result from straining. It also helps relieve constipation and reduces the painful elimination of hard stools.

E-54 A client has been placed on enzyme replacement for treatment of chronic pancreatitis. In teaching the client about this therapy, the nurse advises the client not to mix enzyme preparations with foods containing which element? A - High fat B - High fiber C - Carbohydrates D - Protein

D - Protein The nurse tells the client not to mix enzyme preparations with foods containing protein because the enzymes will dissolve the food into a watery substance. Pancreatic-enzyme replacement therapy (PERT) is the standard of care to prevent malnutrition, malabsorption, and excessive weight loss. Pancrelipase is usually prescribed in capsule or tablet form and contains varying amounts of amylase, lipase, and protease.No evidence suggests that enzyme preparations should not be mixed with carbohydrates, food with highfat content, and food with high-fiber content.

ATI 55 A nurse on a medical-surgical unit is admitting a client who has hepatitis B with ascites. Which of the following actions should the nurse include in the plan of care? A - Initiate contact precautions B - Weight the client weekly C - Measure abdominal girth at the base of the ribcage D - Provide a high-calorie, high-carbohydrate diet.

D - Provide a high-calorie, high-carbohydrate diet. Rational A - Hep B is transmitted via blood B - Daily weights C - Abdominal girth is measure over the largest part of the abdomen which will vary by client D - A pt with Hep B should have a diet high in calories and carbs

E - 55 An older adult client needs additional dietary protein, but refuses to drink the prescribed liquid protein supplements. Which nursing intervention helps the client to increase protein intake? A - Keeping a food and fluid intake diary for at least 3 days B - Administering the liquid supplement with routine medications C - Giving a glucose polymer modular supplement D - Providing protein modular supplements in the form of puddings

D - Providing protein modular supplements in the form of puddings The appropriate intervention to increase the client's protein intake is to provide protein modular supplements in the form of puddings. This would increase the client's protein intake in a format other than a liquid supplement.Administering the liquid supplement with routine medications will not be effective because the client has already refused to drink the supplements. Glucose polymer modular supplements will increase the client's calorie intake but not protein intake. A food and fluid diary will provide information about the client's typical intake pattern, but will not increase protein intake.

E - 53 How would the home care nurse best modify the client's home environment to manage side effects of lactulose? A - Obtains a walker for the client. B - Rearranges furniture to declutter the home. C - Removes throw rugs to prevent falls. D - Requests a bedside commode for the client.

D - Requests a bedside commode for the client. The home care nurse would modify the client's home environment to manage side effects of lactulose by making a bedside commode available to the client. Lactulose therapy increases the frequency of stools. A bedside commode is especially necessary if the client has difficulty reaching the toilet.

E - 53 The nurse collaborates with the registered dietitian nutritionist in providing teaching for a client who has ascites from cirrhosis. What daily dietary restriction would the nurse include in the health teaching? A - Calcium B - Potassium C - Magnesium D - Sodium

D - Sodium Mild to moderate sodium restriction is often tried as the first intervention to decrease body fluid retention, including ascites.

ATI - P A nurse is caring for a client who is taking phenytoin for a seizure disorder and has a new prescription for sucralfate to treat a duodenal ulcer. The nurse should instruct the client to take the drugs at least 2 hr apart for which of the following reasons? A - Phenytoin increases the metabolism of sucralfate. B - Phenytoin reduces the effectiveness of sucralfate. C - Sucralfate increases the risk for phenytoin toxicity. D - Sucralfate interferes with the absorption of phenytoin.

D - Sucralfate interferes with the absorption of phenytoin. Rational A - Phenytoin increases the metabolism of oral contraceptives and corticosteroids. It is unlikely to increase the metabolism of sucralfate. B - Phenytoin increases the metabolism of oral anticoagulants, reducing their effectiveness. It is unlikely to reduce the effectiveness of sucralfate. C - Omeprazole, another GI-system drug, can increase phenytoin levels and increase the risk for phenytoin toxicity. Sucralfate is unlikely to increase the risk for phenytoin toxicity. D - Sucralfate decreases the absorption of phenytoin. The nurse should instruct the client to allow at least 2 hr between taking the two drugs and should monitor the client's phenytoin levels.

ATI 47 A nurse is caring for a client following a paracentesis. Which of the following findings indicated the bowel was perforated during the procedure? A - Client report of upper chest pain B - Decreased urine output C - Pallor D - Temperature elevation

D - Temperature elevation Rational A - Report of sharp, constant abdominal pain is associated with bowel perforation B - Decreased urine output is associated with bladder perforation during a paracentesis C - Pallor may indicate hypovolemia related to fluid removal of ascites fluid during the procedure D - Fever is an indication of bowel perforation

ATI A nurse is assessing a client who has a duodenal ulcer. Which of the following findings should the nurse expect? A - The client states that the pain is in the upper epigastrium. B - The client is malnourished. C - The client states that ingesting food intensifies the pain. D - The client reports that pain occurs during the night.

D - The client reports that pain occurs during the night. Rational A- Pain associated with a duodenal ulcer is typically located below or to the right of the epigastrium. B - Typically, a client who has a duodenal ulcer is not malnourished. C - Typically, a client who has a duodenal ulcer reports that ingesting food diminishes their pain level. D - Pain associated with a duodenal ulcer occurs when the stomach is empty, which is typically 1.5 to 3 hr after meals and during the night.

ATI 53 A nurse is reviewing a new prescription for chenodiol with a client who has choleliithiasis. Which of the following information should the nurse include in the teaching? A - This medication is used to decrease acute biliary pain B - This medication requires thyroid function monitoring every 6 months C - This medication is not recommended for client who have diabetes mellitus D - This medication dissolves gallstones gradually over a period of up to 2 years.

D - This medication dissolves gallstones gradually over a period of up to 2 years. Rational A - Opioid are preferred for acute biliary pain B - should have an ultrasounds of the gallbladder every 6 months during first year of treatment C -Chenodiol is used cautiously in client who have hepatic conditions D - This med can be taken for up to 2 years

ATI 48 A nurse is admitting a client who has bleeding esophageal varices. The nurse should expect a prescription for which of the following medications? A - Propranolol B - Metoclopramide C - Famotidine D - Vasopressin

D - Vasopressin Rational A -Propranolol is not used for actively bleeding. It can be given prophylactically to decrease portal hypertension B - Metoclopramide decreased motility of the esophagus and stomach C - Famotidine - HIstamine-receptor antagonists are administered following surgical procedures for bleeding esophageal varices D - Vasopressin constricts blood vessels and is used to treat bleeding esophageal varices.

E - 52 A client is admitted to the hospital with right lower quadrant abdominal pain, nausea, and vomiting. What assessment would the nurse monitor to identify a potentially life-threatening complication based on the client's condition? A - Intake and output B - Electrolyte values C - Abdominal assessment D - Vital signs

D - Vital signs The client most likely has appendicitis which can result in perforation of the appendix and peritonitis. If this complication occurs, the client would develop tachycardia and a fever. Therefore, the nurse would monitor for changes in vital signs.

Mastery - 54-1 Which statement by the client who is prescribed to take pancreatic enzyme replacements indicates a need for further teaching by the nurse? A. "I need to take the enzymes at every meal and with snacks." B. "After taking the enzymes, I should drink a glass of water." C. "I should wipe my mouth in case any of the enzyme got on my lips." D. "I should chew each capsule carefully so that it works in my stomach."

D. "I should chew each capsule carefully so that it works in my stomach."

49-6 The nurse is caring for a client with esophageal cancer who is scheduled for surgery. When the client asks, "Is this treatment going to cure me?", which is the appropriate nursing response? Select all that apply. A. "The surgery has been useful for many patients so it should work for you." B. "You can beat this disease if you just put your mind to it and do not give up." C. "Yes, and you have the best surgeon around who specializes in cancer treatment." D. "Your surgeon can give more information about the effectiveness of this treatment." E. "It sounds like you are concerned about surgical outcomes; let's talk about your feelings."

D. "Your surgeon can give more information about the effectiveness of this treatment." E. "It sounds like you are concerned about surgical outcomes; let's talk about your feelings." Rational The nurse will respond by addressing the client's question, as well as the possible fear that accompanies it. The surgeon obtains informed consent and will share information about the evidence-based outcomes of the surgery (including whether a surgery is curative or palliative in nature); the nurse can witness the informed consent. The nurse will address the client's emotions by verbalizing the implied (stating that it sounds like the client is concerned out the outcome) and asking if the client would like to talk. The nurse should not indicate that the surgery should work for the client since it worked for others, nor should the nurse give false hope by telling the client that they can beat the disease if they will themselves to do it. Although the client may be treated by a surgeon with a favorable reputation, it is not appropriate to answer "yes", because this implies that the surgery will be curative in nature.

53-3 The nurse is caring for a client diagnosed with hepatitis A. Which transmission-based precautions are required when providing care for this client? Select all that apply. A. Place client in a private room. B. Wear a mask when handling patient bedpan. C. Wear gloves when touching the client. D. Wear a gown when providing personal care to this patient. E. Wear eye goggles when providing care.

D. Wear a gown when providing personal care to this patient. Rationale: Hepatitis A is transmitted via the fecal-oral route. Therefore, the nurse needs to use gloves and a gown when providing care to the client and having contact with the client's stool Choice D). Gloves are not needed to touch the client unless the nurse is cleaning after the client has a stool (Choice C). A mask and googles are not needed because hepatitis of any type is not transmitted as an airborne or droplet disease (Choice B and E). Therefore, a private room is not necessary (Choice A).

E - 49 The nurse is caring for a client who has been diagnosed with esophageal cancer. The client appears anxious, and asks the nurse, "Does this mean I am going to die?". Which nursing response is appropriate? Select all that apply. a - "Let me sit with you for awhile and we can discuss how you are feeling about this." b - "You can beat this disease if you just put your mind to it." c - "No, surgery can cure you." d - "It sounds like death frightens you." e - "Let me call the hospital chaplain to talk with you."

a - "Let me sit with you for awhile and we can discuss how you are feeling about this." d - "It sounds like death frightens you." Acknowledging that death may frighten the client, and offering to talk about how the client is feeling, are therapeutic nursing interventions.Telling the client that surgery is curative, and promising the client can beat the disease, are non therapeutic responses that provide false hope. Although talking with the chaplain at a later time may be requested by the client, the immediate need is to allow the client to express feelings to the nurse.

E - 49 While undergoing radiation treatment for oral cancer, a client develops xerostomia. What collaborative resource does the nurse suggest for this client's care? a - Dentist b - Occupational therapist c - Speech therapist d - Psychiatrist

a - Dentist Xerostomia is the subjective feeling of oral dryness. It is a long-term effect of radiation therapy and requires ongoing oral care such as the use of saliva substitutes and follow-up dental visits.Occupational therapists, psychiatrists, and speech therapists are not the appropriate resource for a client with xerostomia.

E - 50 The nurse is caring for a client who reports stomach pain and heartburn. Which assessment finding is most significant suggesting the client's ulcer is duodenal and not gastric? a - Pain occurs 1½ to 3 hours after a meal, usually at night. b - The client is a man older than 50 years. c - Pain is worsened by the ingestion of food. d - The client has a malnourished appearance.

a - Pain occurs 1½ to 3 hours after a meal, usually at night. A key symptom of duodenal ulcers is that pain usually awakens the client between 1:00 a.m. and 2:00 a.m. (0100 and 0200) and occurs 1½ to 3 hours after a meal.Pain that is worsened with ingestion of food and a malnourished appearance are key features of gastric ulcers. A male over 50 years is a finding that could apply to either type of ulcer.

E - 49 Which practice does the nurse include when teaching a client about proper oral hygiene? a - Perform self-examination of the mouth every week, and report any unusual findings. b - Brush the teeth daily and floss as needed. c - Wear dentures that fit a bit loosely for movement when chewing. d - Use mouthwash with alcohol unless lesions are present.

a - Perform self-examination of the mouth every week, and report any unusual findings. The nurse will teach the client that proper oral care involves self-examination of the mouth every week and to report any unusual findings to the Health Care Provider.Clients need to brush teeth and floss every day—not just as needed. Clients are taught to avoid contact with agents that may cause inflammation of the mouth (such as, alcohol-based mouthwashes). Dentures should fit snugly, not loosely.

E - 49 The nurse is teaching a client how to maintain effective oral health. Which measure does the nurse include in the teaching plan? Select all that apply. a - Regular dental checkups. b - Eating a balanced diet. c - Use of mouthwashes containing alcohol. d - Managing stress as much as possible. e - Ensuring that dentures are slightly loose-fitting.

a - Regular dental checkups. b - Eating a balanced diet. d - Managing stress as much as possible. Regular dental checkups are important, so potential problems can be prevented or attended to promptly. Stress suppresses the immune system, which can increase the client's risk for infections such as Candida albicans. Eating a balanced diet can reduce the risk for dental caries and infections such as C. albicans or stomatitis.Mouthwashes that contain alcohol may irritate tissues and cause inflammation, and should be avoided. Dentures must be in good repair and need to fit properly, not loosely.

E - 50 A client with peptic ulcer disease (PUD) asks the nurse whether licorice and slippery elm might be useful in managing the disease. What is the nurse's best response? a - "No, they probably won't be useful. You should use only prescription medications in your treatment plan." b - "These herbs could be helpful. However, you should talk with your primary health care provider before adding them to your treatment regimen." c - "No, herbs are not useful for managing this disease. You can use any type of over-the-counter drugs though. They have been shown to be safe." d - "Yes, these are known to be effective in managing this disease but make sure you research the herbs thoroughly before taking them."

b - "These herbs could be helpful. However, you should talk with your primary health care provider before adding them to your treatment regimen." The nurse's best response is that although licorice and slippery elm may be helpful in managing PUD, the client must consult his or her primary health care provider before making a change in the treatment regimen.Alternative therapies may or may not be helpful in managing PUD. The client should not use over-the-counter medications without first discussing it with his or her primary health care provider.

E - 50 The nurse and the registered dietitian nutritionist are planning sample diet menus for a client who is experiencing dumping syndrome. Which sample meal is most appropriate for this client? a - Liver, bacon, and onions b - Chicken and white rice c - Chicken salad on whole wheat bread d - Green vegetable salad with buttermilk ranch dressing

b - Chicken and white rice Chicken and rice is the most appropriate sample meal for this client. It is the only selection suitable for the client who is experiencing dumping syndrome because it contains high protein without the addition of milk or wheat products. The client with dumping syndrome should not have much mayonnaise, onions, or buttermilk ranch dressing. Buttermilk dressing is made from milk products. The client may have whole wheat bread only in very limited amounts.

The nurse is assessing a client who is suspected of having early gastric cancer. What signs and symptoms would the nurse expect? Select all that apply. a - Fatigue b - Feeling of fullness c - Dyspepsia d - Weakness e - Weight loss f - Nausea and vomiting

b - Feeling of fullness c - Dyspepsia The client who has early gastric cancer usually has no or few signs and symptoms, but may have dyspepsia and a feeling of fullness. More distressing changes are manifested when the cancer becomes more advanced.

E - 50 The nurse is caring for a client who has a gastric ulcer. For which potentially life-threatening complication would the nurse monitor for this client? a - Hypokalemia b - Hemorrhage c - Nausea and vomiting d - Infection

b - Hemorrhage Clients who have gastric ulcers are particularly at risk for upper GI bleeding, or hemorrhage. They may also experience nausea and vomiting causing dehydration. However, hemorrhage is most serious.

E - 49 A client with oral cancer who is to have a radical neck dissection reports being depressed. What is the nurse's priority response? a - Suggest seeking support from a community group. b - Listen to the client's concerns. c - Explain the grieving process. d - Reassure that it is normal to feel depressed about the diagnosis.

b - Listen to the client's concerns. The nurse's priority response is to listen to the client and allow him or her to process feelings.After the client has processed feelings, he or she is more open to learning about the normalcy of feeling depressed about this diagnosis, understanding the grieving process, and considering referral to a community group of clients undergoing a similar experience.

E - 49 The nurse is caring for a client diagnosed with aphthous ulcers. Which food will the nurse recommend that the client avoid? Select all that apply. a - Apples b - Pasta c - Baked potato d - Nuts e - Cheese

b - Pasta c - Baked potato d - Nuts e - Cheese Aphthous ulcers (canker sores) are small, shallow lesions that develop on the soft tissues in the mouth or at the base of the gums. The nurse tells the client that certain foods such as cheese, nuts, potatoes, and foods containing gluten (like pasta) may trigger allergic responses that cause aphthous ulcers and should be avoided.Apples and bananas are not acidic and do not trigger allergic responses that cause aphthous ulcers.

E - 49 A client who has undergone surgery and completed radiation therapy to treat oral cancer reports persistent dry mouth. What will the nurse teach this client about managing this symptom? a - This condition is common but is temporary. b - Use saliva substitutes, especially when eating dry foods. c - This indicates a complication of therapy. d - Use lozenges and hard candies to prevent dry mouth.

b - Use saliva substitutes, especially when eating dry foods. Xerostomia is a common effect of oral irradiation. Clients should be advised to use saliva substitutes.The condition is common, but often permanent. Lozenges and hard candies are not as effective as saliva substitutes. Dry mouth is a side effect of therapy, not a symptom of complications. Taking frequent sips of water is the preferred method of treating xerostomia during radiation therapy.

E - 49 A client reports ongoing episodes of "heartburn." Which food will the nurse recommend that the client eliminate from the diet? a - Steak b - Carrots c - Chocolate d - Popcorn

c - Chocolate Foods that decrease esophageal sphincter pressure, such as fatty foods, caffeine, and chocolate, should be avoided.Steak, carrots, and popcorn do not decrease esophageal sphincter pressure.

E - 50 The nurse is caring for a client with peptic ulcer disease who has been vomiting profusely at home before coming to the emergency department. For which acid-base imbalance will the nurse expect for this client? a - Respiratory acidosis b - Respiratory alkalosis c - Metabolic alkalosis d - Metabolic acidosis

c - Metabolic alkalosis Gastric contents are rich in acid (hydrogen and chloride ions). When this fluid is lost through vomiting, the client has less acid causing an alkalotic state.

E - 49 When caring for a client with oral cancer who has developed stomatitis as a complication of radiation and chemotherapy, which action does the nurse delegate to the assistive personnel (AP)? a - Instruct how to use nystatin oral rinses. b - Assist with making appropriate dietary choices that do not irritate tissues. c - Provide oral care using a soft toothbrush. d - Inspect the oral mucosa for evidence of oral candidiasis.

c - Provide oral care using a soft toothbrush. Providing oral care using a soft toothbrush for a client with oral lesions is an appropriate assignment for an AP.Assessments, client teaching, and assisting clients with oral problems in making appropriate dietary choices are the responsibilities of licensed nursing staff.

E - 49 The nurse is providing instructions to a client with a history of stomatitis. Which instructions does the nurse include in the teaching plan? a - Encourage the client to eat acidic foods to decrease bacteria. b - Mouth care should be performed twice daily at the maximum. c - Rinse the mouth frequently with warm saline or sodium bicarbonate. d - Use a medium-bristled toothbrush for oral care.

c - Rinse the mouth frequently with warm saline or sodium bicarbonate. Rinsing the mouth frequently with warm saline or sodium bicarbonate or a combination of the two decreases inflammation and pain.Acidic foods increase inflammation and should be avoided. Mouth care should be done after each meal and as often as needed, at the minimum of twice daily. If stomatitis is not controlled, mouth care may have to be done every 2 hours or more frequently. A soft toothbrush, not medium-bristled one, needs to be used for oral care.

E - 49 Which food does the nurse teach a client undergoing chemotherapy with secondary stomatitis to avoid? a - Broiled fish b - Ice cream c - Salted pretzels d - Scrambled eggs

c - Salted pretzels Salty foods like pretzels can further irritate ulcers in the client's mouth, causing pain.Cool or cold foods and foods high in protein, such as fish, eggs, and ice cream, may be included in the diet of the client with stomatitis.

E - 49 The community nurse is discussing risk factors for esophageal cancer with a group of clients. Which client behavior requires further teaching? a - Eats a small snack each night before bedtime. b - Walks at the shopping mall three times weekly. c - Smokes 1/3 of a pack of cigarettes daily. d - Elevates pillows at night.

c - Smokes 1/3 of a pack of cigarettes daily. Tobacco use is one of the primary risk factors for esophageal cancer. This client behavior requires teaching about lifestyle risks that could increase the risk for esophageal cancer.Other reported client behaviors are acceptable and do not increase risk for esophageal cancer.

E - 50 The nurse is planning health teaching about omeprazole for a client who has acute gastritis. What would the nurse include in the health teaching? a - Crushing the drug and mixing in applesauce b - Avoiding alcohol while taking this drug c - Taking the drug 30 minutes before a meal d - Taking the drug when the client has gastric pain

c - Taking the drug 30 minutes before a meal This drug is a proton pump inhibitor and is activated by the presence of food in the stomach. Therefore, it should be taken before a meal.

E - 50 The nurse is teaching a client how to prevent recurrent chronic gastritis symptoms before discharge. Which statement by the client demonstrates a correct understanding of the nurse's instruction? a - "I will need to take vitamin B12 shots for the rest of my life." b - "I should eat small meals about six times a day." c - "It is okay to continue to drink coffee in the morning when I get to work." d - "I should avoid alcohol and tobacco of any type."

d - "I should avoid alcohol and tobacco of any type." The client's statement that he or she needs to avoid alcohol and tobacco shows that the client correctly understands the nurse's instructions.The client also needs to eliminate caffeine from the diet. The client will need to take vitamin B12 shots only if he or she has pernicious anemia. The client would also not eat six small meals daily. This practice may actually stimulate gastric acid secretion.

E - 49 The nurse is assessing a client who reports having a history of gastroesophageal reflux disease (GERD). Which assessment finding does the nurse report to the primary health care provider? a - "My family likes to eat small meals every 3 to 4 hours throughout the day." b - "When I buy meat, I ask for the leanest cut that is available." c - "I quit smoking 6 months ago." d - "Sometimes I wake up gasping for air in the middle of the night."

d - "Sometimes I wake up gasping for air in the middle of the night." Gasping for air upon waking in the middle of the night can be a sign of sleep apnea; the nurse must report this finding to the primary health care provider. Often patients who have one condition (sleep apnea or GERD) also experience the other.Quitting smoking 6 months ago, eating small meals, and eating lean meats are favorable findings that do not need to be reported.

E-49 A client has recently developed acute sialadenitis. Which intervention does the nurse include in this client's care? a - Request a prescription for an opioid to manage pain. b - Restrict fluids. c - Apply cold compresses. d - Massage the salivary gland.

d - Massage the salivary gland. Sialadenitis is inflammation of a salivary gland. The salivary gland is massaged to stimulate the flow of saliva. This is done by milking the edematous gland with the fingertips toward the ductal opening.To promote the flow of saliva, warm (not cold) compresses are applied to the affected salivary gland. Pain from this condition is managed with NSAIDs, not opioids. Hydration promotes salivary flow.


Conjuntos de estudio relacionados

NU471 Week 4 EAQ #3 Evolve Elsevier: Safety - Mastery Level Target: Level 3

View Set

Psychosocial Integrity (HESI Compass Practice Questions)

View Set

Selective and Differential Media

View Set

Patho Final - Ch 43, Patho Final - CH 44, Patho Final - CH 45, Patho Final - Ch 46, Patho Final - Ch 47, Patho Final - Ch 49, Patho Final - CH 48, Patho Final - Ch 52, Patho Final - Ch 51, Patho Final - Ch 50

View Set

QA interview questions + follow up (not priority but better to learn)

View Set

Ch. 1 What Is Organizational Behavior

View Set

Principles of Marketing test 3 (warm ups)

View Set

Texas LT care and partnership policies

View Set

NUR 423 Exam 2 Book/quiz question practice

View Set

Chapter 24-The Fetal Head and Brain

View Set